Vous êtes sur la page 1sur 131

ENDI 2010 1

Security K Wave 1

SECURITY KRITIK
Security kritik 1nc 3 Impact links:
economic competitiveness link 56
humanitarian assistance link 57
LINKS: environmental security link 58
Security generic links: environ. security t/o solvency 59
Threat construction link 7
Realism/ psychological motives link 9 critical inequality link 60
Threat construction link – positivism 10 inequality link - discrimination 63
Threats to homeland link 11 inequality link - discrimination 64
Crisis management link 12 inequality link – international / capitalism 65
Securitization of non-military link 13
Link helpers 66
Agent links:
Sovereignty link 14 IMPACTS:
Hegemony link 15 Securitization bad – kills criticism 67
Soft power link 18 Securitization bad – violence 68
Governmentality link 19 Securitization bad - resentment 69
International norms/Rulemaking link 20 Heg bad-imperialism 70
Myths impact 71
Representation links Kritik turns case-war 72
Terrorism discourse link 21
AT: Terrorists are irrational 24 ALTERNATIVE(S):
Borders link 25 2NC Alt solves 73
Borders link - realism 26 Alt solves - violence 74
AT: Borders key to security 27 Alt solves - sovereignty 75
Alt solves - Epistemology 76
Area links: Alt solves – metanarratives 77
South China Sea link 29 Environ security alt - exclude securitization 78
Middle East Link 30
North Korea Link 33 FRAMEWORK:
China link- threat 35 Discourse shapes reality/policy 79
China link – economy/competitiveness 37 Discourse shapes reality – metaphor 81
China link – hostile rise / power vacuum 38 security = speech act 82
Russia link 39 Discourse 1st 84
AT: Rational actor 85
Epistemology links:
Peace link 40 2NC:
Positivism/empiricism link 41 AT: No impact to representation 86
Positivism link - state 42 AT: Perm- positivism 87
Root cause link 43 AT: Case outweighs 89
Neorealism link 44 AT: Predictions/Scenario planning good 91
AT: Realism inevitable 92
Prolif specific: AT: Realism good 93
proliferation link 45 AT: Realism good: nuclear war 96
proliferation link - state 47 AT: Realism good- Hobbes 97
proliferation link – weapon label 48 AT: realism good - critical reasons 98
proliferation link –stability 49 AT: Securitization key to action 100
proliferation link – ‗wildfire‘ / cancer metaphor 50 AT: Post-structuralism bad 101
proliferation link – weapons spread 51 AT: Criticisms that make fun of post-structuralism 103
proliferation link – peaceful/ military distinction 52 AT: Environmental securitization good 105
proliferation link – rogue states/ loose nukes 53 AT: Link turns – aff stops seeing x as enemy 106
proliferation link – rogues/ monitoring 54 AT: Link turn – we establish alliances 107
proliferation discourse turns case 55 AT: Kritik is ideological 108
ENDI 2010 2
Security K Wave 1

******AFF****** 109
Framework – AT: Discourse first 110
AT: Reps first 112

Positivism good 113


AT: Scenario planning bad 115
AT: Predictions Fail 116

2AC Cede The Political 117


AT: State links 118
AT: Threat construction 119
AT: psychology links 120
AT: Middle East Link 121
AT: Terror Link 122

2AC impact calc - Consequences First 123


2AC impact calc – AT: Value to life 124
AT: Value to life 125
AT: Structural Violence Impact 126

2AC-Permutation 127
Critical realism perm 129

2AC- Alt fails 130


Realism good 131
ENDI 2010 3
Security K Wave 1

SECURITY KRITIK 1NC

Security is a speech act that manufactures low probability threats and worst case scenarios in
order to build up the state‘s defenses and defend its territory
Lipschutz 1998 [Ronnie, prof of politics at UC Santa Cruz, ―Negotiating the Boundaries of Difference and Security at
Millennium's End,‖ On Security, ed. Ronnie Lipschutz, http://www.ciaonet.org/book/lipschutz/index.html]

What then, is the form and content of this speech act? The logic of security implies that one political actor must be protected from the
depredations of another political actor. In international relations, these actors are territorially defined, mutually exclusive
and nominally sovereign states. A state is assumed to be politically cohesive, to monopolize the use of violence within the
defined jurisdiction, to be able to protect itself from other states, and to be potentially hostile to other states. Self-protection
may, under certain circumstances, extend to the suppression of domestic actors, if it can be proved that such actors are acting in a manner
hostile to the state on behalf of another state (or political entity). Overall, however, the logic of security is exclusionist: It proposes to exclude
developments deemed threatening to the continued existence of that state and, in doing so, draws boundaries to discipline the behavior of
those within and to differentiate within from without. The right to define such developments and draw such boundaries is, generally speaking,
the prerogative of certain state representatives, as Wæver points out. 3 Of course, security, the speech act, does draw on material conditions "out there." In
particular, the logic of security assumes that state actors possess "capabilities," and the purposes of such capabilities are interpreted as part of the speech
act itself. These interpretations are based on indicators that can be observed and measured--for example, numbers of tanks in the field, missiles in silos,
men under arms. It is a given within the logic--the speech act--of security that these capabilities exist to be used in a threatening fashion--either for
deterrent or offensive purposes--and that such threats can be deduced, albeit incompletely, without reference to intentions or, for that matter, the domestic
contexts within which such capabilities have been developed. Defense analysts within the state that is trying to interpret the meanings
of the other state's capabilities consequently formulate a range of possible scenarios of employment, utilizing the most
threatening or damaging one as the basis for devising a response . Most pointedly, they do not assume either that the
capabilities will not be used or that they might have come into being for reasons other than projecting the imagined threats .
Threats, in this context, thus become what might be done, not, given the "fog of war," what could or would be done, or the fog of
bureaucracy, what might not be done. What we have here, in other words, is "worst case" interpretation. The "speech act" security thus
usually generates a proportionate response , in which the imagined threat is used to manufacture real weapons and
deploy real troops in arrays intended to convey certain imagined scenarios in the mind of the other state .
Intersubjectivity, in this case, causes states to read in others, and to respond to, their worst fears . It is important to recognize that, to
the extent we make judgments about possibilities on the basis of capabilities, without reference to actual intentions, we are trying to imagine how those
capabilities might be used. These imagined scenarios are not, however, based only on some idea of how the threatening actor
might behave; they are also reflections of what our intentions might be, were we in the place of that actor, constructing
imagined scenarios based on what s/he would imagine our intentions might be, were they in our place. . . . and so on, ad
infinitum . Where we cut into this loop, and why we cut into the loop in one place and not another, has a great deal to do with where we start in our
quest to understand the notion of security, the speech act.
ENDI 2010 4
Security K Wave 1

SECURITY KRITIK 1NC

The affirmative’s securitizing representations reduce human freedom and agency to a calculation- this is
uniquely dehumanizing and destroys the value to life
Dillon 1996 (Michael is a professor of politics at the University of Lancaster, Politics of Security, p. 26)
Everything, for example, has now become possible. But what human being seems most impelled to do with the power of
its actions is to turn itself into a species; not merely an animal species, nor even a species of currency or consumption
(which amount to the same thing), but a mere species of calculation. For only by reducing itself to an index of calculation
does it seem capable of constructing that oplitical arithmetic by which it can secure the security globalised Western thought
insists upon, and which a world made uncreasingly unpredictable by the very way human being acts into it now seem to
require. Yet, the very rage for calculability which securing security incites is precisely also what reduces human
freedom, inducing either despair or the surender of what is human to the de-humanising calculative logic of what seems to
be necessary to secure security. I think, then, that Hannah Arendt was right when she saw late modern humankind caught in
a dangerous world-destroying cleft between a belief that everything is possible and a willingness to surender itself to so-
called laws of necessity (calculability itself) which would make everything possible. That it was, in short, characterized by
a combination of reckless omnipotence and reckless despair. But I also think that things have gone one stage further- the
surrender to the necessity of realising everything that is possible- and that this found its paradigmatic expression for
example in the deterrent security policies of the Cold War; where everything up to and inclduing self-immolation not only
became possible but actually necessary in the interests of (inter)national security. The logic persists in the metaphysical
core of modern politics- the axiom of Inter-state security relations, popularized for example, through strategic discourse-
even if the details have changed.

And, treating security as an a priori legitimizes the WMD suicide pact and billions of deaths
Der Derian 1998 [James, prof of political science at Brown, ―The Value of Security: Hobbes, Marx, Nietzsche, and
Baudrillard On Security,‖ ed. Ronnie Lipschutz, http://www.ciaonet.org/book/lipschutz/index.html]

No other concept in international relations packs the metaphysical punch, nor commands the disciplinary power of
"security." In its name, peoples have alienated their fears, rights and powers to gods, emperors, and most recently, sovereign states,
all to protect themselves from the vicissitudes of nature --as well as from other gods, emperors, and sovereign states. In its name,
weapons of mass destruction have been developed which have transfigured national interest into a security dilemma
based on a suicide pact. And, less often noted in international relations, in its name billions have been made and millions killed
while scientific knowledge has been furthered and intellectual dissent muted . We have inherited an ontotheology of security, that
is, an a priori argument that proves the existence and necessity of only one form of security because there currently happens to be a
widespread, metaphysical belief in it. Indeed, within the concept of security lurks the entire history of western
metaphysics, which was best described by Derrida "as a series of substitutions of center for center" in a perpetual search for the "transcendental
signified." 1 From God to Rational Man, from Empire to Republic, from King to the People--and on occasion in the reverse direction as well, for history is
never so linear, never so neat as we would write it--the security of the center has been the shifting site from which the forces of
authority, order, and identity philosophically defined and physically kept at bay anarchy, chaos, and difference. Yet the center,
as modern poets and postmodern critics tell us, no longer holds. The demise of a bipolar system, the diffusion of power into new political, national, and
economic constellations, the decline of civil society and the rise of the shopping mall, the acceleration of everything --transportation, capital and
information flows, change itself--have induced a new anxiety. As George Bush repeatedly said--that is, until the 1992 Presidential election went into full
swing--"The enemy is unpredictability. The enemy is instability." 2 One immediate response, the unthinking reaction, is to master this anxiety
and to resecure the center by remapping the peripheral threats. In this vein, the Pentagon prepares seven military scenarios for future
conflict, ranging from latino small-fry to an IdentiKit super-enemy that goes by the generic acronym of REGT ("Reemergent Global Threat"). In the
heartlands of America, Toyota sledge-hammering returns as a popular know-nothing distraction. And within the Washington beltway, rogue powers such
as North Korea, Iraq, and Libya take on the status of pariah-state and potential video bomb-site for a permanently electioneering elite.
ENDI 2010 5
Security K Wave 1

SECURITY KRITIK 1NC

The alternative is to reject the affirmative’s appeals to securitization. Questioning the conditions
of possibility for power relations created through the affirmative’s representations refuses to
participate in calculative and depoliticizing worst case scenario predictions.
Edkins 1999 [Jenny, Senior Lecturer in International Politics at the University of Wales Aberystwyth, Postructuralism and
International Relations: Bringing the Political Back In, p. 1-3]
Ironically, what we call "politics" is an area of activity that in modern Western society is "depoliticized" or "technologized."
These two terms are more or less synonymous (as far as my usage here goes), but the latter is perhaps more useful as a term because of the sense it
conveys that what is going on is something positive. We are not talking about an absence of the political through some sort of lapse or mistake but an
express operation of depoliticization or technologization: a reduction to calculability. In this context ideology is the move that
conceals the depoliticization of politics and hides the possibility-the risks-of "the political." Technologization has its
dangers, too, and one of the fields where its perils can be seen is international politics. As examples, I examine briefly the
technologization of famine relief and the notion of securitization as a form of extreme depoliticization. In the final section of this chapter, I outline how the
authors whose work I discuss later in the book see processes of technologization and depoliticization. POLITICS AND THE POLITICAL The
distinction I employ here between "politics" and "the political" is similar to that between what is sometimes called a "narrow" meaning
of the political and a broader one. In the narrow sense, the political is taken to be that sphere of social life commonly called "politics":
elections, political -parties, the doings of governments and parliaments, the state apparatus. and in the case of "international politics,"
treaties, international agreements, diplomacy, wars, institutions of which states are members (such as the United Nations), and the actions of statesmen and
-women. As James Donald and Stuart Hall point out, what gets to be counted as politics in this narrow form is not in any sense given. It is the
result of contestation. It is ideological, contingent on a particular organization of the social order, not natural .6 Donald and Hall
refer to the struggle in the 1970s and 1980s by the women's movement to extend the range of politics to include, for example, relations of power within the
home or between men and women more broadly. "The personal is political" was their slogan. A similar extension of international politics has been
advocated by Cynthia Enloe, this time with the phrase "the personal is international. "7 In other words, the question of what gets to count as "politics" (in
the narrow sense) is part of "the political" (in the broader sense): It is a political process. Or in Fred Dallmayr's words, "Whereas politics in the narrower
sense revolves around daY7to-day decision making and ideological partisanship . . . "the political" refers to the frame of reference within which actions,
events, and other phenomena acquire political status in the first place."8 In the broader sense, then, "the political" has to do with the
establishment of that very social order which sets out a particular, historically specific account of what counts as politics and
defines other areas of social life as not politics. For Claude Lefort, the political is concerned with the "constitution of the social space, of
the form of society."9 It is central to this process that the act of constitution is immediately concealed or hidden: Hence, "the political is ... revealed,
not in what we call political activity, but in the double movement whereby the mode of institution of society appears and is obscured."10 How does this
relate to the link that is generally made between "power" and the political? Following Lefort again, "the phenomenon of power lies at the centre of
political analysis," but this is not because relations of power should be seen as autonomous and automatically defining "politics." Rather, it is because "the
existence of a power capable of obtaining generalised obedience and allegiance implies a certain type of social division and articulation, as well as a
certain type of representation ... concerning the legitimacy of the social order."" In other words, what is important about power is that it
establishes a social order and a corresponding form of legitimacy. Power, for Lefort, does not "exist" in any sort of naked form, before
legitimation: Rather, the ideological processes of legitimation produce certain representations of power. For a political analysis,
in the broadest sense, what needs to be called into question are the conditions of possibility that produced or made
conceivable this particular representation of power. The question is, "What change in the principles of legitimacy, what
reshaping of the system of beliefs, in the way of apprehending reality, enabled such a representation of power to
emerge?"12
ENDI 2010 6
Security K Wave 1

SECURITY KRITIK 1NC

Language matters- debating the affirmative’s representations is key to overcoming dominant


descriptions of agents and objects in international relations

Der Derian 98 (James, a Watson Institute research professor of international studies and directs the
Information Technology, War, and Peace Project and the Global Media Project,
“International/Intertextual Relations: Postmodern Readings of World Politics‖, Lexington Books, p.13)

Once we give adequate recognition to the texts within which the world emerges and provided an understanding of
politics that focuses on such impositions of meaning and value , we can appreciate the intimate relationship between
textual practices and politics. It is the dominant, surviving textual practices that give rise to the systems of meaning
and value from which actions and policies are directed and legitimated . A critical political perspective is, accordingly, one
that questions the privileged forms of representation whose dominance has led to the unproblematic acceptance of subjects,
objects, acts, and themes through which the political world is constructed. In as much as dominant modes of understanding
exist within representational or textual practices, criticism or resistant forms of interpretation are conveyed less through an
explicitly argumentative form than through a writing practice that is resistant to familiar modes of representation , one that is
self-reflective enough to show how meaning and writing practices are radically entangled in general or one that tends to denaturalize familiar reunites by
employing impertinent grammars and figurations, by, in short, making use of an insurrectional textuality. To appreciate the effects of this
textuality, it is necessary to pay special need to language, but this does not imply that an approach emphasizing textuality
reduces social phenomena to specific instances of linguistic expression. To textualize a domain of analysis is to recognize, first of all,
that any "reality" is mediated by a mode of representation and, second, that representations are not descriptions of a world of
facility, but are ways of making facility. Their value is thus not to be discerned in their correspondence with something, but
rather in the economies of possible representations within which they participate. Modes of reality making are therefore worthy of
analysis in their own right. Such analysis can be a form of interpretation in which one scrutinizes the effects on behavior or policy that the
dominance of some representational practices enjoy, or it can be a form of critique in which one opposes prevailing
representational practices with alternatives. Therefore, a concern with textuality must necessary raise issues about the texuality
(the meaning and value effects) of the language of inquiry itself. In order, then, to outline the textualist approach, we must develop
further our understanding of the language analysis.
ENDI 2010 7
Security K Wave 1

THREAT CONSTRUCTION LINK

Security threats are created through acts of interpretation—representations enable securitizing


actions
Mutimer 2000 [David, associate professor of political science at York University and Deputy Director of the Center for
International and Security Studies, The Weapons State, pg 16-17]

A further point is to be made concerning Campbell's work. The focus of Writing Security is not, in fact, on the way in which danger is interpreted- the
manner by which the interpretation of risk and the consequent creation of threat occur. Rather, Campbell's argument shows the way in which
the interpreting subject-in this instance the United States-is itself created by those acts of identifying danger. If we can
accept that both the threats and the subjects of international security are created in acts of interpretation, it should be clear
that the interests those subjects pursue are also consequences of these same acts . It would be difficult to argue that interests
remain fixed when the bearer of those interests does not. Jutta Weldes has made the case with respect to interests: In contrast to the realist conception
of "national interests" as objects that have merely to be observed or discovered, then, my argument is that national interests are social constructions created
as meaningful objects out of the intersubjective and culturally established meanings with which the world, particularly the international system and the
place of the state in it, is understood. More specifically, national interests emerge out of the representations . .. through which state
officials and others make sense of the world around them . 13 These "representations through which state officials and others make sense of
the world around them" are central to my argument in this book. Rather than take the objects of study as given, I ask questions about the construction of a
particular object, a particular set of identities and interests, and the specific practices through which proliferation is confronted. The key to answering these
questions is to identify the way in which the problem is represented or, to use the language I deploy later, the image that is used to frame the issue in
question. This image serves to construct the object of analysis or policy, to identify the actors, and to define their interests.
It is therefore the image that enables the practices through which these actors respond to the problem of
proliferation.

Constructing threats necessitates an other to fear and respond to


Lipschutz 95- Professor of Politics and Associate Director of the Center for Global, International and
Regional Studies at the UCSC ( Ronnie D. Lipshutz: On Security Pg. 8-9 1995)

Conceptualizations of security-from which follow policy and practice-are to be found in discourses of


security. These are neither strictly objective assessments nor analytical constructs of threat, but rather
the products of historical structures and processes, of struggles for power within the state, of conflicts
between the societal groupings that inhabit states and the interests that besiege them. Hence, there are not
only struggles over security among nations, but also struggles over security among notions. Winning the
right to define security provides not just access to resources but also the authority to articulate new
definitions and discoursed of security, as well. As Karen Liftlin points out, ―As determinants of what can and cannot be thought,
discourses delimit the range of policy options, thereby functioning as precursors to policy outcomes … The supreme power is the power to delineate the
boundaries of thought – an attribute not so much of specific agents as it is of discursive practices. These discourses of security, however clearly articulated,
nonetheless remain fraught with contradictions, as the chapters in this volume make clear. How do such discourses begin? In his investigation of historical
origins of the concept, James Der Derian (Chapter 2: ―The Value of Seurity: Hobbes, Marx, Nietzsche, and Baudrillard‖) points out that, in the past, security
has been invoked not only to connote protection from threats, along the lines of the conventional
definition, but also to describe hubristic overconfidence as well as a bond or pledge provided in a financial
transaction. To secure oneself is, therefore, a sort of trap, for one can never leave a secure place without incurring risks. (Elsewhere, Barry Buzan has
pointed out that ―There is a cruel irony in [one] meaning of secure which is ‗unable to escape‘. Security, moreover, is meaningless
without an ―other‖ to help specify the conditions of insecurity. Der Derian, citing Nietsche, points out
that this ―other‖ is made manifest through differences that create terror and collective resentment
of difference – the state of fear – rather than a preferable coming to terms with the positive
potential of difference.
ENDI 2010 8
Security K Wave 1

THREAT CONSTRUCTION LINK

The term security is used to allow states to use whatever means necessary to eliminate the threats
they have created
Lipschutz 95 (Ronnie D, a Professor of Politics and Codirector of the Center for Global, International, and Regional Studies at the University of
California–Santa Cruz , On Security, p. 9-10)
Operationally, however, this means: In naming a certain development a security problem, the ―state‖ can claim a special right, one
that will, in the final instance, always be defined by the state and its elites. Trying to press the kind of unwanted fundamental political chance on a ruling
elite is similar to playing a gam in which one‘s opponent can change the rules at any time s/he likes. Power holders can always try to use the
instrument of securitization of an issue to gain control over it. By definition, something is a security problem when the elites
declare it to be so: and because the End of this Institution [the Leviathan, the Sovereign], is the Peace and Defense of them all; and whosoever has
the right to the End, has right to the Means; it belongeth of Right, to whatsoever Man, or Assembly that hath the Soveraignty, to be Judge
both of the meanes of Peace and Defense; and also of the hindrances, and disturbances of the same; and to do whatsoever he shall think
necessary to be done, both before hand, for the preserving of Peace and Security, by prevention of Discord at home and thus, that those who
administer this order can easily use it for specific, self-serving purposes is something that cannot be easily avoided‖. What then is security? With the help
of language theory, we can regard ―security‖ as a speech act. In this usage, security is not of interest as a sign that refers to somethingmore rea; the
utterance itself is the act. By saying it, something is done (as in betting, giving a promis, naming a ship). By uttering ―security‖, a state-
representative moves a particular development into a specific area, and thereby claims a special right to use whatever means are necessary
to block it. The clearest illustration of this phenomenon- on which I will elaborate below- occurred in Central and Eastern Europe during the Cold War,
where ―order‖ was clearly, systematically, and institutionally linked to the survival of the system and its elites. Thinking about chainge in the East-West
relations and/or in Eastern Europe throughout this period meant, therefore, rying to bring about change without generating a ―securitization‖ response by
elites, which would have provided the pretext for acting against those who had overstepped the boundaries of the permitted. Consequentally, to ensure that
this mechanism would not be triggered, actors had to keep their challenges below a certain thershold and/or through the political process-wheter national
or international- have the threshold negotiated upward. As Egbert Jahn put it, the task was to turn threats into challenges; to move developments from the
sphere of existential fear to one where they could be handled by ordinary means, as politics, economy, culture, and so on. As part of this exercise, a crucial
political and theoretical issue became the definition of ―intervention‖ or ―interference in domestic affairs‖, whereby change-oriented agents tried, through
international law, diplomacy, and various kinds of politics, to raise the threshold and make more interaction possible.

The will to security is an incitement to violence- only a break from the politics of security gives
meaning to international relations
Dillon 1996 (Michael is a professor of politics at the University of Lancaster, Politics of Security, p. 19)
We now know that neither metaphysics nor our politics of security can secure the security of truth and of life which was their
reciprocating raison d‘ etre (and, rason d‘ etat). More importantly, we now know that the very will to security-the will to power of sovereign
presence in both metaphysics and modern politics- is not a prime incitement to violence in the Western tradition of thought, and to
the globalization of its (inter)national politics, but also self defeating; in that it does not in its turn merely endanger, but
actually engenders danger in response to its own discursive dynamic. One does not have to be persuaded of the destinal sending of
Being, therefore, to be persuaded of the profundity- and of the profound danger- of this modern human condition. That, then, is why the crisis of Western
though is as much a fundamental crisis of (inter)national politics, as the criss of (inter)national politics is a crisis of thought. Moreover, that is why in
doubting the value of security, and doubting in a Nietzschean mode better than Descartes, we are also enjoined by the circumstances of this critical
conjunction of the philosophical and the political to doubt metaphysical truth. For the political truth of security is the metaphysical truth of
correspondence and adequation in declension to mathesis; the mere, but rigorously insistent, measuration of calculabilty . To
bring the value of security into question in the radical way required by the way it now, ironically, radically endangers us, correspondingly requires that we
attend to metaphysics‘ own continous process of deconstruction. In doing this, however, we go beyond mere doubting- which, after all, is the mere
counterpart of the desire for certainty- and find non-apocalyptic ays of affirming and so continuing to enjoy and celebrate (in)security; that is to say human
being‘s own obligatory freedom. Ultimately, now, our (inter)national politics of security is no longer even distinguished or
driven by humanistic considerations. It is a security simply ordering to order. But it is only by virtue of the fact that our
(inter)national politics of security has come to this end that we can in fact begin to consider the relationship between its end and its beginnning. Through
this we do not, in a sense, go back to anything at all. Neither does this turn disguise some covert nostalgia for a phantom past. Rather, attention is turned
towards consideration of what is entailed in the preparation and inception of continuous new political growth. This is also why, at the limit, it is useful to
think about these origins and limits again. Not because they hold an answer that is now lost but because, antecedent to metaphysics, they make us think
about the very liminal character of origins and limits, of the relationship which obtains between them, and of what proceeds from them, in ways that are
not utterly determined by metaphysics. That way we may get some clues to some ways of thinking that are not metaphysical; nor, indeed, pre-
metaphysical, because we cannot be pre-metaphysical at the end of metaphysics. What happens, instead, is that the whole question of emergence
and origination, of the very possibility of repeating ourselves, opens-up again; specifically in the sense of the historical
possibilities of the obligatory freedom of human being now terminally endangered globally by its very own (inter)national
‗civilising‘ practices.
ENDI 2010 9
Security K Wave 1

REALISM/ PSYCHOLOGICAL MOTIVES LINK

We should critically analyze the very idea of beliefs and psychological motivations in order to de-
stabilize the modern subject
Der Derian 98 (James, a Watson Institute research professor of international studies and directs the
Information Technology, War, and Peace Project and the Global Media Project,
“International/Intertextual Relations: Postmodern Readings of World Politics‖, Lexington Books, p.18)

What poststructuralist approaches have shown so far is that the orthodoxies of our social and political worlds are recreated
in the process of writing, in the style of the texts through which our dominant understandings of the world have been
constructed. And no form of writing is exempt; analysis itself is a textual practice that is intimately related to the political
practices it aims to disclose. In order to exemplify analysis as a form of textual practice, it is useful once again to turn to a
contrast between a textually oriented mode of reading and the more familiar, political psychology. The psychological
approach to international relations, has focused, among other things, on the cognitive components through which
individuals "perceive" aspects of policy. In order to textualize political psychology and, at the same time, demonstrate the
difference in problematization between a psychological and textual approach, we offer a brief reading of what could best be
termed the politics of fear. We begin this reading with the recognition that individuals, in their contributions to the
meanings shaping public life, cannot be understood simply as mentalistic information processors, but rather as socially and
temporally situated beings, connected to each other in a network of practices. This means, among other things, that we must
resist many of modernity's professional and academic discourses that have produced modern "man" as psychological being
(as Pltilip Rieff pointed out a few decades ago). Were we to treat this psychological identity as a fact rather than as a
historically produced text, our analysis would be paralyzed in the same way as are these psychologizing practices. Rather
than focusing on individual beliefs or other cognitive components, then, we argue that it is more enabling to understand
how understandings are situated in domains of practice. Instead of exploring people's beliefs, for example, we can do a
genealogy of belief itself, locating beliefs in the context of the history of practices related to the management of danger.
Beliefs, as an identity for persons, are a kind of data, providing a way of reading the script of modernity, rather than an
analytic device aiding interpretation. To note that modern individuals have "beliefs" is not to take cognizance of a fact
about persons, but to notice the contemporary way of constructing them. By analyzing this practice for constituting the
modern self, we can also move in the direction of disclosing the more cryptic modes of legitimating for public (and
"foreign") policy.
ENDI 2010 10
Security K Wave 1

THREAT CONSTRUCTION – POSITIVISM LINK

Threats are imagined and constructed in order to legitimate existing political dispositions and
responses
Lipschutz 2000, (Ronnie D, a Professor of Politics and Codirector of the Center for Global, International, and Regional Studies at the
University of California–Santa Cruz, After Authority: War, Peace, and Global Politics in the 21 st Century, ch. 3, p. 56)

To return to an earlier question, who constructs and articuates contesting discourses of national security? Among such people are mainstream ―defense
intellectuals‖ and strategic analysts, those individuals who, in sharing a particular political culture, can agree on a common framework for defining
security threats and policy responses (what might be called a security ―episteme‖). While their discourse is constructed around the
interpretation of ―real‖ incoming data, their analysis is framed in such a way as to, first, define the threat as they see it and,
second, legitimate those responses that validate their construction of the threat (see, e.g., Schlesinger, 1991). To repeat: this does not
mean that threats are imaginary. Rather they are imagined and constructed in such a way as to reinforce existing predispositions
and thereby legitimate them. This legitimation, in turn, helps to reproduce existing policy or some variant of it as well as the material
basis for that policy. Finally, we might ask why ―redifine‖ security? Who advocates such an idea? During the 1980‘s at the time this argument was first
made (Ullman, 1983; Matthews, 1989), the individuals compromising this group were an amorphous lot, lacking an integrated institutional base or
intellectual framework (a situation that has slowly changed during the 1990s). Most tended to see consensual definitions and dominant discourses of
security as failing to properly percieve or understand the objective threat environment, but they did not question the logic whereby threats and security
were defined. In other words, the redefiners proposed that the ―real‖ threats to security were different from those that policymakers and defense authorities
were generally concerned about, but the threats were ―really out there‖.
ENDI 2010 11
Security K Wave 1

THREATS TO HOMELAND LINK

Constructing threats to the homeland as coming from the outside is the basis of statist identity
construction and the legitimation of steps toward security
Tickner 95 - Professor of Policy at Holy Cross University (J. Ann Ticknery: International Relations
Theory Today, p. 189)

When nationalsecurity is defined negatively, as protection against outside military threats, the sense of threat is
reinforced by the doctrine of state sovereignty, which strengthens the boundary between a secure
community and a dangerous external environment. For this reason, many critics of realism claim that, if security is to start with the
individual, its ties to state sovereignty must be severed. While E. H. Carr argued for he retention of the nation-state to satisfy people's need for identity, those
who are critical of state-centric analysis point to the dangers of a political identity constructed out of exclusionary practices. In
the present
international system, security is tied to a nationalist political identity which depends on the construction of
those outsides as 'other' and therefore dangerous . (Walker 1990) David Campbell suggests that security
the boundaries of this statist identity demands the construction of 'danger' on the outside: Thus, threats to
security in conventional thinking are all in the external realm. Campbell claims that the state requires
this discourse of danger to secure its identity and legitimation which depend on the promise of security for
its citizens. Citizenship becomes synonymous with loyalty and the elimination of all that is foreign.
Underscoring this distinction between citizens and people reinforced by these boundary distinctions,
Walker argues that not until people, rather than any citizens, are the primary subjects of security can a truly
comprehensive security be achieved.
ENDI 2010 12
Security K Wave 1

CRISIS MANAGEMENT LINK

Realist decisionmaking to protect the homeland shifts into crisis management, drawing the
boundaries between self and other and quelling political opposition
Der Derian 98 (James, a Watson Institute research professor of international studies and directs the
Information Technology, War, and Peace Project and the Global Media Project, Textual Strategies
of the Military, “International/Intertextual Relations: Postmodern Readings of World Politics‖, Lexington Books,
p.101)

Strategic discourse is the mode of coordinating and disciplining that peacelessness that reigns without, beyond the borders
of the sovereign state: "over there" in bordering or far distant regions where strange forces of otherness well u p to
challenge domestic order. Like the popular television show of the mid-1960s, "Get Smart," the realist bifurcates domestic "control" and foreign
"chaos." This arises from more general dichotomies such as "the self ' and "the other," of the one and the many, 9 of domestic
order and international anarchy, all of which require that politics stop at the water's edge and that loyalty reign
supreme at home lest the forces of disorder be emboldened. Traditionally, realism has negotiated this political terrain in terms of two
strategies. The first is a kind of statist monism. Here we find a thoroughly articulated political apparatus that presides over civil society, 10 an architecture
of bureaucratic Leviathanism that in its absolutist form was celebrated as the embodiment o f domestic reason and interests writ large. Out of this arose a
set of practices that have continued to exercise a decisive hold upon the theoretical imaginations of contemporary realists. I
speak here of the various means by which the state is supposed to maximize the well being of an undifferentiated populace. Mercantilism; pursuing the
national interest; defending national security: these are the economic, political, and military spheres attended to by the state.
Of course, there are people who populate the state: real-life kings and queens, cabinet members, diplomats, and statesmen.
Situated atop, presiding over their sovereign realm, they alone are accorded a freedom of action that is quite literally heroic in its scope. Witness the
gallant, globetrotting Kissingerian figure who has transcended the limits of bureaucratic structure and circumvented all manner of domestic accountability.
II And this is the second strategy of realism: the military genius, the visionary statesman. The IR struggles to secure the well being of their
polity occupy the stage upon which is played out the drama of the realist tradition. Contemporary neorealist has modified this
somewhat, rendering more austere and technocratic the exercise of state power. Domestically, we have witnessed the advent of decision-
making inquiries in a conceptual at tempt to understand the formerly posited unity o f a monolithic realist state.
Internationally, one finds a proliferation of techno strategies and techno diplomacies that have displaced the drama of the
heroic, visionary statesman, replacing him with the avatars of a disciplinary politics in the form of crisis management,
command and control, intelligence gathering, and centrally coordinated war-fighting from computer-laden bunkers,
dutifully carried out by attached-case carrying military staff. ll This is where strategic discourse achieves its hegemony, as
the intellectual sphere within which these practices of "organized peacelessness"13 became standard operating procedure.
ENDI 2010 13
Security K Wave 1

SECURITIZATION OF NON-MILITARY LINK

Securitizing things outside of the military defense of the state gives the state free reign to
endlessly expand its military agenda until it has swallowed politics and social relations
Waever 1998 [Ole, professor of International Relations at the Department of Political Science, University of Copenhagen,
―Securitization and Desecuritization,‖ On Security, ed. Ronnie Lipschutz,
http://www.ciaonet.org/book/lipschutz/index.html]

Widening along the referent object axis--that is, saying


that "security is not only military defense of the state, it is also x and y and
z"--has the unfortunate effect of expanding the security realm endlessly, until it encompasses the whole social and political
agenda. This is not, however, just an unhappy coincidence or a temporary lack of clear thinking. The problem is that, as concepts, neither
individual security nor international security exist . National security, that is, the security of the state, is the name of an ongoing
debate, a tradition, an established set of practices and, as such, the concept has a rather formalized referent; conversely, the "security" of
whomever/whatever is a very unclear idea. There is no literature, no philosophy, no tradition of "security" in non-state terms; it is only as a
critical idea, played out against the concept and practices of state security, that other threats and referents have any meaning. An abstract idea of "security"
is a nonanalytical term bearing little relation to the concept of security implied by national or state security. To the extent that we have an idea
of a specific modality labelled "security" it is because we think of national security and its modifications and limitations,
and not because we think of the everyday word "security." The discourse on "alternative security" makes meaningful statements not by
drawing primarily on the register of everyday security but through its contrast with national security. Books and articles such as Jan Øberg's At Sikre
Udvikling og Udvikle Sikkerhed , Richard H. Ullman's "Redefining Security," and Jessica Tuchman Mathews's "Redefining Security" are, consequently,
abundant with "not only," "also" and "more than" arguments. 6 This reveals that they have no generic concept of the meaning of security--only the one
uncritically borrowed from the traditional view, and multiplied and extended to new fields. Thus, it seems reasonable to be conservative along this axis,
accepting that "security" is influenced in important ways by dynamics at the level of individuals and the global system, but not by propagating unclear
terms such as individual security and global security. The concept of security refers to the state.
ENDI 2010 14
Security K Wave 1

SOVEREIGNTY LINK

The invocation of sovereignty is an aesthetic and political practice that should be questioned for
its constructed content
Ashley and Walker 1990 [Richard, Associate Professor of Political Science, Department of Political. Science, Arizona
State University, Tempe, RBJ, professor of political science at the University of Victoria, ―Conclusion: Reading
Dissidence/Writing the Discipline: Crisis and the Question of Sovereignty in International Studies,‖ International Studies
Quarterly, Vol. 34, No. 3, (Sep., 1990), pp. 367-416, jstor]

Developing this third line of reply at some length, we shall show that the
question of sovereignty, viewed as a practical political problem,
is an intrinsically paradoxical problem that can never be named, rationally deliberated, and solved . Whether one speaks of
the sovereignty of a disci- pline or the sovereignty of a modern state, the question is one whose naming and explicit
deliberation would preclude its practical resolution. It is a question whose tentative resolution, if resolution there be, can depend
upon aesthetic practices alone. As we shall suggest, the aesthetic practices of these and similar critical readings, including their construction of a
double bind, labor to produce the effect of a sovereign center of judgment-in this case, the sovereignty of a "discipline" -in response to events that put an
institutional order in crisis and in doubt. As we shall also want to suggest, the aesthetic practices at work in these critical readings are
instructive in far wider scope. They offer helpful examples of a widely practiced strategic art by which the effect of
sovereignty-be it the sovereignty of a territorial state or the sovereignty of a "state of the discipline"-is produced under
conditions of crisis wherein notions of space, time, and political identity are shaken to the core . What occasions this strategic labor
of art? What does it labor to do? How does it do it? What are the conditions of this art's effective performance? Can this strategic art any longer be
effectively performed in a discipline or culture in which territorial bound- aries are everywhere in question and a sense of crisis is acute? What are the
implica- tions for works of thought that would speak in reply to the opportunities and dan- gers of political life today? Developing this third line of reply,
we shall explore these questions.
ENDI 2010 15
Security K Wave 1

HEGEMONY LINK
Hegemony is an organizing myth in our culture- belief in inevitable US dominance as necessary
to protect the globe is based on inaccurate IR assumptions and sanitizes violence to protect our
regime
Preston, 2005 [Scott B.A. Honours, Communications, University of Regina, Regina, Saskatchewan, Canada.
Dark Age Blog 2-22, http://www.darkage.ca/blog/_archives/2005/2/22/363696.html]

The recent history of American interventions around the globe doesn't suggest that Mr. Ferguson's thesis has much
merit. Central America, America's erstwhile neglected "backyard" and the site of much US military and political
meddling, still lies outside the umbrella of American benevolence, languishing in the Hobbesian gloom of that
dark age that Mr. Ferguson's thesis suggests should not exist under the hegemony of the tutelary power . Nor
does the history of US military intervention in Southeast Asia inspire much confidence in the thesis, designed as it was
to bomb North Vietnam "back into the stone age", as one ferocious military planner put it -- an objective almost realised. American
government efforts to roll back or preclude social revolution and the struggle against history in some of the
darkest areas of the world seems to fly directly counter to Ferguson's (mis)representation of affairs .What bothers
me about Ferguson's damn fool either/or treatment of the situation is that all-too-typical tendency of the modern
mentality to aspire to grand abstractions of history in the famous "25 words or less". "We tend to assume that
power, like nature, abhors a vacuum" and therefore "the struggle for mastery is both perennial and universal". That
human beings might be something more than Newtonian forces of nature living on the brink of a Hobbesian
condition of "the war of all against all" just never seems to cross their minds. They call this their "realism" and they
are proud of their little realities. Mr. Ferguson relies on the precedents of history to support his contention that "a world with no hegemon at
all.... could turn out to mean a new Dark Age of waning empires and religious fanaticism; of endemic rapine in the world's no-go zones; of
economic stagnation and a retreat by civilisation into a few fortified enclaves" (presumably something like " Fortress America" and the
gated communities of entrenched mentality in North American suburbia, paranoiac survivalist refuges from the
largely fantasised gathering Hobbesian gloom of the surrounding world and society) . However, the precedents of
history offer no guide to the unprecedented condition in which we find ourselves today, and therefore the past is no certain guide to the
present or the future (thank God). We now live in an interconnected world. This is unprecedented. Our perceptions of
reality are (at least in part) no longer guided by official gatekeepers and authorised guardians of conscience keeping
watch at the portals of the mind, despite the considerable barrage of propaganda we are daily subjected to
designed to counteract this emergent globalism of one world and one humanity (like the whole "clash of civilisations"
creed). In some ways, it truly is a Global Village, even if from inside the walls of Fortress America it might look like the proverbial "jungle out
there" (while to those of us on the outside of Fortress America peering in, it's beginning to look virtually medieval inside those walls).
Human beings are not, after all, forces of nature -- or at least, not entirely so. They speak, and speech is super-natural. Speech
is already effective power and the organisation of power, amongst other things. Into the "vacuum of power" may
global dialogue flow! Human beings may have different interests, but they are also creatures with identical interests too, and those
identical interests are what makes dialogue possible at all. It always strikes me as suspicious how the modern "mentality" simply overlooks
human speech as if it just wasn't there. It seems to offend their "realism". Yet it is speech, and not power relations, that
defines us as human beings. Where speech does not exist, in fact, only violence can restore order amongst
human beings, and a truly Hobbesian state of nature would indeed prevail. Violence is a disease of speech. Mr.
Ferguson's "power vacuum" is actually a "speech vacuum".But the real mendacity of Ferguson's either/or
proposition is the way he overlooks the situation in the US itself . The notion that American imperialism might itself
precipitate the Dark Age, which he presupposes is already lurking beyond the walls of Fortress America, never intrudes to stain
the spotlessness of his cogitations. What he has described as the Hobbesian condition in the absence of a hegemon is really a
condition of speechlessness -- the absence of dialogue. Yet, in the US today, the Bush Administration's emphasis on
unilateralism, pre-emption, rejection of dialogue, contempt for dissenting views, the cooking of intelligence, resort to
propaganda, dismissal of scientific evidence not in conformity with policy, subordination of the universities to
political objectives, the Inquisitions of the Patriot Act, and intimidation of the press all conspire to produce the very conditions of
darkness and speechlessness and atrophy of dialogue that Ferguson claims belong only to the Hobbesian darkness "outside"! Like Robert
Kaplan, who warns of The Coming Anarchy and prescribes US imperialism, "warrior politics" and a return to the good old "pagan ethos" of
the Roman emperor Tiberius, the proposed solution conspires to produce the very barbarism and Dark Age it is
alleged to ameliorate. It's a self-devouring logic and a tautology. What lunacy! It's like the Dance of St. Vitus (and in that
sense Ferguson is right. History can indeed be a guide to the present, at least in terms of the universal madness of groupthink).
The cookie-cutter minds of the modern mentality seem to have no inkling and no self-consciousness at all of
their self-devouring tautological mentations and ruminations. They all possess in common what I call a
"mentality" -- the gated community of the contemporary mind. They have become an obsolete type. Neoliberal,
neoconservative, and neosocialist are virtually indistinguishable. They look alike. They sound alike. Ferguson
and Fukuyama, Messrs. Roberts Cooper, Kaplan, Kagan, and Michael Ignatieff, or Blair and Bush themselves,
CONTINUES…
ENDI 2010 16
Security K Wave 1

HEGEMONY LINK
CONTINUED…
seem to have been cast from a single mould, oblivious to their own petty tyrannies and hypocrisies and
duplicities and the deep nihilism they seem determined to pin and blame on others. I once thought this
duplicity, hypocrisy, and nihilism was the result of a deliberate propaganda of obfuscation. I have since come to
see it as the pathological condition of the late modern "mentality" itself. The modern mentality has become self-
devouring, and these men don't have the slightest consciousness of their condition.

Violent US leadership is wrong methodologically- making hegemony work ―better‖ simply


greases the wheels of future interventions. It‘s essential to challenge the neoconservative logic of
their advantage to demilitarize American politics
James B Rule, PhD Harvard, MA Oxford, BA Brandeis, The Military State of America and the Democratic Left, Dissent Vol. 57 No 1, Winter 2010

Iraq was a defining moment


The invasion of the kind of war that many Americans believed formed no part of this country's
for the United States. This was

repertoire - an aggressive war of choice. Its aim was not to stop some wider conflict or to prevent ethnic cleansing or mass
killings; indeed, its predictable effect was to promote these things. The purpose was to extirpate a regime that the United States had built up but that had morphed into an obstacle to this country - and to replace it with one that would represent a more

the war was a demonstration of American ability and willingness to remove and replace regimes anywhere
compliant instrument of American purpose. In short,

in the world. Even in the wake of the Iraq fiasco, no one in high places has declared repetitions of such exploits "off the
table those on the democratic Left look to the
" - to use the expression favored by this country's foreign policy elites. For those of us who opposed the war, there is obvious relief at the conclusion - we hope - of a conflict that has consistently brought out the worst in this country. But at the same time,

future with unease. Even under a reputedly liberal president, we have reason to worry about new versions of Iraq - in
Afghanistan, Pakistan, Iran or venues yet undisclosed . To its credit, Dissent has not joined the rush to avert attention from the endgames of the Iraq conflict. The Spring 2009 edition features a section of articles under the rubric "Leaving Iraq." The essays focus on the moral and political

the thinking that gave us the American invasion of Iraq in


quandaries of America's departure from a country that it did a great deal to break, but where its ability to repair things is rapidly diminishing. But, a look at the proposals put forward there makes it clear that

the first place has not gone away. Packer inveighs against those seeking a quick exit for American forces. The
George , for example,

balance of power among Iraq's domestic forces could easily be upset , he holds, and valuable progress undone, without a longlingering presence of Americans as enforcers. Obviously playing to the sensitivities of Dissent readers, he concludes that "much

The sense of all this, from Packer's standpoint, becomes clear when you recall his
as we might wish [the war] had never happened at all, America will have obligations as well as interests in Iraq for a long time to come."

efforts to discredit Americans' resistance to the war in the months before it began. The antiwar movement, he wrote in the New York Times

has a serious liability . . . it's controlled by the furthest reaches of the American Left
Magazine in December, 2002, " ." He goes on, in this same article, to envisage a quite different role for those on the Left, like himself,

O'Leary also
who took what he considered a more enlightened view: The "liberal hawks could make the case for war to suspicious Europeans and to wavering fellow Americans," he wrote; "they might even be able to explain the connection between the war in Iraq and the war on terrorism ..." Brendan , another contributor to Dissent's Spring 2009 "Leaving Iraq" section,

stresses responsibility . He, too, means by this continued readiness to apply U.S. coercion to manage Iraqi domestic politics. To judge from his words, he has no difficulty in principle with the notion of remaking Iraq by outside military force: "Reasonable historians should judge ... that removing the genocidal Baathists was overdue," he avers. "The younger
Bush made up for his father's mistake, though he did so for the wrong reasons." Still, O'Leary allows that the invasion hasn't quite unfolded as he might have wished: "... grotesque mismanagement of regime-replacement ... unnecessary and arrogant occupation ... incompetence of American direct rule... numerous errors of policy and imagination ... in the horrors and brutalities that have followed." The America
occupiers have sometimes proved "blindly repressive," he allows - but sometimes, apparently, not repressive enough. Still, leaving before America sets things straight would be irresponsible. If the United States just keeps trying, it may yet get it all right. This country must now manage the political forces set in motion by its invasion according to O'Leary's exacting formula: defend the federalist constitution,
keep resurgent Sunni and Shiite forces from each other's throats, and preserve the autonomy of the Kurds. Just the same, he notes, "After the United States exits, an Arab civil war may re-ignite, as well as Kurdish-Arab conflict." To some of us, an invasion that leaves such possibilities simmering after six years of American-sponsored death and destruction itself seems more than a little irresponsible. Some of the
aims invoked by Packer and O'Leary are beyond reproach. Certainly the United States bears profound responsibilities to protect Iraqis at risk from their collaboration with or employment by American forces - and for that matter, to help repair damage to the country's infrastructure resulting from the invasion. And certainly this country should do everything possible to prevent regional, communal, and ethnic

the deeper, mostly unstated assumptions underlying these


groupings from exploiting a U.S. pullout to oppress others. But making good on any of these estimable goals, as the authors seem to realize, will be a very big order - especially given America's record thus far. Yet

authors' proposals ought to strike a chill throughout the democratic Left. Their problems with the Iraq invasion - and implicitly, future American military

- have to do with execution, not the larger vision of American power that inspired the enterprise Their words
exploits of the same kind .

strike an eerie resonance with those of Friedman, before the invasion occurred he favored war as "a job worth Thomas L. : George W. Bush's "audacious" plan

doing," but only "if we can do it right the continuing mission of the United ." America's violent remaking of Iraq would have been entirely acceptable, it seems, if only Friedman's sensibilities could have guided it. More important:

States as maker and breaker of regimes around the world remained unquestioned When any country gets seriously in the .

way of American power, the global responsibilities of this country are apt to require action like that taken in Iraq. We hear
this thinking most outof-the-closet form from neoconservatives
kind of in its But its roots in American history lie at - who gave us the Iraq invasion in the first place.

least as far back as Manifest Destiny a particularly aggressive form of American exceptionalism. Some higher
notions of . Its key inspiration is

power has created opportunities, indeed obligations, for America to set things straight on a global scale.
- fate, Divine Providence, or special "moral clarity" -

Versions of this idea are pervasive among thinkers who would disclaim identification with the neocons - American foreign policy elites, and those who would guide them - .
Often conveying the doctrine are code words referring to special "responsibilities" of the United States to guarantee world "stability." Or, as Madeleine Albright, then U.S. ambassador to the United Nations, stated, "If we have to use force, it is because we are America. We are the indispensable nation. We stand tall. We see further into the future. . ." To her credit, Albright's effusions in this direction stopped

Accepting this view of America as the ultimate and rightful arbiter of global affairs - as
short of support for invading Iraq - something that cannot be said for the so-called liberal hawks. master hegemon or

world superpower triggers the weightiest implications and consequences Nearly all of them run in collision course
, to use less upbeat terms - . , I hold,

to the best aims and directions of the democratic Left. Yet even for thinkers who identify themselves as being on the Left,
acceptance of a hyper-militarized America, and its concomitant role of global enforcer, often passes without question For .

those of us who challenge this view invasion was wrong for fundamental political and moral reasons. Not because it
, the of Iraq - indeed -

was mismanaged. Not because too few troops were dispatched; not because the Iraqi Army was disbanded; not because the occupation was incompetent, corrupt, and often criminally negligent. It was wrong because wars of this kind are always wrong - aggressive, opportunistic wars of choice, aimed at revamping entire countries to fit the dictates of the invaders. These

wars are wrong because of the destruction and distortions that they spread both abroad and at home they countervail . Among nations,

against one of the subtle but hopeful tendencies in the world today - the movement away from sole reliance on brute state
power to resolve international conflict and toward supranational authorities, multilateral decisi on -making, and
establishment of powers above the level of states At home, the effects are even more insidious. For in order to make itself .

the kind of country capable of "projecting power" anywhere in the world, as America has done so unsuccessfully in Iraq, it
has had to impose vast demands and distortions upon its own domestic life .
ENDI 2010 17
Security K Wave 1

HEGEMONY LINK

Unilateral hegemony is a unique form of state sovereignty that perpetuates the myth of stability in
the international system
Walker 2002 [RBJ, professor of political science at the University of Victoria, ―International/Inequality,‖ International
Studies Review, Vol. 4, No. 2, International Relations and the New Inequality (Summer, 2002), pp. 7-24, jstor]

Sovereigns make the final discrimination. Yet while sovereigns may be supreme in this respect, they are neither alone nor universal. Sovereigns depend on
the system of sovereigns that enable their particular sovereignty. Sovereignty may be the highest authority within a particular territory, but any particular
authority depends on the even higher authority of the principle that the states system itself must survive in order to enable sovereigns to claim the highest
authority. In this sense, the states system affirms a unity, even a univer- sality, first and a plurality, or anarchy, only second. The difficulty with this
instantiation of modern discriminations between unity and diversity is that the pluralities that are enabled are inherently unstable. The modern states
system is always susceptible to war, to the necessity of sovereigns declaring a state of emergency and an exception to all
norms. It is also susceptible to processes through which the states system itself dissolves into something else: into empire,
and the substitution of a vertical hierarchy for a horizontal field of spatially differentiated political communities. Most
accounts of international relations have been preoccupied with the problem of war, and quite properly so. The other problem
has remained largely in the background, largely because it has seemed reasonable to hope that the absence of
empirical equality in a system of formally equal states would be a primary pillar of an interstate order rather than a
fundamental threat to the balance between unity and diversity that sustains that order. In this context, the primary
difficulty is to know how to judge between hegemony in a system of states, an inequality that implies unequal responsibil-
ities, and a hegemony, or a unilateralism, or an empire of some kind that finally turns the constitutive principle of sovereign
equality into little more than a token gesture. All of which is to say that the problem of inequality is already deeply inscribed in
our modern accounts of the international, and thus of modern politics, even before any consideration of the dynamics
associated with modern capitalism as a specific form of economic life that thrives on the production of inequality as a
condition of its own dynamism.
ENDI 2010 18
Security K Wave 1

SOFT POWER LINK

Soft power is the velvet glove of hegemony‘s iron fist- masks implicit racism and violence
Kaplan, 2003 [Amy, Prof. of English @ Univ. of Pennslyvania, American Quarterly 56.1, ―Violent
Belongings and the Question of Empire Today,‖ p. muse]
Another dominant narrative about empire today, told by liberal interventionists, is that of the "reluctant imperialist." 10 In this
version, the United States never sought an empire and may even be constitutionally unsuited to rule one, but it had the
burden thrust upon it by the fall of earlier empires and the failures of modern states, which abuse the human rights of their own people and spawn terrorism.
The United States is the only power in the world with the capacity and the moral authority to act as military
policeman and economic manager to bring order to the world. Benevolence and self-interest merge in this narrative;
backed by unparalleled force, the United States can save the people of the world from their own
anarchy, their descent into an [End Page 4] uncivilized state. As Robert Kaplan writes—not reluctantly at all—in "Supremacy by Stealth: Ten Rules for Managing the
World": "The purpose of power is not power itself; it is a fundamentally liberal purpose of sustaining the key characteristics of an orderly world. Those characteristics include
basic political stability, the idea of liberty, pragmatically conceived; respect for property; economic freedom; and representative government, culturally understood. At this
This
moment in time it is American power, and American power only, that can serve as an organizing principle for the worldwide expansion of liberal civil society." 11
narrative does imagine limits to empire, yet primarily in the selfish refusal of U.S. citizens to sacrifice and shoulder the burden for
others, as though sacrifices have not already been imposed on them by the state. The temporal dimension of this narrative entails the aborted effort of
the future projects the end of empire only when the world is
other nations and peoples to enter modernity, and its view of
remade in our image. This is also a narrative about race. The images of an unruly world, of anarchy and
chaos, of failed modernity, recycle stereotypes of racial inferiority from earlier colonial discourses about races
who are incapable of governing themselves, Kipling's "lesser breeds without the law," or Roosevelt's "loosening ties of civilized society," in his
corollary to the Monroe Doctrine. In his much-noted article in the New York Times Magazine entitled "The American Empire," Michael Ignatieff appended the subtitle "The
Denial and
Burden" but insisted that "America's empire is not like empires of times past, built on colonies, conquest and the white man's burden." 12
exceptionalism are apparently alive and well. In American studies we need to go beyond simply exposing the racism of empire and examine the
dynamics by which Arabs and the religion of Islam are becoming racialized through the interplay of templates of U.S. racial codes
and colonial Orientalism. These narratives of the origins of the current empire—that is, the neoconservative and the liberal interventionist—
have much in common. They take American exceptionalism to new heights: its paradoxical claim to uniqueness and universality at the same time. They
share a teleological narrative of inevitability, that America is the apotheosis of history, the embodiment of universal values of human rights, liberalism,
and democracy, the "indispensable nation," in Madeleine Albright's words. In this logic, the United States claims the authority to "make sovereign
judgments on what is right and what is wrong" for everyone [End Page 5] else and "to exempt itself with an absolutely clear conscience from all the rules
that it proclaims and applies to others." 13 Absolutely protective of its own sovereignty, it upholds a doctrine of
limited sovereignty for others and thus deems the entire world a potential site of intervention .
Universalism thus can be made manifest only through the threat and use of violence. If in these narratives
imperial power is deemed the solution to a broken world, then they preempt any counternarratives that claim U.S. imperial
actions, past and present, may have something to do with the world's problems. According to this logic, resistance to empire can
never be opposition to the imposition of foreign rule; rather, resistance means irrational opposition to
modernity and universal human values. Although these narratives of empire seem ahistorical at best, they are buttressed not only by nostalgia for
the British Empire but also by an effort to rewrite the history of U.S. imperialism by appropriating a progressive historiography that has exposed empire as a dynamic engine of
American history. As part of the "coming-out" narrative, the message is: "Hey what's the big deal. We've always been interventionist and imperialist since the Barbary Coast
and Jefferson's 'empire for liberty.' Let's just be ourselves." A shocking example can be found in the reevaluation of the brutal U.S. war against the Philippines in its struggle for
independence a century ago. This is a chapter of history long ignored or at best seen as a shameful aberration, one that American studies scholars here and in the Philippines
have worked hard to expose, which gained special resonance during the U.S. war in Vietnam. Yet proponents of empire from different political perspectives are now pointing to
the Philippine-American War as a model for the twenty-first century. As Max Boot concludes in Savage Wars of Peace, "The Philippine War stands as a monument to the U.S.
armed forces' ability to fight and win a major counterinsurgency campaign—one that was bigger and uglier than any that America is likely to confront in the future." 14
Historians of the United States have much work to do here, not only in disinterring the buried history of imperialism but also in debating its meaning and its lessons for the
present, and in showing how U.S. interventions have worked from the perspective of comparative imperialisms, in relation to other historical changes and movements across the
globe. The struggle over history also entails a struggle over language and culture. It is not enough to expose the lies when Bush hijacks words [End Page 6] such as freedom,
democracy, and liberty. It's imperative that we draw on our knowledge of the powerful alternative meanings of these key words from both national and transnational sources.
reluctant imperialists are making arguments about "soft power," the global circulation of
Today's
American culture to promote its universal values. As Ignatieff writes, "America fills the hearts and minds of an entire planet with its dreams and
desires." 15 The work of scholars in popular culture is more important than ever to show that the Americanization of global culture is not a one-way
street, but a process of transnational exchange, conflict, and transformation, which creates new cultural forms that express dreams and desires not dictated
by empire. In this fantasy of global desire for all things American, those whose dreams are different are
often labeled terrorists who must hate our way of life and thus hate humanity itself. As one of the authors of the
Patriot Act wrote, "when you adopt a way of terror you've excused yourself from the community of human beings." 16 Although I would not
minimize the violence caused by specific terrorist acts, I do want to point out the violence of these definitions of who belongs to humanity. Often in our juridical system under
the Patriot Act, the accusation of terrorism alone, without due process and proof, is enough to exclude persons from the category of humanity. As scholars of American studies,
such exclusions from personhood and humanity
we should bring to the present crisis our knowledge from juridical, literary, and visual representations about the way
have been made throughout history, from the treatment of Indians and slaves to the internment of
Japanese Americans during World War II.
ENDI 2010 19
Security K Wave 1

GOVERNMENTALITY LINK

The 1ac is an example of governmentality – its the intersection between the desire to control
populations and the desire to maintain a global order. This manifests itself in a never-ending
cycle of responses to ―emergencies‖ that are doomed to fail
Dillon and Reid 2000 [Michael, Professor of Political Science at Lancaster and internationally renowned author, and
Julian, lecturer on international relations and professor of political Science at King‘s College in London; from Alternatives,
Volume 25, Issue 1: Global Governance, Liberal Peace, and Complex Emergency]

As a precursor to global governance, governmentality, according to Foucault's initial account, poses the question of order not in terms of
the origin of the law and the location of sovereignty, as do traditional accounts of power, but in terms instead of the management of
population. The management of population is further refined in terms of specific problematics to which population
management may be reduced. These typically include but are not necessarily exhausted by the following topoi of
governmental power: economy, health, welfare, poverty, security, sexuality, demographics, resources, skills, culture, and so
on. Now, where there is an operation of power there is knowledge, and where there is knowledge there is an operation of
power. Here discursive formations emerge and, as Foucault noted, in every society the production of discourse is at once controlled, selected,
organised and redistributed by a certain number of procedures whose role is to ward off its powers and dangers, to gain mastery over its chance events, to
evade its ponderous, formidable materiality.[ 34] More specifically, where there is a policy problematic there is expertise, and where
there is expertise there, too, a policy problematic will emerge. Such problematics are detailed and elaborated in terms of
discrete forms of knowledge as well as interlocking policy domains. Policy domains reify the problematization of life in
certain ways by turning these epistemically and politically contestable orderings of life into "problems" that require the
continuous attention of policy science and the continuous resolutions of policymakers. Policy "actors" develop and compete
on the basis of the expertise that grows up around such problems or clusters of problems and their client populations . Here,
too, we may also discover what might be called "epistemic entrepreneurs." Albeit the market for discourse is prescribed and policed in ways that
Foucault indicated, bidding to formulate novel problematizations they seek to "sell" these, or otherwise have them officially
adopted. In principle, there is no limit to the ways in which the management of population may be problematized. All aspects of human conduct, any
encounter with life, is problematizable. Any problematization is capable of becominga policy problem. Governmentality thereby
creates a market for policy, for science and for policy science, in which problematizations go looking for policy sponsors
while policy sponsors fiercely compete on behalf of their favored problematizations. Reproblematization of problems is
constrained by the institutional and ideological investments surrounding accepted "problems," and by the sheer difficulty of
challenging the inescapable ontological and epistemological assumptions that go into their very formation . There is nothing
so fiercely contested as an epistemological or ontological assumption. And there is nothing so fiercely ridiculed as the
suggestion that the real problem with problematizations exists precisely at the level of such assumptions . Such "paralysis of
analysis" is precisely what policymakers seek to avoid since they are compelled constantly to respond to circumstances
over which they ordinarily have in fact both more and less control than they proclaim. What they do not have is precisely the
control that they want. Yet serial policy failure--the fate and the fuel of all policy--compels them into a continuous search for the new analysis that
will extract them from the aporias in which they constantly find themselves enmeshed.[ 35] Serial policy failure is no simple shortcoming
that science and policy--and policy science--will ultimately overcome. Serial policy failure is rooted in the ontological and
epistemological assumptions that fashion the ways in which global governance encounters and problematizes life as a
process of emergence through fitness landscapes that constantly adaptive and changing ensembles have continuously to
negotiate. As a particular kind of intervention into life, global governance promotes the very changes and unintended
outcomes that it then serially reproblematizes in terms of policy failure. Thus, global liberal governance is not a linear
problem-solving process committed to the resolution of objective policy problems simply by bringing better information
and knowledge to bear upon them. A nonlinear economy of power/knowledge, it deliberately installs socially specific and
radically inequitable distributions of wealth, opportunity, and mortal danger both locally and globally through the very
detailed ways in which life is variously (policy) problematized by it.
ENDI 2010 20
Security K Wave 1

INTERNATIONAL NORMS/RULEMAKING LINK

Setting international norms is a hegemonic practice seeks to order international space


Tuathail 96‘ – Associate Professor of Geography at VT and Professor of Government and International
Affairs (Gearoid O Tuathail, Critical Geopolitics p.1-2)

Our final thesis placed the study of geopolitical reasoning within the context of the study of hegemony in a
nonstatist, Gramscian-inspired sense. A hegemonic power like the United States is by definition a "rule
writer" for the world community. Those occupying positions of power within the United States "become the
deans of world politics, the administrators, regulators and geographers of international affairs. Their power
is a power to constitute the terms of geopolitical world order, an ordering of international space which
defines the central drama in international politics in particularistic ways" (195). What is important here is the
activity of rule making and rule following rather than state dominance, for hegemony IS more than the
primus inter pares power of a state. One can have a condition of hegemony without a hegemonic state,
although, in the period from 1945 to 1985, the rules governing world order were overwhelmingly shaped
by the institutional power and disciplinary power/knowledge apparatuses headquartered in the United
States.
ENDI 2010 21
Security K Wave 1

TERRORISM DISCOURSE LINK

Using ―terrorist‖ labels discursively shifts the perceived meaning of a particular actor and
shapes other actor’s responses
Mutimer 2000 [David, associate professor of political science at York University and Deputy Director of the Center for
International and Security Studies, The Weapons State, pg 21-22]

It is not difficult to imagine a similar set of descriptors of direct relevance to international relations: I have invited a Nobel
'prize winner to the discussion. I have invited a prime minister to the discussion. I have invited a noted freedom fighter to
the discussion. I have invited a former terrorist to the discussion. Imagining Security 21 These four descriptors could all be
applied to a single individual, and indeed they have been applied to at least one individual. Just as each of the epithets Lakoff and
Johnson apply to their hypothetical dinner guest highlights and downplays or hides various parts of the person in question, so do those of my discussant.
The description, given to another member of the group, forms a key part of the image of her fellow di~cussant. Indeed, having no other image on
the basis of which to frame behavior toward this person, she will base her actions on the image created by that description .
The first epithet downplays the high political office of the individual in question and hides her former terrorist activity. Similarly, the epithet terrorist
downplays or hides the person's prime ministerial role, as well as her status as a Nobel laureate. Not only will the image of
the other discussant be altered in relation to each descriptor, but so will that person's conversational strategies and interests.
Indeed, it is not difficult to imagine that someone who would happily sit at a table with a person described as a Nobel Prize
winner might refuse the invitation to sit with a former terrorist . There is a fairly serious concern with Lakoff and Johnson's
formulatiou of the role of metaphor in our understauding. They speak of "grounding" our conceptual system in terms of simple elements
of our everyday lives that we can experience directly, without social mediation. Thus, for example, spatial metaphors of "up"
and "down," "in" and "out" are based on our experiences of the world-we have an inside and an outside, we stand erect, we
sleep lying down and rise when we awaken.2' Lakoff and Johnson have been criticized for betraying a biological bias, and although they clearly
want to ground metaphors in part on our unmediated physiological experience of the world, they also allow for social rather than biological grounding: "In
other words, these 'natural' kinds of experience are products of human nature. Some may be universal, while others will vary from culture to culture."29
Nevertheless, the very idea of grounding tends to assume a hierarchy of knowledge and the possibility of preconstituted experience that is not socially
mediated. We do not need to accept this possibility of presocial knowledge, however, to make use of their iusights into metaphor.

No risk of a turn: using the terrorist label constructs an image not based on the real but its
redeployments are unstable and can still be used to demonize certain actors and populations- only
refusing securitizing discourse avoids this
Mutimer 2000 [David, associate professor of political science at York University and Deputy Director of the Center for
International and Security Studies, The Weapons State, pg 22-23]

Consider again the earlier example I derived from Lakoff and Johnson: the individual described as a Nobel Prize winner, a
prime minister, a freedom fighter, or a terrorist. We might expect that this example means there is a person who is each of
these things, that her characteristics are prediscursive. Even if we reject the possibility of the prediscursive, however, in other
words, if we accept that nothing exists outside discourse, we can retain all that is important in this argument. Each epithet
relates to a particular discourse or set of discourses and can be seen as an indicator of a discursively constituted identity.
This is most obvious in the relation between terrorist and freedom fighter . These labels are identity markers constituted
in particular discourses rather than in any particular features of the individual in question or her activities. In other words, we
can think of the distinctions among highlighting, downplaying, and hiding in terms of the evocation of particular discursive
representations. To use the epithet terrorist is to evoke one discourse with a certain set of entailments that go along
with it, whereas using the epithet freedom fighter evokes a different discourse and a different set of entailments .
Generally, the use of freedom fighter downplays the role of the individual in perpetrating acts of violence, a role highlighted
by the entailments of terrorist. This is not always the case, however. The use of freedom fighter by the Reagan
administration in the 1980s meant that in certain circles the term has come to be a pejorative and not only entails the role of
the individual so named in perpetrating acts of violence but marks those acts as violence in the cause of a reactionary
politics. This difference in the entailments of the same label in different circumstances is important, because it demonstrates that not only does
metaphor link discourses but that the production of those links depends on the discursive context in which the metaphor is
evoked. Metaphors are not grounded in a real or literal experience; further, even the "discursive connections they create are
never entirely stable.
ENDI 2010 22
Security K Wave 1

TERRORISM DISCOURSE LINK

Rethinking the epistemological and ideological foundations of how we understand terrorism is


necessary to understand the roots of violence. The affirmative‘s call for a hard approach merely
obscures the conditions that cause retaliatory violence to begin with
Der Derian – Rhodes Scholar at Oxford University, where he completed a M.Phil. and D.Phil. in
international relations (James Der Derian, Critical practices in international theory: selected essays p. 69-70
RC)
It is more difficult - and certainly less popular - to assess the intellectual and structural obstacles blocking an inquiry into
terrorism. The first obstacle is epistemological: even the most conscientious and independent student of
terrorism faces a narrowly bounded discipline of thought. During the 1980s, terrorist studies became a
fortress-haven at the edge of the social sciences, a positivist's armory of definitions, typologies, and
databases to be wielded as much against the methodological critic as the actual terrorist who might call into
question the sovereign reason and borders of the nation-state. The second obstacle is ideological: to gain
official entry into the terrorist debate one must check critical weapons at the door, and join in the chorus
of condemnation- or risk suspicion of having sympathy for the terrorist devil. What this means is that following a
rash of terrorist incidents - at the moments of highest tension when sober thinking is most needed - responses other than instant
excoriation and threats of retaliation are seen as "soft," or worse, collaborationist. As others have noted, this is very
reminiscent of the regimentation of critical thinking by threat-mongering that marked Cold War I in the 1940s and 1950s and the most morbid moments of
Cold War II in the early 1980s. Let Oliver North remind us once again: "It is very important for the American people to understand that this is a dangerous
world that we live at risk and this nation is at risk in a dangerous world‘s However, as Gorbachev worked hard to improve relations with the United States, and
as the Soviet bloc began to disintegrate, it proved increasingly difficult to find, let alone maintain the credibility of an alien, uniform, foe. In the future, there will
the third, ontological, reason
indeed be external dangers, but it is US national identity, not the United States as a nation, that it is truly at risk. Here lies
for the intractability of terrorism: it has been subsumed by the traditional gambit of defining and unifying a
national identity through the alienation of others. In spite of the odds that we are more likely to die from a
lightning strike, an automobile accident, or even a bee sting, many have come to accept the ubiquity o f the
terrorist threat as well as tale on the identity of the victim. The terrorist discourse Yet, even in polls taken
immediately after a terrorist strike, the majority of Americans are reluctant to endorse military retaliation.6
Common sense probably plays a conservative role: if polled, most Americans would probably not (for similar reasons) endorse surgical air-strikes on automobile
plants or bee colonies to lessen the chances of an unlikely death. But I suspect something beyond common sense is at work. Reflecting the diverse and highly
individualistic forces behind terrorism, weare not - nor can we be - of one mind, of one identity, or of one course of
action when it comes time to think and act collectively against the terrorist threat. What the polls probably reflected is
that after Vietnam (and before another Lebanon debacle), many preferred the non-identity of a silent but safe majority when it
comes to taking on an enemy that is fearsome but faceless, anywhere and nowhere. This is not to claim that one
must sympathize with terrorism in order to understand it, although this chapter does attempt a better understanding of the terrorist in
situ. Nor is it to pretend that a total comprehension of terrorism is possible, remedial, or even preferable,
although this chapter does try to reconstruct our knowledge and to critique current practices of terrorism
and anti-terrorism. Rather, it is to argue at the outset that any productive reading of terrorism requires
difficult, even contorted feat, of stepping outside of the one – dimensional identities that terrorism and the
national security culture have implanted in both sides the conflict.
ENDI 2010 23
Security K Wave 1

TERRORISM DISCOURSE LINK

The aff reacts to the threat of terrorism through exaggerated anxiety created by political elites.
This encourages retaliatory violence and animosity—terror talk leads to war
Huddy et al 05 (Leonie, Stanley Feldman, Charles Taber and Gallya Lahav, Leonie, a Professor. Department of Political
Science, Stanley, Professor and Associate Director, Center for Survey Research, Charles, Department of Political Science at
Stony Brook University, Gallya, Associate Professor,Department of Political science at stony university, ―Threat, Anxiety,
and Support of Antiterrorism Policies‖, American Journal of Political Science, Vol. 49, No. 3 (Jul., 2005), pp. 593-608)

The perception of threat and the experience of anxiety are distinct but related public reactions to terrorism. Anxiety increases risk
aversion, potentially undercutting support for dangerous military action, consistent with terrorist‘ typical aims. Conversely, perceived
threat increases a desire for retaliation and promoters animosity toward a threatening enemy inline with the usual
goals of effected governments. Findings from a national telephone survey may confirm the differing political effects of anxiety and perceived
threat. The minority of Americans who experienced high levels of anxiety in response to the September 11 attacks were less
supportive of aggressive military action against terrorism, less approving of President Bush, and favored increased American
isolationism. In contrast, the majority of America ns who perceived a high threat of future terrorism in the United States (but
were not overly anxious) supported the Bush administration‘s antiterrorism policies domestically and internationally.
Psychological reactions to terrorism play a pivotal role in understanding public support for government
antiterrorist policies. As Crenshaw argues: ―The political effectiveness of terrorism is importantly determined by a psychological effects of
violence on audiences‖ (1986-400). In an area of research characterized by disagreement over the definition and objectives of terrorism, there is persuasive
agreement that the effects of terrorism extend well beyond its immediate victims and physical destruction to include a much broader target population
(Crenshaw 1986; Wardlaw 1982). There are differing psychological reactions to external threat , however, and these reactions shape
support of government policies designed to combat terrorism. Based on a review of the literature below, we draw a critical distinction
between perceived threat and the anxiety it can elict. The political importance of this distinction between perceived threat and the anxiety rests on their
typical psychological effects: anxiety leads to an overtimaniation of risk and risk-averse behavior (Lerner and Keltner 2000, 2001;
Raghunathan and Pham 19999) whereas external and perceived threat increase support for outwardly focused retaliatory action
(Hermann, Terlock, and Visser 1999; Jentleson 1992; Jentleson and Britton 1998). The distinction between perceived threat and anxiety is intimately tied
to the major objectives of terrorists and governments in countries targeted by terrorism. A major function of terrorist violence is to instill
anxiety in a target population; this anxiety then places pressure on political élites to negotiate and make concessions with
terrorists on order to mollify their frightened citizens (Friedland and Merari 1985; Long 1990). Long argues that terrorists often ―use the
unreasonable fear and the resulting political disaffection it has generated among the public to intimidate governments into
making political concessions in line with its political goals‖ (1990, 5). In this sense, terrorists may have a good grasp of psychological
reality. The intended effects of terrorism are consistent with the psychological link between anxiety and risk aversion. These motives
contrast starkly, however, with the need of governments in vulnerable countries to take forceful action against terrorists. As Berry puts it: ―A target that
is incapable of responding to terrorism will lose public support and lessen its capabilities and confidence to thwart terrorism
in the future‖ (1987, 296). Moreover, tough antiterrorist policies require firm public revolve because they can be long lasting, expensive, and intrusive
(Long 1990; White 2002; Wilkinson 1987). A serious threat to national security typically promotes support for military action, in line
with the objectives of targeted governments (Jentleson 1992; Jentleson and Britton 1998). But this response may be undercut by heightened
anxiety and an associated increase in the risk aversion among affected individuals.
ENDI 2010 24
Security K Wave 1

AT: TERRORISTS ARE IRRATIONAL

The psychology of terrorism proves it is the result of rational frustrations and victimization, not
the pathologized evil the affirmative attributes to acts of terror
Hamden 06 (Raymond H., Ph.D., Clinical & Forensic Psychologist, ―Psychology of Terrorists: 4 types‖, 6/17, p. 1-2,
http://www.all-about-psychology.com/support-files/psychology-of-terrorists.pdf)

Commonly, terrorists are seen as psychopathic or having a religious or political cause. Yet, if we look at the individual
players, we may see personal motives verses motives of principle. A common assumption is that terrorists use force or
threat of force instrumentally in a conscious and premeditated fashion because they misguidedly think that it will enhance
their probability of achieving a certain political or religious goal or set of goals. Psychologists may tend to see political or religious
goals as an arena in which emotions originating elsewhere are stimulated and played out. So, the psychologist may ask - What nonpolitical
frustrations or drives are at the base of the behavior ? Berkowitz (1969) points out two basic variations on this theme. First is the
situation in which an individual is suffering from the effects of very unpleasant present or past conditions (e.g. painful events or
frustrations). This will give rise to a ―fairly specific internal inclination to be aggressive,‖ which can be triggered by some political
situation or event. Berkowitz‘s second variation (1975) is that a person may merely be excited or aroused. This general, initially
nonaggressive arousal can - under appropriate conditions - be channeled into political violence. The classic example is that of a large group
of people that suddenly turns into an angry, violent mob . Jeanne Knutson‘s research (1981) resulted in her belief that victimization is the
motive force behind much political violence in the contemporary world. Victimization is a personally experienced injustice
which the victim knows to be unnecessary and which creates a basic fear of annihilation. Discrete victimization events that have the
strength to change the victim‘s perception of the world can cause the victim to act in defense of him or herself and his or her group in
order to reduce the chances for further aggression against the self, family, community, or all three. Political psychologists are
inclined to look at instances of the use of force or the threat of force in terms of both of these perspectives instrumental and expressive. Each act is usually
based on some mixture of instrumental motivation and underlying psychological dynamics. Therefore, try to ascertain the particular mixture of the
underlining acts of political violence. The use of force or the threat to use force usually implies the use of some form of violence.
The question of the origins and triggers of human violence has intrigued students of human behavior at least since the earliest
days of written history. During the twentieth century, scholars have advanced a wide variety of theories of human aggressiveness. These can be
roughly divided into three categories: (1) biological theories (psychophysiological, sociobiological, and ethological); (2) there are the psychological and
social-psychological theories (from Freudian theories to theories of situational conformity), and (3) the discipline of political psychology (generating many
theories to better understand terrorism and international violence). Knutson and Etheredge have emerged in direct response to events in
the political world of psychological understanding. Dollard (1939) and others have formed the basis for more politically oriented work. Ted
Robert Gurr (1970) has developed a theory of revolutionary behavior based on frustration-aggression theory. Freud, as well as
Dollard and his associates, and Etheredge (1979), focuses on what happens inside the individual. Situational conformity theory
concentrates on what is happening in the microenvironment. Social learning theory, as well as Knutson‘s victimization theory, concentrates on
the impact of both the microenvironment and the macro environment on individual behavior . Ethnocentrism focuses on the
dynamics of the micro environmental interactions within groups, as well as the macro environmental issues involved in the relationships between groups,
which may consist of entire cultural or national collectivities.
ENDI 2010 25
Security K Wave 1

BORDERS LINK

Recognizing and naming boundaries is an essential component of securitization strategies


Lipschutz 1998 [Ronnie, prof of politics at UC Santa Cruz, ―Negotiating the Boundaries of Difference and Security at
Millennium's End,‖ On Security, ed. Ronnie Lipschutz, http://www.ciaonet.org/book/lipschutz/index.html]

As a speech act, security is about specifying, through discourse, the permitted conditions under which acts that "secure" the
state can take place. In a world of relatively autonomous states, with low levels of interaction, it is possible to draw the conceptual boundaries that
establish difference between two states and that also define a range of permitted behavior and beliefs. Specifying the goals of other states'
behaviors, as friendly or hostile, could also be a part of this boundary-drawing. Whether we accept such boundary definition as
justifiable or not is beside the point; the state is clearly the referent of security as speech act and as behavior. The most secure state is, under
these conditions, the one most successful in excluding outside influences by drawing boundaries that can be secured; in
Barry Buzan's terms, a "closed" state. But, as Buzan's analysis suggests, a closed state is either very sure of itself and its purpose in the world, or very
insecure about its viability. 7 It is either very confident of its ability to ward off the efforts of others to penetrate it, and very sure that it has the undying
loyalty of its citizens, so that no social and economic intercourse is desirable or necessary. Or, it is so weak and insecure that, as in the case of North
Korea, closure is the only way to ensure that the state and its citizens will not be subverted and "turned" by external influences.

The aff‘s invocation of borders reifies geographic boundary drawing as a violent practice of
mastery that leads to war and displacement while marginalizing those without the power over
mapping
Tuathail 96‘ – Associate Professor of Geography at VT and Professor of Government and International
Affairs (Gearoid O Tuathail, Critical Geopolitics p.1-2)

Geography is about power. Although often assumed to be innocent the geography of the world is not a product of nature but a
product of histories of struggle between competing authorities over the power to organize, occupy, and administer space.
Imperial systems throughout history, from classical Greece and Rome to China and the Arab world, exercised their power through their
ability to impose order and meaning upon space. In 16th century Europe, the centralizing states of the ―new monarchs‖ began
organizing space around an intensified principle of royal absolutism. In regions both within and beyond the nominal domain of the Crown, the
power of royal authority over space was extended and deepened by newly powerful court bureaucracies and armies. The results
in many instances were often violent, as the jurisdictional ambitions of total authority met the determined resistance of certain
local and regional lords. Within the context of this struggle, the cartographic and other descriptive forms of knowledge that took the name
―geography‖ in the early modern period and that were written in the name of the sovereign could hardly be anything else but political. To the
opponents of the expansionist court, ―geography was a foreign imposition, a form of knowledge conceived in imperial capitals and
dedicated to the territorialization of space along the lines established by royal authority. Geography was not something
already posses by the earth but an active writing of the earth by an expanding, centralizing imperial state. It was not a
noun but a verb, a geo-graphing, an earth-writing by ambitious endocolonizing and exocolonizing states who sought to seize
space and organize it to fit their own cultural visions and material interests. More than five hundred years later, this struggle
between centralizing states and authoritative centers, on the on hand, and rebellious margins and dissident cultures, on the other hand , is
still with us. While almost all of the land of the earth has now been territorialized by states, the process by which this disciplining of space
by modern states occurs remain highly contested . From Chechnya to Chiapas and from Rondonia to Kurdistann and East Timor, the
jurisdictions of centralized nation-states strive to eliminate the contradictions of marginalized peoples and nations . Idealized
maps from the center clash with the lived geographies of the margin, with the controlling cartographic visions of the former frequently
inducing cultural conflict, war, and displacement. Indeed, the rise in absolute number of displaced peoples in the past twenty-five years is
testimony to the persistence of struggles over space and place. In 1993 The United Nations High Commisioner for Refugees estimated that roughly 1 in
every 130 people on earth has been forced into flight because of war and state persecution. In 1970 there were 2.5 million refugees in the world; today that
figure is well over 18.2 million. In addition an estimated 24 million people are internally displaced within their own states because
of conflict. More recently, genocide in Rwanda left over 500,000 murdered and produced an unprecedented exodus of refuges from that state into
surrounding states. Refugees continue to be generated by ―ethnic cleansing‖ campaigns in the Balkans; economic collapse in Cuba; ethnic wars in the
Caucus; state repression in Guatemala, Turkey, Indonesia, Iraq, and Sudan; and xenophobic terror in many other states . Struggles over the
ownership, administration, and mastery of space are an inescapable part of the dynamic of contemporary global politics .
ENDI 2010 26
Security K Wave 1

BORDERS LINK - REALISM

Realist theories of IR rely on a historically specific spatial ontology that delineate political
communities according to inside/outside dichotomies, depoliticizing criticism
Edkins 1999 [Jenny, Senior Lecturer in International Politics at the University of Wales Aberystwyth, Postructuralism and
International Relations: Bringing the Political Back In, p. 139 ]

Theories of international relations can be complicit in this process of concealment. As R. B. J. Walker has pointed out, they are
"expressions of the limits of modern politics [that] reveal some of the crucial conditions under which modern political life is possible at all, as well as the
conditions under which alternatives to the present have been rendered implausible or even unthinkable. "79 Theories of international relations
contribute to the depoliticization of the international and the domestic every time they take for granted the separation of the
two, with the domestic realm within the sovereign state being seen as the realm of "political community" and the
international arena as the domain of anarchy, where political or ethical community is replaced by power politics in some
raw state of nature. This separation is for some (but by no means all) scholars the very self-definition of the "discipline," and it claims a past as well
as a present; it traces its origins and analyzes its Westphalian genesis. The concept of sovereignty plays a crucial part in delimiting domestic "politics." As
with the master signifier "politics," however, that of "sovereignty" erases the traces of its own historicity. In Walker's words again, international
relations theory is a discourse that systematically reifies an historically specific spatial ontology, a sharp delineation of here
and there, a discourse that both expresses and constantly affirms the presence and absence of political life inside and
outside the modern state as the only ground on which structural necessities can be understood and new realms of freedom
and history can be revealed.80
ENDI 2010 27
Security K Wave 1

AT: BORDERS KEY TO SECURITY

Borders don‘t enhance security and realism does not predict security problems—they are
unprepared for contemporary security dilemmas and construct them in ways that create self-
fulfilling prophecies
Der Derian 05 – Rhodes Scholar at Oxford University, where he completed a M.Phil. and D.Phil. in
international relations
(James Der Derian, Predicting the President, http://hir.harvard.edu/articles/1430/ RC)

It often takes a catastrophe to reveal the illusory beliefs we continue to harbor in national and homeland security. To
keep us safe, we place our
faith in national borders and guards, bureaucracies and experts, technologies and armies. These and other
instruments of national security are empowered and legitimated by the assumption that it falls upon the
sovereign country to protect us from the turbulent state of nature and anarchy that permanently lies in
wait offshore and over the horizon for the unprepared and inadequately defended. But this parochial fear, posing as a
realistic worldview, has recently taken some very hard knocks. Prior to September 11, 2001, national borders were thought to be
necessary and sufficient to keep our enemies at bay; upon entry to Baghdad, a virtuous triumphalism and a revolution in military affairs
were touted as the best means to bring peace and democracy to the Middle East; and before Hurricane Katrina, emergency preparedness and an intricate system
of levees were supposed to keep New Orleans safe and dry. The intractability of disaster, especially its unexpected, unplanned, unprecedented
nature, erodes not only the very distinction of the local, national, and global, but, assisted and amplified by an
unblinking global media, reveals the contingent and highly interconnected character of life in general. Yet
when it comes to dealing with natural and unnatural disasters, we continue to expect (and, in the absence of a credible alternative, understandably so)
if not certainty and total safety at least a high level of probability and competence from our national and homeland
security experts. However, between the mixed metaphors and behind the metaphysical concepts given voice by US Homeland Security Director Michael
Chertoff early into the Katrina crisis, there lurks an uneasy recognition that this administration—and perhaps no national government—is up to the task
of managing incidents that so rapidly cascade into global events. Indeed, they suggest that our national plans and preparations
for the ―big one‖—a force-five hurricane, terrorist attack, pandemic disease—have become part of the problem, not the solution. His use of hyberbolic terms
like ―ultra-catastrophe‖ and ―fall-out‖ is telling:
such events exceed not only local and national capabilities, but the
capacity of conventional language itself. An easy deflection would be to lay the blame on the neoconservative faithful of the first term
of US President George W. Bush, who, viewing through an inverted Wilsonian prism the world as they would wish it to be, have now been forced by natural
and unnatural disasters to face the world as it really is—and not even the most sophisticated public affairs machine of dissimulations, distortions, and lies can
close this gap. However, the discourse of the second Bush term has increasingly returned to the dominant worldview of national security, realism. And if
language is, as Nietzsche claimed, a prisonhouse, realism is its supermax penitentiary. Based on linear notions
of causality, a correspondence theory of truth, and the materiality of power, how can realism possibly account—let alone
prepare or provide remedies—for complex catastrophes, like the toppling of the World Trade Center and attack on the
Pentagon by a handful of jihadists armed with box-cutters and a few months of flight-training? A force-five hurricane that might well have begun with the
flapping of a butterfly‘s wings? A northeast electrical blackout that started with a falling tree limb in Ohio? A possible pandemic triggered by the mutation of an
avian virus? How, for instance, are we to measure the immaterial power of the CNN-effect on the first Gulf War,
the Al-Jazeera-effect on the Iraq War, or the Nokia-effect on the London terrorist bombings? For events of such complex,
non-linear origins and with such tightly-coupled, quantum effects, the national security discourse of realism is simply not up to
the task. Worse, what if the ―failure of imagination‖ identified by the 9/11 Commission is built into our national and homeland security systems? What if
the reliance on planning for the catastrophe that never came reduced our capability to flexibly respond and improvise for the ―ultra-catastrophe‖ that did? What

CONTINUES…
ENDI 2010 28
Security K Wave 1

AT: BORDERS K SECURITY


CONTINUED…
if worse-case scenarios, simulation training, and disaster exercises—as well as border guards, concrete barriers and earthen
levees—not only prove inadequate but might well act as force-multipliers—what organizational theorists
identify as ―negative synergy‖ and ―cascading effects‖ —that produce the automated bungling (think Federal Emergency
Management Agency) that transform isolated events and singular attacks into global disasters? Just as ―normal accidents‖ are built into new technologies—from
the Titanic sinking to the Chernobyl meltdown to the Challenger explosion—we
must ask whether ―ultra-catastrophes‖ are no
longer the exception but now part and parcel of densely networked systems that defy national management; in other
words, ―planned disasters.‖ What, then, is to be done? A first step is to move beyond the wheel-spinning debates that
perennially keep security discourse always one step behind the (global event. It might well be uni-, bi-, or
multi-polar, but it is time to recognize that the power configuration of the states-system is rapidly being
subsumed by a heteropolar matrix, in which a wide range of different actors and technological drivers are
producing profound global effects through interconnectivity. Varying in identity, interests, and strength, these new actors and
drivers gain advantage through the broad bandwidth of information technology, for networked communication systems provide the means to traverse political,
economic, religious, and cultural boundaries, changing not only how we interpret events, but making it ever more difficult to maintain the very distinction of
intended from accidental events. According to the legal philosopher of Nazi Germany, Carl Schmitt, when the state is unable to deliver on its traditional
promissory notes of safety, security, and well-being through legal, democratic means, it will necessarily exercise the sovereign ―exception:‖ declaring a state of
emergency, defining friend from foe, and, if necessary, eradicating the threat to the state. But what if the state, facing the global event, cannot discern the
accidental from the intentional? An external attack from an internal auto-immune response? The natural as opposed to the ―planned disaster‖? The enemy
within from the enemy without? We can, as the United States has done since September 11, continue to treat catastrophic threats as issues of national rather
than global security, and go it alone. However, once declared, bureaucratically installed, and repetitively gamed, national states of emergency grow recalcitrant
and become prone to even worse disasters. As Paul Virilio, master theorist of the war machine and the integral accident once told me: ―The full-scale accident is
now the prolongation of total war by other means.‖
ENDI 2010 29
Security K Wave 1

SOUTH CHINA SEA LINK

Threat of war in the South China Sea is a falsely constructed maneuver of securitization
CALLAHAN 2004 [PROF IR AND DIRECTOR CENTRE FOR CHINESE STUDIES U DURHAM, CONTINGENT
STATES: GREATER CHINA AND TRANSNATIONAL RELATIONS, PAGE 71-74]
China further formalized its claim in February 1992, when the National People's Congress passed the "Law of the People's Republic of China on its
Territorial Waters and Contiguous Areas." This action disturbed the region because it unilaterally made a legal claim for ownership not just of the islands
in the South China Sea but also for Taiwan, the Diaoyu Islands, Penghu Islands, Dongsha Islands, and "other islands that belong to the PRC" ("Law on
Territorial Waters" BBC/SWB 28 November i99z, Wh-2). But even this aggressive unilateral action is still phrased in the familiar language of state
sovereignty: the law is "to enable the PRC to exercise its sovereignty over its territorial waters and its rights to exercise control over their adjacent areas,
and to safeguard state security as well as its maritime rights and interests" ("Law on Territorial Waters" BBC/SWB 28 November 1992, W/T-z). The right
to defend this sovereignty through military action is included in this law. Once again, both sympathetic and critical readings of China's
diplomacy in the South China Sea reaffirm that China is being converted to the Westphalian notion of the sovereignty of
nation-states. To put it another way, Chinese actions in the South China Sea are quite predictable. China is involved in the age-old
process of "writing security" (Campbell 7yg8a). Through its military and diplomatic narratives, China-like the other states in the
dispute-is creating a problem in the South China Sea to craft and manage borders that otherwise do not make sense. Although the South
China Sea is commonly seen as one of the main "security problems" in East Asia, in fact there is little actual conflict there. As
in the Kasmiri conflict between India and Pakistan, where the greatest casualties are to altitude sickness and frostbite
(Krishna 1996, zoo-ZOi), in the South China Sea soldiers do not fight each other so much as storms and sunstroke. As the
newspaper articles tell us, the main enemy in the South China Sea is the sea itself: "In October 1993, Typhoon No. Zo hit the main pillbox. High waves
rolled over the rooftop of the three-story-high building. Erected structures and equipment lying on an area of 60o square meters of the construction site
were swept away" (Hu Zhanfan rgg4, m). When the sea does not get you, the sun will: "[W]e heard of instances of asphalt felt melting and thermometers
bursting under the scorching sun of the Nansha islands" (Hu Zhanfan 1994, 11; Whiting 1998, z99). The "Nansha Spirit" describes enduring the hardship
of the weather conditions, rather than surviving the horrors of battle. Hence equipment upgrading concentrates on stronger air conditioners and better fresh
water supplies, rather than on bigger guns (Ling Xingzheng 1998; Zheng Degang 1999; Austin i998, 3r2). Indeed, although the South China Sea
disputes are a hot topic in English-language security studies journals, the Chinese press, and popular histories, they are not
a common item in Chinese security studies journals (Stenseth 1999, 36). The White Paper on China's National Defense Zoo2
declares, "The situation in the South China Sea area has been basically stable, as the relevant countries have signed the
Declaration on the Conduct of Parties in the South China Sea ."In other words, there is no "there" there: in addition to a lack of
military conflict, there is no substantial territory to defend , fisheries are depleted, and there is little sign of the promised petrochemical
riches. National maritime territory has to be created to manufacture threats to national security that are tied to writing the security
of the newly discovered ancient "sacred territory." It is the conceptualization of "security" itself, which creates the subjectivity of
the state, that has made "a relatively peaceful area into one of serious security concerns" (Zha, Daojiong zooi, 34). As Walker puts it, "the subject of
security is the subject of security" (Walker Ty97, 78; Campbell r9y8a, i9g). The South China Sea disputes thus show how the
primary purpose of state security is not to secure a particular nation-state, but to secure the limitation of politics to the
spatio-temporal demarcations of state sovereignty that limit identity to citizenship. The very active project of transforming
China from a continental power to a maritime power serves as a cogent example of security not defending us so much as
"tell[ing] us who we must be" (Walker 1997, 7i-72; Campbell z998a, i99). To rethink security-and to rethink the "problem" and "solution" of the
Spratly Islands disputes-we have to "rethink the character and location of the political" by asking who or what is to be secured, and under what conditions?
(Walker i997,ti9).
ENDI 2010 30
Security K Wave 1

MIDDLE EAST LINK


Desire for stability in the middle east represent violent unconscious desires for global control
Engelhardt 9 Tom Engelhardt, co-founder of the American Empire Project and contributor to Foreign Policy In Focus, runs the Nation Institute's
TomDispatch.com, 3/1/09 Foreign Policy In Focus, ―The Imperial Unconscious‖ Google

Here, according to Bloomberg News, is part of Secretary of Defense Robert Gates‘s


recent testimony on the Afghan War
U.S. goals in Afghanistan must be 'modest, realistic,' and
before the Senate Foreign Relations Committee:
'above all, there must be an Afghan face on this war,' Gates said. 'The Afghan people must believe this is their
war and we are there to help them. If they think we are there for our own purposes, then we will go the way of every other
foreign army that has been in Afghanistan. Now, in our world, a statement like this seems so obvious, so reasonable as to be
beyond comment. And yet, stop a moment and think about this part of it: ―There must be an Afghan face on this war.‖ U.S.
military and civilian officials used an equivalent phrase in 2005-2006 when things were
going really, really wrong in Iraq. It was then commonplace — and no less unremarked upon — for
them to urgently suggest that an ―Iraqi face‖ be put on events there. Evidently back in vogue for a different
war, the phrase is revelatory — and oddly blunt. As an image, there‘s really only one way to understand it (not
that anyone here stops to do so). After all, what does it mean to ―put a face‖ on something that assumedly
already has a face? In this case, it has to mean putting an Afghan mask over what we know to be
the actual ―face‖ of the Afghan War — ours — a foreign face that men like Gates recognize,
quite correctly, is not the one most Afghans want to see. It‘s hardly surprising that the
Secretary of Defense would pick up such a phrase, part of Washington‘s everyday arsenal of
words and images when it comes to geopolitics, power, and war. And yet, make no mistake, this is
Empire-speak, American-style. It‘s the language — behind which lies a deeper structure of
argument and thought — that is essential to Washington‘s vision of itself as a planet-
straddling goliath. Think of that ―Afghan face‖ mask, in fact, as part of the flotsam and jetsam
that regularly bubbles up from the American imperial unconscious. Of course, words create
realities even though such language, in all its strangeness, essentially passes unnoticed here.
Largely uncommented upon, it helps normalize American practices in the world, comfortably
shielding us from certain global realities; but it also has the potential to blind us to those
realities, which, in perilous times, can be dangerous indeed. So let‘s consider just a few entries in what
might be thought of as The Dictionary of American Empire-Speak.
ENDI 2010 31
Security K Wave 1

MIDDLE EAST LINK

Representations can’t be divorced from policy actions- they establish a


framework for thinking about the Middle East. They selectively reveal and
conceal aspects of the Middle East to represent it as conflict prone
Bilgin, 2005 [Pinar, PhD International Politics, University of Wales, Aberystwyth, Department of International Relations
Bilkent Univ., Regional Security in the Middle East p. 1

Throughout the twentieth century, the Middle East remained as an arena of incessant conflict attracting global attention. As the
recent developments in Israel/Palestine and the US-led war on Iraq have showed, it is difficult to exaggerate the signifcance of Middle Eastern insecurities for
world politics. By adopting a critical approach to re-think security in the Middle East, this study
addresses an issue that continues to attract the attention of students of world politics. Focusing on the constitutive
relationship between (inventing) regions, and (conceptions and practices of) security, the study argues that the current state of 'regional security' -
often a euphemism for regional insecurities - has its roots in practices that have throughout history been shaped by its
various representations - the geopolitical inventions of security. In doing this, it lays out the
contours of a framework for thinking differently about regional security in the Middle East.
Prevailing approaches to regional security have had their origins in the security concerns and interests
of Western states, mainly the United States. The implication of this Western bias in security thinking within
the Middle Eastern context has been that much of the thinking done on regional security in the Middle East has been
based on Western conceptions of 'security'. During the Cold War what was meant by 'security in the Middle
East' was maintaining the security of Western (mostly US) interests in this part of the world and its military defence against other
external actors (such as the Soviet Union that could jeopardise the regional and/or global status quo). Western security interests in the Middle East
during the Cold War era could be summed up as the unhindered flow of oil at reasonable prices, the cessation
of the Arab-Israeli conflict, the prevention of the emergence of any regional hegemon, and the maintenance of
'friendly' regimes that were sensitive to these concerns. This was (and still is) a top-down conception of security that was
military-focused, directed outwards and privileged the maintenance of stability. Let us take a brief look at these
characteristics. The Cold War approach to regional security in the Middle East was top-down because threats to security were defined largely from the perspective of
external powers rather than regional states or peoples. In the eyes of British and US defence planners, communist infiltration and Soviet intervention constituted
the greatest threats to security in the Middle East during the Cold War. The way to enhance regional security, they argued, was for regional states to enter into

alliances with the West. Two security umbrella schemes, the Middle East Defence Organisation (1951) and the Baghdad Pact (1955), were designed for this purpose. Although there were regional
states such as Iraq (until the 1958 coup), Iran (until the 1978-79 revolution), Saudi Arabia, Israel and Turkey that shared this perception of security to a certain extent, many Arab policy-makers

begged to differ. Traces of this top-down thinking are still prevalent in the US approach to security in the
'Middle East'. During the 1990s, in following a policy of dual containment US policy-makers presented Iran and Iraq as the main threats to
regional security largely due to their military capabilities and the revisionist character of their regimes that were not

subservient to US interests. In the aftermath of the events of September 11 US policy-makers have focused on 'terrorism' as a major threat to security in the Middle East and elsewhere. Yet,
US policy so far has been one of 'confronting the symptoms rather than the cause' (Zunes 2002:237) as it
has focused on the military dimension of security (to the neglect of the socio-economic one) and relied
on military tools (as with the war on Iraq) in addressing these threats. This is not to underestimate the threat posed by weapons of mass destruction or terrorism to global and regional security. Rather, the
point is that these top-down perspectives, while revealing certain aspects of regional insecurity at

the same time hinder others. For example, societal and environmental problems caused by resource scarcity do not only threaten the
security of individual human beings but also exacerbate existing conflicts (as with the struggle over water resources in Israel/Palestine; see Sosland 2002). Besides, the lives of

women in Kuwait and Saudi Arabia were made insecure not only by the threat caused by Iraq's
military capabilities, but also because of the conservative character of their own regimes that restrict women's rights
under the cloak of religious tradition. For, it is women who suffer disproportionately as a result of militarism and the

channelling of valuable resources into defence budgets instead of education and health (see Mernissi 1993). What is more, the measures that are
adopted to meet such military threats sometimes constitute threats to the security of individuals and
social groups. The sanctions regime adopted to rid Iraq of weapons of mass destruction has caused a problem of food insecurity for Iraqi people during the 1990s. In the aftermath of the US-led war on Iraq, Iraqi
people are still far from meeting their daily needs. Indeed, it is estimated that if it were not for the monthly basket distributed as part of the United Nations' 'Oil for Food' programme, 'approximately 80 percent of the Iraqi population
would become vulnerable to food insecurity' (Hurd 2003). Such concerns rarely make it into analyses on regional security in the Middle
East.
ENDI 2010 32
Security K Wave 1

MIDDLE EAST LINK


Representations are the root cause of policy failures and middle east
conflict
Bilgin, 2005 [Pinar, PhD International Politics, University of Wales, Aberystwyth, Department of International Relations
Bilkent Univ., Regional Security in the Middle East p. 12-5

Reflecting upon the history of US engagement with the Middle East, Douglas Little identifies representations of
the region as the problem behind policy failures . According to Little, it is 'American Orientalism' defined as
'a tendency to underestimate the peoples of the region and to overestimate America's ability to make a bad
situation better' that has often misled US policy-makers in their dealings with the region. Regarding the future, Little
(2002:314) writes: Although there is greater appreciation for the complexities of the Muslim world than a generation ago, most
Americans still view radical Islam as a cause for instant alarm. Having been fed a steady diet of books, films and
news reports depicting Arabs as demonic anti-Western others and Israelis as heroic pro-Western partners and
having watched in horror the events of 11 September, the American public understandably fears Osama bin
Laden and cheers Aladdin. Little's argument builds upon that of Edward Said in his 1978 book Orientalism, where the author
pointed to the relationship between representations and practice . Said's point was that the academic discourse of
Orientalism (defined as 'a style of thought based upon an ontological and epistemological distinction made between ―the Orient‖ and
[most of the time] ―the Occident‖' [Said 1995a: 2]) had not only helped to make the Middle East what it has become but
also made it difficult to become something else: a book on how to handle a fierce lion might … cause a series of books to
be produced on such subjects as the fierceness of lions, the origins of fierceness, and so forth. Similarly, as the focus of the text
centers more narrowly on the subject - no longer lions but their fierceness - we might expect that the ways by which it is
recommended that a lion's fierceness be handled will actually increase its fierceness, force it to be fierce since
that is what it is, and that is what in essence what we know or can only know about it. (Said 1995a: 94) This is because the Orientalist
discourse does not merely represent the 'Orient' but also lays down the rules that enable one to 'write, speak
and act meaningfully' (Agnew and Corbridge 1995:45). In his later works (see Said 1994b, 1995b, 1997, 2001) Said went on to show
how contemporary representations of the Middle East (and Islam) in the media (as well as academia) have reduced it to
terrorism and very little else. Said's argument is in line with E.P. Thompson's observation on the impact British historical
representations of India have had on Indian politics (Said 2001:44-5). According to Thompson, writings on India in English 'simply left out
the Indian side of things' thereby deepening the irreconcilability between Indians and the British. Thompson wrote: Our misrepresentation
of Indian history and character is one of the things that have so alienated the educated classes of India that even their moderate elements
have refused to help the Reforms [of colonial policy]. Those measures, because of this sullenness, have failed, when they deserved a better
fate. (quoted in Said 2001:45) Reading Thompson, one is reminded of the numerous attempts made by US policy-makers
during the Cold War to generate reform and modernisation movements in the Middle East; some of which attempts
have backfired (as with Iraq, Libya and Iran) (Little 2002:193-227). What Little, Thompson and Said are pointing to are the
different impact representations have on those who produce the representations and those who are represented.
What all share is the damaging effect representations have had on both groups of actors. According to Said, the
Middle East as a spatial representation has been repressive in that it has had 'the kind of authority … [that]
doesn't permit or make room for interventions on the part of those represented' (Said 2001:42). The Middle
Eastern security discourse, which is informed by this representation, has reflected the Cold War security concerns
of the great powers while neglecting that of regional states and peoples . Hence the argument that the current state
of regional insecurity in the Middle East has its roots in practices that have been informed by its
dominant representation: the 'Middle East'. By way of adopting this spatial representation, the Middle East has been
categorised in terms of its politics (as the region that 'best fits the realist theory of international politics' [Nye
2000:163]) and the type of foreign policy its 'nature' demands. In the immediate aftermath of the US-led war on Iraq, one
newspaper columnist warned: 'Middle East is not Europe' (Zaharna 2003). Indeed. Yet, this should not be taken to suggest that
the Middle East is destined to relive its insecure past. Such representations that emphasised Middle Eastern
insecurities without reflecting upon their roots have had the effect of privileging certain security practices (such
as the 1998-99 bombing campaign directed at obtaining Iraqi cooperation with the UN team inspecting the Iraqi weapons of mass
destruction programme) whilst marginalising others (such as the adoption of a more comprehensive long-term policy of
creating a nuclear-free zone in the Middle East). Becoming aware of the 'politics of the geographical specification of
politics' (Dalby 1991:274) and exploring the mutually constitutive relationship between (inventing) regions, and
(conceptions and practices of) security is not mere intellectual exercise; it helps reveal the role human
agency has played in the past and could play in the future. Such awareness , in turn, would enable one to begin
thinking differently about regional security to help constitute an alternative future whilst remaining sensitive
to regional actors' multiple and contending conceptions of security, what they view as referent object(s), and
how they think security should be sought in this part of the world.
ENDI 2010 33
Security K Wave 1

NORTH KOREA LINK


Claims of Korean ―instability‖ are rooted in exclusionary racism
Seng, 2002 [Tan See, Prof of Security Studies @ IDSS Singapore, July, ―What Fear Hath Wrought: Missile
Hysteria and The Writing of America, IDSS Commentary No. 28,
http://www.sipri.org/contents/library/0210.pdf]

Otherness, in Wolfowitz‘s rendition, is also discursively constituted along a moral/immoral – or, alternatively,
responsible/irresponsible – axis. Equally interesting is the notion that authoritarian or rogue-state leaders, besides
lacking in rationality and viewing problem solving as a form of weakness, are ―ruthless and avaricious‖ – an
intentional, not accidental, choice of predicates. That (and here we are left to infer) ―North Korea‖ or ―Iraq‖ is ruled by such
roguish elements can only mean that such states can, indeed they should, therefore be properly referred to as rogue states. Against these
inscriptions of immorality or amorality stand, in diametric contrast, moral ―America.‖ And here the unequal adoption by
Wolfowitz‘s discourse, in the case of ―democracies,‖ of the analytical level of state/regime connotes that all America, and not only its leaders or certain
individuals, is thereby kind, compassionate, altruistic – the polar opposite of all that rogue states, and possibly even China and Russia, represent. To be
sure, nowhere in his words does Wolfowitz imply that there are as such no immoral or irresponsible Americans. Nor does he even hint that all citizens of
the discursive effect is such
rogue states are therefore roguish; political correctness, after all, is the norm in these enlightened times. But
that we are left with the impression that leaders of rogue nations – Saddam Hussein, Kim Chong-il, and their ilk –
epitomize the darkest of the dark metaphysics of human nature. And roguish as such are their foreign policies. In his
evaluation of the missile threat from North Korea, the deputy CIA director asserted: Like everyone else, we knew the [Pyongyang] regime was brutal
within its borders and a menace beyond. Its commando raids into South Korea and its assassination attempts against successive South Korean presidents –
including the 1983 bombings in Rangoon that killed 21 people – were clear windows into the minds and morals of North Korean leaders.62 Again, it
bears reminding that the argument here does not refuse the historical ―reality‖ and tragic consequences either of Pyongyang‘s oppressive policies at home
or its ruinous forays abroad. In terms of exclusionary practices, however, interpretive conclusions concerning the brutality of
the Pyongyang regime cannot be separated from the morality axis on which this particular statement
turns. What, for instance, is the effect created by the use of the opening phrase, ―Like everyone else‖? To who exactly does ―everyone‖ refer? That this
analysis is intelligible at all depends upon the presupposition that this particular reading – an American reading, to be precise – is universally accepted by
one and all. But this is clearly not the case as implied by the vociferous and potentially violent tide of militant Muslims in Pakistan and parts of the
Middle East, who hold Washington in contempt for the latter‘s alleged ―brutality‖ and ―menace‖ toward, say, the Iraqis, (by proxy) the Palestinians, or
(most recently) the Afghans. As such, the discursive effect of the preceding constructions is the naturalization of the
Pyongyang regime as immoral, irresponsible, or just plain evil given the damning evidence of dastardly deeds that proffer
―clear windows into the minds and morals of North Korean leaders.‖ Further, that the enumerated acts above were those perpetrated by Kim Il-song and
not by his son, Kim Chong-il, seems not to matter in this analysis, although it is the latter Kim‘s government with whom the Bush Administration must
deal. This is not to imply that this intelligence estimate on Kim was essentially all caricature and thereby shorn of ―truth.‖ The CIA official continues in
his assessment: It is easy to caricature Kim Chong-il – either as a simple tyrant blind to his dilemma or as a technocratic champion of sweeping change.
But the extreme views of him tend to be the product of bias, ignorance, or wishful thinking. The reality is more complex… Like his father, he has been
shrewd enough to make bad behavior the keystone of his foreign policy. He knows that proliferation is something we want to stop. Thus, Kim Chong-il
has tried to drum up outside assistance by trading off international concerns about his missile programs and sales. He has – more subtly, of course – done
much the same thing with foreign fears of renewed famine and the chaos that could accompany any unravelling of his regime.63 The evident attempt at
nuance in the above analysis, however, does not preclude the continued deployment of representational practices along the axis of responsibility. ―Like his
father,‖ we are told, the ―shrewd‖ Kim makes ―bad behavior the keystone of his foreign policy‖ – an indication
of chronic irresponsibility in North Korea‘s international relations. We may note here the likely intrusive influence of another discourse, particularly that
on nineteenth-century European diplomacy as it figures in American intellectual and popular culture. As historian Barbara Tuchman once noted, for most
Americans the notion of diplomacy carries with it ―all the wicked devices of the Old World, spheres of influence, balances of power, secret treatises, triple
alliances‖64 and other such forms of Machiavellian intrigue for which America, idealized as the New World – a seemingly virginal, innocent, and
righteous identity – had no place. Indeed, just such a pristine identity is often adduced as the universal ideal to which all nations and peoples are presumed
to aspire – a point made forcefully in the earlier cited ―end of history‖ thesis popular in mainstream political debate at the close of the Cold War.65 In
other words, what is good for America is obviously good for the whole world (or, at least those parts that are ―rational,‖ ―responsible,‖ ―moral‖). ―Missile
defense,‖ one congressman averred, ―is for Americans, for Europeans, for Russians, and for all peace-loving peoples on the face of the Earth.‖66 Without
what those
ignoring or denying North Korean complicity in the light of its sizeable transfers of missile technology to the Middle East,
exclusionary practices produce is the materializing effect of a Pyongyang regime that, if anything, can
be expected to harm the US at the slightest provocation – a representation of danger that finds easy
resonance with American policymakers because of its familiarity rather than any likelihood of such an
eventuation. Further, what is effaced or erased by the above statement are plausible illustrations of bad behaviour in
American foreign policy: a policy orientation that, even by most orthodox accounts, has been realist – in both its prudential as well as
Machiavellian aspects – throughout much of the Cold War period.67 Indeed, this effacement stands out starkly in the light of resistant discourses – mostly
but not exclusively from European sources – which portray America as a rogue state68 given the apparent lack of ―strategic restraint‖ in its post-Cold War
foreign policy.69 Hence the tenuousness of such constructions of identity through excluding contradictions
and tensions that are as much a part of Self as it is of the Other.
ENDI 2010 34
Security K Wave 1

NORTH KOREA LINK

Representations of North Korea are rooted in ideological hegemony not objective data

Shim, 2008 [David, Phd Candidate @ GIGA Institute of Asian Studies, Paper prepared for
presentation at the 2008 ISA, Production, Hegemonization and Contestation of Discursive Hegemony:
The Case of the Six-Party Talks in Northeast Asia, www.allacademic.com/meta/p253290_index.html]
Laclau and Mouffe‘s (2001: chapter 2) concept of hegemony, which is used here, rely on a notion developed by Antonio Gramsci (1971). Gramsci broadened the
traditional notion of hegemony beyond the view of mapping hegemony in terms of leadership and dominance, which are based on material capabilities, by
introducing inter-subjective and ideological aspects into this concept. Accordingly, hegemony
contains the ability of a class (bourgeois) to project the world view over another (workers, peasantry) in terms of
the former, so that it is accepted as common sense or reality. His merit was to conceptualize hegemony in terms of power without the use of force to reach consent by the dominated class
through education and, what he calls, the role of intellectuals (―men of letters‖) such as philosophers, journalists and artists (Gramsci 1971: 5-43). The process of fixing meaning, that is, in terms of Laclau
and Mouffe (2001: 105), when an element (sign with unfixed meaning) is transformed through articulation into a moment (sign with fixed meaning), is hegemonic, since it reduces the range of possibilities

and excludes alternative meanings by determining the ways in which the signs are related to each other. That is to say, when meaning is fixed, i.e.
hegemonized, it determines, what can be thought, said or done in a meaningful way. 13 Applied to this
case, the exclusive character of a hegemonic discourse makes it unintelligible to make sense of

North Korea‘s nuclear program in terms of, for instance, energy needs, because – as it is argued –
practices of problematization hegemonized the ways of thinking, acting and speaking about North Korea.

Discursive hegemony can be regarded as the result of certain practices, in which a particular understanding or interpretation appears to be the natural order of things (Laclau/Mouffe 2001). This

naturalization consolidates a specific idea, which is taken for granted by involved actors and
makes sense of the(ir) world. As Hall (1998: 1055-7) argues, common sense resembles a hegemonic discourse, which is a dominant interpretation and representation of
reality and therefore accepted to be the valid truth and knowledge. Referring to the productive character of discursive hegemony, the Six-Party Talks can be regarded as an outcome of the dominating

interpretation of reality (cf. also Jackson 2005: 20; Cox 1983; Hajer 2005). The hegemonic discourse regarding North Korea provides
the framework for a specific interpretation in which the words , actions or policies of it are
attached with meaning, that is, are problematized. As Jacob Torfing argues ―a discursive truth regime […] specifies the
criteria for judging something to be true of false‖, and further states, that within such a discursive framework the criteria for
acknowledging something as true, right or good are negotiated and defined (Torfing 2005a: 14; 19; cf. also Mills 2004: 14-20).
However, important to note is, if one is able to define this yardstick, not only one is able to define what is right, good or true, but also what kinds of action are possible. In other words, if you can

mark someone or something with a specific label, then certain kinds of acts become feasible.14 Basically, it can
be stated that discursive hegemony depends on the interpretation and representation by actors of real events since the interpretation of non-existent facts would not make sense. But the existence of real

actions do not need to be carried


events does not necessarily have to be a prerequisite for hegemonizing interpretational and representational practices because

out, thus, to become a material fact, in order to be interpreted and represented in a certain way
(Campbell 1998: 3). Suh Jae-Jung (2004: 155) gives an example of this practice. In 1999 US intelligence agencies indicated to preparing

measures taken by North Korea to test fire a missile. Although the action was not yet
executed, it was treated as a fact, which involved and enabled certain implications and material
consequences such as the public criticism of North Korea, the issuance of statements, diplomatic activity and efforts to
hegemonize and secure this certain kind of reality, i.e. to build a broad majority to confirm this view on North Korea. In other words, the practices
of problematizing North Korea took place even before an action was done.
ENDI 2010 35
Security K Wave 1

CHINA LINK- THREAT

Representations of China as a threat ignore the normative value-judgments inherent to the


process of claiming to empirically know Chinese national and political identity—this makes
security threats self-fulfilling prophecies
Pan 04‘ – PhD in Political Science and International Relations and member of the International Studies
Association ISA (Chengxin Pan: ―The "China threat" in American self-imagination: the discursive
construction of other as power politics‖, Alternatives RC)

China and its relationship with the United States has long been a fascinating subject of study in the mainstream U.S. international
relations community. This is reflected, for example, in the current heated debates over whether China is primarily a
strategic threat to or a market bonanza for the United States and whether containment or engagement is
the best way to deal with it. (1) While U.S. China scholars argue fiercely over "what China precisely is," their debates have been underpinned by
some common ground, especially in terms of a positivist epistemology. Firstly, they believe that China is ultimately a knowable
object, whose reality can be, and ought to be, empirically revealed by scientific means. For example, after
expressing his dissatisfaction with often conflicting Western perceptions of China, David M. Lampton, former president of the National Committee on U.S.-
China Relations, suggests that "it is time to step back and look at where China is today, where it might be going, and what consequences that direction will hold
for the rest of the world." (2) Like many other China scholars, Lampton views his object of study as essentially "something we can stand back from and observe
with clinical detachment." (3) Secondly, associated with the first assumption, it is commonly believed that China scholars merely serve as "disinterested
observers" and that their studies of China are neutral, passive descriptions of reality. And thirdly, in pondering whether China poses a threat or offers an
opportunity to the United States, they rarely raise the question of "what the United States is." That is, the meaning of the United States is believed to be certain
and beyond doubt. I do not dismiss altogether the conventional ways of debating China. It is not the purpose of this article to venture my own "observation" of
"where China is today," nor to join the "containment" versus "engagement" debate per se. Rather, I want to contribute to a novel dimension of the China
debate by questioning the seemingly unproblematic assumptions shared by most China scholars in the mainstream IR community in the United States. To
perform this task, I will focus attention on a particularly significant component of the China debate; namely, the "China threat" literature. More specifically, I
want to argue thatU.S. conceptions of China as a threatening other are always intrinsically linked to
how U.S. policymakers/mainstream China specialists see themselves (as representatives of the
indispensable, security-conscious nation, for example). As such, they are not value-free, objective descriptions of
an independent, preexisting Chinese reality out there, but are better understood as a kind of normative,
meaning-giving practice that often legitimates power politics in U.S.-China relations and helps
transform the "China threat" into social reality. In other words, it is self-fulfilling in practice, and is always
part of the "China threat" problem it purports merely to describe. In doing so, I seek to bring to the fore two interconnected
themes of self/other constructions and of theory as practice inherent in the "China threat" literature--themes that have been overridden and rendered largely
invisible by those common positivist assumptions. These themes are of course nothing new nor peculiar to the "China threat" literature. They have been
identified elsewhere by critics of some conventional fields of study such as ethnography, anthropology, oriental studies, political science, and international
relations. (4) Yet, so far, the China field in the West in general and the
U.S. "China threat" literature in particular have shown
remarkable resistance to systematic critical reflection on both their normative status as discursive practice
and their enormous practical implications for international politics. (5) It is in this context that this article seeks to make a
contribution. I begin with a brief survey of the "China threat" argument in contemporary U.S. international relations literature, followed by an investigation of
how this particular argument about China is a discursive construction of other, which is predicated on the predominant way in which the United States imagines
itself as the universal, indispensable nation-state in constant need of absolute certainty and security. Finally, this article will illustrate some of the dangerous
practical consequences of the "China threat" discourse for contemporary U.S.-China relations, particularly with regard to the 1995-1996 Taiwan Strait missile
crisis and the 2001 spy-plane incident.
ENDI 2010 36
Security K Wave 1

CHINA LINK - THREAT


The characterization of China has a threat robs it of subjectivity and makes it only strategically
useful for the U.S. to construct as a threat

Pan 04‘ – PhD in Political Science and International Relations and member of the International Studies
Association ISA (Chengxin Pan: Alternatives: Global, Local, Political, Vol. 29 Pg. 305 -307 RC)

By now, it seems clear that neither China's capabilities nor intentions really matter. Rather, almost by its mere
geographical existence, China has been qualified as an absolute strategic "other," a discursive construct
from which it cannot escape. Because of this, "China" in U.S. IR discourse has been objectified and deprived of
its own subjectivity and exists mainly in and for the U.S. self. Little wonder that for many U.S. China specialists, China
becomes merely a "national security concern" for the United States, with the "severe disproportion between the keen
attention to China as a security concern and the intractable neglect of China's [own] security concerns in the current debate."62 At this point, at
issue here is no longer whether the "China threat" argument is true or false, but is rather its
reflection of a shared positivist mentality among mainstream China experts that they know China better than
do the Chinese themselves. "We" alone can know for sure that they consider "us" their enemy and thus
pose a menace to "us." Such an account of China, in many ways, strongly seems to resemble Orientalists'
problematic distinction between the West and the Orient. Like orientalism, the U.S. construction of the Chinese
"other" does not require that China acknowledge the validity of that dichotomous construction.
Indeed, as Edward Said points out, "It is enough for 'us' to set up these distinctions in our own minds;
[and] 'they' become 'they' accordingly. "64

China threats are products of narcissistic understandings of the U.S.‘s role in global politics
Pan 04‘ – PhD in Political Science and International Relations and member of the International Studies
Association ISA (Chengxin Pan: Alternatives: Global, Local, Political, Vol. 29 Pg. 305 -307 RC)

I have argued above that the


"China threat" argument in mainstream U.S. IR literature is derived, primarily, from a discursive
construction of otherness. This construction is predicated on a particular narcissistic
understanding of the U.S. self and on a positivist-based realism, concerned with absolute
certainty and security, a concern central to the dominant U.S. self-imaginary. Within these
frameworks, it seems imperative that China be treated as a threatening, absolute other since it is
unable to fit neatly into the U.S.-led evolutionary scheme or guarantee absolute security for the
United States, so that U.S. power preponderance in the post–Cold War world can still be legitimated. Not only
does this reductionist representation come at the expense of understanding China as a dynamic,
multifaceted country but it leads inevitably to a policy of containment that, in turn, tends to enhance the
influence of realpolitik thinking, nationalist extremism, and hard-line stance in today's China. Even a
small dose of the containment strategy is likely to have a highly dramatic impact on U.S.-China
relations, as the 1995-1996 missile crisis and the 2001 spy-plane incident have vividly attested. In this respect, Chalmers Johnson is right
when he suggests that "a policy of containment toward China implies the possibility of war, just as it
did during the Cold War vis-a-vis the former Soviet Union. The balance of terror prevented war
between the United States and the Soviet Union, but this may not work in the case of China."93
ENDI 2010 37
Security K Wave 1

CHINA LINK – ECONOMY/COMPETITIVENESS

Representations of China‘s advanced economic competitiveness construct it as a threat to the


U.S.
Pan 04‘ – PhD in Political Science and International Relations and member of the International Studies
Association ISA (Chengxin Pan: ―The "China threat" in American self-imagination: the discursive
construction of other as power politics‖, Alternatives RC)

That China constitutes a growing "threat" to the United States is arguably one of the most important
"discoveries" by U.S. IR scholars in the post-Cold War era. For many, this "threat" is obvious for a variety of reasons concerning
economic, military, cultural, and political dimensions. First and foremost, much of today's alarm about the "rise of China" resolves
around the phenomenal development of the Chinese economy during the past twenty-five years: Its overall size
has more than quadrupled since 1978. China expert Nicholas Lardy of the Brookings Institution suggested that "the pace of China's industrial development and
trade expansion is unparalleled in modern economic history." He went on: "While this has led to unprecedented improvements in Chinese incomes and living
standards, it also poses challenges for other countries." (6) 
 
 One such challenge is thought to be job losses in the United States. A recent study done for a
U.S. congressional panel found that at least 760,000 U.S. manufacturing jobs have migrated to China since 1992. (7) Associated with this economic
boom is China's growing trade surplus with the United States, which, according to Time magazine journalists Richard Bernstein and Ross
Munro, increased nearly tenfold from $3.5 billion in 1988 to roughly $33.8 billion in 1995. This trade imbalance, as they put it, is 
 
 a function of a
Chinese strategy to target certain industries and to undersell American competition via a system of subsidies and high tariffs.
And that is why the deficit is harmful to the American economy and likely to become an area of ever greater conflict in bilateral
relations in the future. (8) 
 
 For many, also frightening is a prospect of the emergence of so-called "Greater
China" (a vast economic zone consisting of mainland China, Hong Kong, and Taiwan). As Harry Harding points out,
"Although [Greater China] was originally intended in [a] benign economic sense, ... in some quarters it evokes much more aggressive analogies, such as the
Greater East Asia Co-Prosperity Sphere or Greater Germany." (9) 
 
 In this context, some
believe that China's economic challenge
inevitably gives rise to a simultaneous military threat. As Denny Roy argues: "A stronger, wealthier China would
have greater where-withal to increase its arsenal of nuclear-armed ICBMs and to increase their lethality
through improvements in range, accuracy, and survivability. If China continues its rate of economic expansion,
absolute growth in Chinese nuclear capabilities should be expected to increase." (10) Furthermore, U.S. Congressman Bob Schaffer
claimed that China's military buildup, already under way at an alarming rate, was aimed at the United States . (11)
ENDI 2010 38
Security K Wave 1

CHINA LINK – HOSTILE RISE / POWER VACUUM

Predictions of China‘s hostile rise are not objective realities but discursive constructions of
China‘s otherness designed to bolster U.S. hegemony
Pan 04‘ – PhD in Political Science and International Relations and member of the International Studies
Association ISA (Chengxin Pan: Alternatives: Global, Local, Political, Vol. 29 Pg. 305 -307 RC)

Apart from these so-called "domestic" reasons for the "China threat," some commentators arrive at a similar conclusion
based on the historical experience of power realignment as a result of the rise and fall of great powers. China, from this perspective, is regarded as
the most likely candidate to fill the power vacuum created by the end of U.S.-Soviet rivalry in East Asia.
This, according to Kenneth Lieberthal at the University of Michigan (and formerly of the U.S. State Department), "will inevitably present major
challenges to the United States and the rest of the international system since the perennial question has been
how the international community can accommodate the ambitions of newly powerful states, which have
always forced realignment of the international system and have more often than not led to war." (21) 
 
 For
this reason, the rise of China has often been likened to that of Nazi Germany and militarist Japan on the eve
of the two world wars. For example, Richard K. Betts and Thomas J. Christensen argue: 
 
 Like Germany a century ago, China is a late-blooming
great power emerging into a world already ordered strategically by earlier arrivals; a continental power surrounded by other powers who are collectively stronger
but individually weaker (with the exception of the United States and, perhaps, Japan); a bustling country with great expectations, dissatisfied with its place in the
international pecking order, if only with regard to international prestige and respect. The
quest for a rightful "place in the sun" will, it is
argued, inevitably foster growing friction with Japan, Russia, India or the United States. (22) 
 
 At this point, it
seems there has been enough reason and empirical evidence for the United States to be vigilant about China's future ambition. While there are debates over the
extent to which the threat is imminent or to which approaches might best explain it, the "objective" quality of such a threat has been taken for granted. In the
words of Walter McDougall, the Pulitzer Prize-winning historian and strategic thinker at the University of Pennsylvania, recognizing the "China threat" is
"commonsense geopolitics." (23) For Huntington, the challenge of "Greater China" to the West is simply a rapidly growing cultural, economic, and political
"reality." (24) Similarly, when they claim that "China can pose a grave problem," Betts and Christensen are convinced that they are merely referring to "the
truth." (25) 
 
 In the following sections, I want to question this "truth," and, more generally, question the objective, self-evidentiary attitudes that underpin it.
In my view, the
"China threat" literature is best understood as a particular kind of discursive practice that
dichotomizes the West and China as self and other. In this sense, the "truism" that China presents a
growing threat is not so much an objective reflection of contemporary global reality, per se, as it is a
discursive construction of otherness that acts to bolster the hegemonic leadership of the United
States in the post-Cold War world. Therefore, to have a better understanding of how the discursive
construction of China as a "threat" takes place, it is now necessary to turn attention to a particularly
dominant way of U.S. self-imagination.
ENDI 2010 39
Security K Wave 1

RUSSIA LINK

Russian threats of accidents, nuclear use, and disintegration are falsely constructed and create
self-fulfilling prophecies
Rumer and Sokolsky 01 (*Eugene B. Rumer served at the State Department, on the staff of the National
Security Council, and at the Rand Corporation. He holds degrees from Boston University (BA), Georgetown (MA),
and MIT (PhD). **Richard D. Sokolsky served as the director of the Office of Strategic Policy and Negotiations in
the Department of State. He has published numerous articles on foreign and national security policy in leading
journals and newspapers. ―Normalizing U.S.-Russian Relations‖ Published in Strategic Forum; Institute for national
strategic studies. National Defense university. April 2001. RC)

Ten years after the end of the Cold War, mutual hopes that a comprehensive partnership would replace
containment as the major organizing theme in U.S.-Russian relations have not been realized. The record of the
1990s has left both Russia and the United States unsatisfied. Russia looks back at the decade with bitterness and a feeling of being marginalized and slighted
by the world‘s sole remaining superpower. It is also disappointed by its experience with Western-style reforms and
mistrustful of American intentions. The United States is equally disappointed with Russia‘s lack of focus,
inability to engage effectively abroad, and failure to implement major reforms at home. A comprehensive partnership
is out of the question. Renewed competition or active containment are also not credible as organizing principles. Russia‘s economic, military and
political/ideological weakness makes it an unlikely target of either U.S. competition or containment. Not only is Russia no longer a superpower, but its status as
a regional power is in doubt. Current thinking about Russia is divided among four basic approaches: Forget Russia, Enfant Terrible Russia, Evil Russia, and
Russia First. The Forget Russia view holds that Russia is too weak, too corrupt, and too chaotic to matter. After
10 years of trying to help Russia, the United States should focus its resources and attention on more deserving and important world issues. The Enfant Terrible
view holds that, although Russia has been an irresponsible and irritating partner, it is too weak to hurt the United States and therefore need not be feared in
earnest. President Vladimir Putin‘s visits to Cuba and North Korea, courtship of Slobodan Milosevic, and welcoming of Iranian President Mohammad Khatami
to Moscow are of little strategic consequence and thus not worth our attention. This view presupposes the existence of an important U.S.-Russian bilateral
agenda and the need to protect it from childish and irresponsible Russian grandstanding. The
Evil Russia view holds that Russian
courtship of Cuba, Iran, Iraq, and North Korea is a deliberate effort to undermine U.S. influence in the
world and recreate the Soviet empire. Analysts embracing this view take less notice of Russia‘s diminished
capabilities than of ambitious rhetoric by Russian politicians. Given Russia‘s evil purposes, the United States is already on a
collision course with it and might as well do everything it can to box Russia in. The Russia First view holds that Russia still is the most important issue on the
U.S. foreign policy agenda. It accepts the premise that the two sides have shared interests and that Russia, once reborn as a stable, prosperous democracy, can
be a U.S. partner and ally. Therefore, the United States should actively assist Russia in its transformation and engage it in a broad and intense relationship with
renewed vigor and creativity. There are shortcomings in all of these approaches. Notwithstanding its precipitous decline, to Forget Russia is clearly not an
option: the country‘s geographic expanse, nuclear arsenal, and proliferation potential simply make it impossible for U.S. policymakers to ignore. The Enfant
The Evil Russia view risks
Terrible view fails to take Russia seriously and ignores the very real problems that exist between the two countries.
inflating the threat and making the myth of evil Russia a self-fulfilling prophecy. The Russia First view is not
grounded in reality. After a decade of failure, it should be clear that neither the specter of Russia‘s past nor the promise of its future warrants a position near the
top of the U.S. foreign policy agenda. The Need for Normalcy Russia‘s external weakness and internal problems have left the United States without an effective
interlocutor, either as partner or competitor. Thus, the United States should deal with Russia on a case-by-case basis to advance our interests, in much the same
way we deal with most other countries. This path will sometimes lead toward partnership with Rus- sia and at other times toward competition. It may even
result in a situation where Russia and the United States find themselves as partners and competitors simultaneously in different parts of the world or on
different issues. Given its size, history, strategic nuclear capabilities, and future potential, one is tempted to overstate the importance of relations with Russia and
put them at the top of the U.S. national security agenda. Except for geography and nuclear weapons, however, there is little at this stage to justify making
relations with Russia a top priority. Undoubtedly,
Russia can inflict unacceptable damage on the United States. But
fear of Russian nuclear weapons should not be the driving element of the relationship. The hostility and
ideological differences that divided the superpowers during the Cold War are gone. The prospect of Russia
consolidating and rebuilding itself under a militant authoritarian, nationalist regime is remote. Therefore, fears
of a deliberate surprise (attack on the United States are unjustified. Despite a number of bilateral undertakings outside the Cold
War-style security agenda, ranging from regional diplomacy in the Balkans to investment, U.S. engagement with Russia, with the notable exception of the
Cooperative Threat Reduction (CTR) Initiative, is limited.
ENDI 2010 40
Security K Wave 1

PEACE LINK

Peace is the overall concept of security. Dangers are the product of our own actions forcing peace
threats to become risks of security
Waever 2004 [Ole, Ph.D. in Political Science and Professor of International Relations at COPRI, ―Peace and Security‖,
Contemporary Security Analysis and Copenhagen Peace Research, pg.62-63]

After the end of the Cold War, peace reappeared as a Western concept. The 'absolute' concept was revalued when it seemed closer
to realisation. With the 'end of history' in sight, liberalism mutated back from scepticist, Popperian Cold War liberalism to the more evolutionary and optimist belief in its
own truth. When the task of the West changed from fighting a Cold War to building a 'new world order', it suddenly remembered that it actually had a long-term vision of peace
as democracy (and/or liberalism) (Rasmussen 2001; Williams 2001). President Bush senior declared in 1989, 'Once again, it is a time for peace' (quoted by Rasmussen 2001:
341). The famous 'New World Order' speech at the end of the Gulf War (March 6, 1991) was phrased mostly in terms of peace – 'enduring peace must be our mission'. NATO
enlarge- ment is so hard for Russia and others to oppose because it is presented apolitically as the mere expansion of the democratic peace community (Williams 2001). The war
on terror after 11 September 2001 has surpris- ingly few references to either peace or security – operation 'Enduring Freedom' – but President George W. Bush's address on 7
October 2001 ended with 'Peace and freedom will prevail', and the (in)famous 'axis of evil' was presented (29 January, 2002) in terms of a 'threat to peace'. Peace has become the
overarching concept of the two examined in this chapter. Security in turn, is
gradually swallowed up into a generalised concern about 'risk'.
Society's reflections on itself are increasingly in terms of risk ('risk society'). More and more dangers are the product of our
own actions, and fewer and fewer attributable to forces completely external to ourselves – thus threats become risks
(Luhmann 1990). This goes for forms of production and their effects on the environment, and it goes for international affairs,
where it is hard to see the war on terrorism as a pure reaction to something coming to the West from elsewhere. Western
actions in relation to Middle East peace processes, religion, migration and global economic policy are part of what might
produce future terrorism. The short-term reaction to the 11 September attacks on the USA in 2001 might be a re-assertion of single-minded
aspirations for absolute security with little concern for liberty and for boomerang effects on future security (Bigo 2002), but in general debates, the 'risk'
way of thinking about international affairs is making itself increasingly felt. We have seen during the last twenty years a
spread of the originally specifically international concept of security in its securitisation function to more and more spheres
of 'domestic' life, and now society takes its revenge by transforming the concept of security along lines of risk thinking
(Waever 2002). Politically, the concepts of peace and security are changing places in these years. 'Security studies' and 'peace research'
were shaped in important ways by the particular Cold War context, though not the way it is often implied in fast politicians' statements about the
post-Cold War irrelevance of peace research. 'Peace research' and 'security studies' (or rather 'strategic studies') meant, respectively, to
oppose or to accept the official Western policy problematique. Today, it is the other way round. 'Peace research' might be
dated because peace is so apologetic as to be intellectually uninteresting, while 'security' is potentially the name of a radical,
subversive agenda.

Maintainence of global civil peace is a front in the perpetual war that structures dominant social
relations
FOUCAULT 1976 – PROF PHILOSOPHY COLLEGE DE FRANCE
POWER/KNOWLEDGE: SELECTED INTERVIEWS, ED. GORDON, PAGE 90-91
Then again, there is a second reply we might make: if power is properly speaking the way in which relations of forces are deployed and
given concrete expression, rather than analysing it in terms of cession, contract or alienation, or functionally in terms of its maintenance of the
relations of production, should we not analyse it primarily in terms of struggle, conflict and war? One would then confront the original hypothesis,
according to which power is essentially repression, with a second hypothesis to the effect that power is war, a war continued by
other means. This reversal of Clausewitz's assertion that war is politics continued by other means has -a triple significance: in the first place, it implies
that the relations of power that function in a society such as ours essentially rest upon a definite relation of forces that is
established at a determinate, historically specifiable moment, in war and by war. Furthermore, if it is true that political power puts an end
to war, that it installs, or tries to install, the reign of peace in civil society, this by no means implies that it suspends the effects
of war or neutralises the disequilibrium revealed in the final battle. The role of political power, on this hypothesis, is perpetually
to reinscribe this relation through a form of unspoken warfare; to re-inscribe it in social institutions, in economic
inequalities, in language, in the bodies themselves of each and everyone of us. So this would be the first meaning to assign to the
inversion of Clausewitz's aphorism that war is politics continued by other means. It consists in seeing politics as sanctioning and
upholding the disequilibrium of forces that was displayed in war. But there is also something else that the inversion signifies, namely,
that none of the political struggles, the conflicts waged over power, with power, for power, the alterations in the relations of forces, the favouring
of certain tendencies, the reinforcements etc., etc., that come about within this `civil peace'-that none of these phenomena in a political system
should be interpreted except as the continuation of war. They should, that is to say, be understood as episodes, factions and
displacements in that same war. Even when one writes the history of peace and its institutions, it is always the history of this war that one is writing.
The third, and final, meaning to be assigned to the inversion of Clausewitz's aphorism, is that the end result can only be the outcome of war ,
that is, of a contest of strength, to be decided in the last analyses by recourse to arms. The political battle would cease with this
final battle. Only a final battle of that kind would put an end, once and for all, to the exercise of power as continual war.
ENDI 2010 41
Security K Wave 1

POSITIVISM/EMPIRICISM LINK

Positivist epistemology ignores the roots of scientific assumptions about international relations in
the memory of the Cold War, making all of their conclusions products of confirmation bias
Ashley 1984 [Richard, professor of political science at Arizona State University, ―The Poverty of Neorealism,‖
International Organization, Vol. 38, No. 2 (Spring, 1984), pp. 225-286, jstor]

Despite its errors, its idealism, and its emptiness, neorealism succeeds in its illusion of greatness because it at once cues and
caricatures the ghosts of revolutions past, most of all the behavioral revolution in the Cold War study of international re-
lations. Awakened is a remembrance of a naturalistic model of science, the very model in whose name the behavioral revolutionaries marched. Awak-
ened, too, is a one-time revolutionary faith that the light of science will illuminate the conditions of state action, thereby reducing the chances of
miscalculation, overcommitment, and their sometimes disastrous conse- quences. Summoned forth once again is the sense of urgency of the
darkest days of the Cold War. Objectivity, neutrality, detachment, "state-as-actor," a "technology of peace"-these were
among the slogans. Recollecting this heroic revolutionary project, neorealism ennobles its followers. Never mind that the faith in
naturalistic science and the harmonizing force of reason implicit in these memories do violence to the very core of classical
realism's internationalist thought, including its long-standing resistance to behavioralist methods. Never mind that the darkly pessimistic side of
Morgenthau, Carr, Wight, and Herz does not square with the behavioralists' optimism. With Gilpin, one can remember classical realism not as an
embodiment of a continuing dialectical struggle between absolutist darkness and bourgeois Enlightenment but only as a product of the latter: Embedded
in most social sciences and in the study of international relations is the belief that through science and reason the human
race can gain control over its destiny. Through the advancement of knowl- edge, humanity can learn to master the blind forces and construct a
sci- ence of peace. Through an understanding of the sources of our actions and the consequences of our acts, human rationality
should be able to guide statesmen through the crisis of a decaying world order to a reno- vated and stable world order. The
fundamental problem faced, this ar- gument continues, is not uncontrollable passions but ignorance. Political realism is, of course, the embodiment of this
faith in reason and science. An offspring of modern science and the Enlightenment, re- alism holds that through calculations of power and national interest
statesmen can create order out of anarchy and thereby moderate the in- evitable conflicts of autonomous, self-centered, and competitive states.... [T]his
faith that a 'science of international relations' will ulti- mately save mankind still lies at the heart of its studies.8' Like the French gazing upon Louis
Bonaparte, neorealism's followers glimpse in this caricature of past revolutions an image that reflects well on them, that calls
to mind the best images of themselves. Yet like the French, the followers quickly experience a sickening jerk as the whole
project is yanked too soon into closure. The neorealist caricature of science has not deployed its revolutionary images to "fire the imagination,"
or to "glorify the new struggles," but to seal discourse within a continuous "parody of the old." Its discourse is now frozen in acquiescence to
the Cold War conditions of the revolution it recalled: competition among states mu- tually preparing for war. As Gilpin writes,
"The advance of technology may open up opportunities for mutual benefit, but it also increases the power available for political struggle. The advance of
human reason and under- standing will not end this power struggle, but it does make possible a more enlightened understanding and pursuit of national
self-interest."82 How painful this is: the revolutionary science of peace has become a technology of the state! Was it always so? By the time this awful
realization comes, it is too late for such reflections. The neorealist orrery has spun its followers into its arc. A "grotesque mediocrity" reigns. Such
reflections suggest that a serious problem awaits the critic of neo- realism. Despite its serious flaws and totalitarian nature, neorealist orthodoxy will not be
dislodged from its lofty status by the force of logical criticism alone. For neorealism's power is largely due not to its truth or the
consistency of its logic but to its capacity to elicit the collective recognition of women and men, scientists mostly, who
must somehow organize their expectations and coordinate their practices in light of commonly remembered experiences.
True, as I have noted, neorealist theory is like all intellectualist theory in that it contains no terms that would allow it to reflect critically on the practical
social basis and limits of its own power. Yet the fact remains, "the ghosts of revolutions past" are neorealism's main allies. At least, so they appear.
Everywhere the collective remembrance of the study of international politics is on neorealism's side .
ENDI 2010 42
Security K Wave 1

POSITIVISM LINK - STATE

Positivism in IR falsely presupposes the naturalness of states as prior to IR system


Ashley 1984 [Richard, professor of political science at Arizona State University, ―The Poverty of Neorealism,‖
International Organization, Vol. 38, No. 2 (Spring, 1984), pp. 225-286, jstor]

The issue, however, is the theoretical discourse of neorealism as a move- ment, not the protective clauses that individual neorealists deploy to preempt or
deflect criticisms of that discourse's limits. Once one enters this theoretical discourse among neorealists, the state-as-actor model
needs no defense. It stands without challenge. Like Waltz, one simply assumes that states have the status of unitary actors .32
Or, like Gilpin, one refuses to be deterred by the mountainous inconsistencies between the state as a coalition of coalitions (presumably in opposition to
the losing coalitions against which the winning coalition is formed) and the state as a provider of public goods, protector of citizens' welfare, and solver of
the free-rider problem in the name of winners and losers alike. Knowing that the "objectives and foreign policies of states are
determined primarily by the interests of their dominant members or ruling coalitions ,"3 one nonetheless simply joins the
victors in proclaiming the state a singular actor with a unified set of objectives in the name of the collective good . This
proclamation is the starting point of theoretical discourse, one of the unexamined assumptions from which theoretical discourse proceeds. In short, the
state-as-actor assumption is a metaphysical commitment prior to science and exempted from scientific criticism.
Despite neorealism's much ballyhooed emphasis on the role of hard falsifying tests as the measure of theoretical progress,
neorealism immunizes its statist commitments from any form of falsification. Excluded, for instance, is the historically testable
hypothesis that the state-as-actor construct might be not a first-order given of international political life but part of a
historical justificatory framework by which dominant coalitions legitimize and secure consent for their precarious
conditions of rule. Two implications of this "state-centricity," itself an ontological principle of neorealist theorizing, deserve emphasis. The first is
obvious. As a frame- work for the interpretation of international politics, neorealist theory cannot accord recognition to-it
cannot even comprehend-those global collectivist concepts that are irreducible to logical combinations of state-bounded re-
lations. In other words, global collectivist concepts-concepts of transnational class relations, say, or the interests of humankind-can be granted an
objective status only to the extent that they can be interpreted as aggregations of relations and interests having logically and historically prior roots within
state-bounded societies. Much as the "individual" is a prism through which methodological individualists comprehend
collectivist concepts as aggrega- tions of individual wants, needs, beliefs, and actions, so also does the neorealist refract all
global collectivist concepts through the prism of the state.34 Importantly, this means that neorealist theory implicitly takes a side
amidst contending political interests. Whatever the personal commitments of in- dividual neorealists might be, neorealist theory allies with,
accords recognition to, and gives expression to those class and sectoral interests (the apexes of Waltz's domestic hierarchies or Gilpin's victorious
coalitions of coalitions) that are actually or potentially congruent with state interests and legitimations. It implicitly opposes and denies
recognition to those class and human interests which cannot be reduced to concatenations of state interests or transnational
coalitions of domestic interests. The second implication takes longer to spell out, for it relates to neorealist "structuralism"-the neorealist position
with respect to structures of the international system. Reflecting on the fourth element of structuralist ar- gument presented above, one might expect the
neorealist to accord to the structure of the international system an identity independent of the parts or units (states-as-actors in this case); the identities of
the units would be supplied via differentiation. The neorealist orrery disappoints these expectations, how- ever. For the neorealist, the state is
ontologically prior to the international system. The system's structure is produced by defining states as individual unities
and then by noting properties that emerge when several such unities are brought into mutual reference . For the neorealist, it
is impossible to describe international structures without first fashioning a concept of the state-as-actor. The proper analogy, as
Waltz points out, is classical economic theory- microtheory, not macrotheory. As Waltz puts it, "International-political sys- tems, like economic markets,
are formed by the coaction of self-regarding units." They "are individualist in origin, spontaneously generated, and un- intended."35
ENDI 2010 43
Security K Wave 1

ROOT CAUSE LINK

Identifying root causes relies on a teleological understanding of progress, defining the human
community according to the boundaries of modernization and development and excluding all
others
Der Derian 98 (James, a Watson Institute research professor of international studies and directs the
Information Technology, War, and Peace Project and the Global Media Project,
“International/Intertextual Relations: Postmodern Readings of World Politics‖, Lexington Books, p.26)
Enormous literatures now exist on both the proliferation of research orientations and the claim that the substantive character of international relations is
being transformed. However, both literatures rest upon a stance toward the prior claim that there is a tradition. This claim has been subject to relatively
little attention-especially in the United States, where the analysis of international relations has absorbed the unfortunate habit,
characteristic of political science in general, of treating "the tradition" as both somehow naturally given in the "great texts"
and largely irrelevant to the analysis of modern human affairs. Even where the history of political thought is taken seriously, and where the
difficulties in the way of understanding any particular text are well known, it remains almost a truism to claim that there is an identifiable tradition
of international theory against which current tendencies in both theory and practice can be situated, measured, and judged .
Reference to such a tradition may be justified as a simple practical convenience. The story to be told has to begin somewhere. But it is not always
easy to begin at the beginning, if only because the identification of a point of origin depends on where we think we are
now. Thus, a practical convenience is always liable to turn into a powerful myth of origin. Other points of departure are closed off. 2 The
identification of a tradition of international relations theory has now become especially problematic. We live in a world in
which there has been a proliferation not only of research paradigms in the academic analysis of international relations, but more
generally, of myths of origin. The Hegelian trek to universality still echoes as "progress/' "development," or "modernization," but living within
"modernity," we are just as likely to be bemused by histories as seduced by Geist. Even from the centers of fading empires, and ample
evidence of fundamentalist self-righteousness about past and future, origins shift and recede. Reified temporal horizons give
meaning to where we think we may be going. They also provide a sense of who this‖ we.‘ All too often, "we turn out to be
those who have "progressed," "developed," or "modernized,"[and] to be distinguished from "they" who have not. These
horizons seem increasingly tenuous, certainly inadequate as a way of orienting either serious academic analysis or progressive political
practice.
ENDI 2010 44
Security K Wave 1

NEOREALISM LINK

Neorealism falsely atomizes states while naturalizing and forgiving violent uses of power—it is
locked into a self-perpetuating state-centric paradigm
Ashley 1984 [Richard, professor of political science at Arizona State University, ―The Poverty of Neorealism,‖
International Organization, Vol. 38, No. 2 (Spring, 1984), pp. 225-286, jstor]

The autonomy of the neorealist whole is established precisely from the hypostatized point of view of the idealized parts
whose appearances as independent entities provided the starting point of the analysis, the basic material, the props without
which the whole physical structure could never have been erected . From start to finish, we never escape or penetrate
these appearances. From start to finish, Waltz's is an atomistic conception of the international system. At the same time, once neorealists do arrive at
their physicalistic notion of structure, they do attribute to it some of the qualities of structure in structuralist thought. Neorealists do tend to grant to the
international political system "absolute predominance over the parts." In neorealism, as in struc- turalism, diachrony is subordinated to synchrony, and
change is interpretable solely within the fixed logic of the system. And neorealists, like structuralists, do tend to regard the structure that they describe in
the singular. Thus, as noted earlier, there are definite isomorphisms between aspects of neorealist thought and structuralist principles. This, however, is no
compliment. For what it means is that neorealism gives us the worst of two worlds. In neorealism we have atomism's super- ficiality
combined with structuralism's closure such that, once we are drawn into the neorealist circle, we are condemned to circulate
entirely at the surface level of appearances. And what an idealist circle it is! What we have in neorealism's so-called
structuralism is the commonsense idealism of the powerful, projected onto the whole in a way that at once necessitates and
forgives that power. It is the statist idealism developed from the point of view of the one state (or, more properly, the dominant
coalition) that can afford the illusion that it is a finished state-as-actor because, for a time, it is positioned such that the whole
world pays the price of its illusions. With apologies to E. P. Thompson, I would suggest that there is a certain "snake-like" quality to
neorealist structuralism. The head of the snake is an unreflective state-as-actor, which knows itself only to rely on itself and
which will not recognize its own limits or dependence upon the world beyond its skin. It slithers around hissing "self-help"
and projecting its own unreflectivity onto the world . Finding its own unreflectiveness clearly reflected in others, it gets its
own tail into its mouth, and the system is thus defined. Asked to describe the system so defined, the snake says that it
reproduces itself, and it swallows more of its tail. What, though, of the values or norms of this system? The values and norms, the snake
answers, are those that reflect the power and interests of the powerful and interested . What, then, of power? The snake-or what is left of
it, for it is now a wriggling knot- has an answer for this, too. Power is rooted in those capabilities which provide a basis for the state-as-
actor's autonomy. And what of autonomy? In a final gulp, the snake answers. Autonomy is the state-as-actor's privilege of not having to
reflect because the whole world bends to its unreflected projections of itself. "Plop! The snake has disappeared into total
theoretical vacuity."68
ENDI 2010 45
Security K Wave 1

PROLIFERATION LINK

The affirmative takes the supposed problem of proliferation as a given—refusing to take it for
granted, however, shows the process by which the discourse of proliferation shapes the actions
and interests of agents in order to precipitate and incentivize the spread of nuclear technology
Mutimer 2000 [David, associate professor of political science at York University and Deputy Director of the Center for
International and Security Studies, The Weapons State, pg 18-19]

Charles Taylor has provided a clear example of the nature of constitutive intersubjective meanings in practices: "Take the practice of deciding things by
majority vote. It carries with it certain standards, of valid and invalid voting, and valid and invalid results, without which it would not be the practice that it
is.''1' All those who participate in the practice must share an image of the practice in which they are engaged . They must share a
certain collection of rules for fair and unfair voting, as well as knowing what essential behaviors they are expected to perform. They must also understaud
that they are independent agents but also parts of a collective who can decide as a whole through the aggregation of independent decisions. As Taylor
concludes, "In this way, we say that the practices which make up a society require certain self-descriptions on the part of the participants."19 The image
of majority voting constitutes the practice of voting by enabling the actors and actions necessary for the practice and defining
the relationships between the actors and those between the actors and the practice. The same is true for the practices in which states engage,
which are the object of study in international relations. A practice such as waging war, perhaps the definitive practice of the
traditional study of international relations, is conducted in terms of certain standards, as is voting.20 Intersubjectively held meanings
establish the conditions under which war may or may not be waged, as well as establishing which violent conduct is and
which is not to be counted as war. The image constitutive of war is socially held, adjudged, contested, and taught.
Thus, when the United States went to war in Vietnam, it was recognized by the society of states to be waging war, despite
its subjective labeling of the violence as a police action. On the other hand, the U.S. War on Drugs was recognized by those
same states to be metaphorically warlike rather than an instance of the practice of waging war, despite the use of military
and paramilitary violence. If intersubjective meanings constitute practices, engaging in practices involves acting toward the world in the terms
provided by a particular set of intersubjective meanings. Practices ,can therefore be said to carry with them sets of meanings. If we
investigate state action in terms of practices, we can ask questions about the constitutive intersubjective meanings, about
the world these practices make through reproducing meaning. As Roxanne Doty has argued, "Policy makers ... function
within a discursive space that imposes meanings on their world and thus creates reality. "21 At this point I reconnect to the argument
with which this chapter began, because the reality that is created in this discursive space involves the identification of the objects of action, the actors, and
the interests that are pursued. The intersubjective understandings that constitute practices can be thought of, adapting Boulding's usage,
as images that frame a particular reality. This framing is fundamentally discursive; it is necessarily _tied to the language
through which the frame is expressed. A problem-for example, that of the proliferation of weapons-is not presented to
policymakers fully formed. Weapons proliferation as a problem does not slowly dawn on states but rather is constituted by
those states in their practices. What is more, this practically constituted image of a security problem shapes the interests states
have at stake in that problem and the forms of solutions that can be considered to resolve it . To understand how an image shapes
interest and policy, it is useful to consider the place of metaphor in shaping understanding.
ENDI 2010 46
Security K Wave 1

PROLIFERATION LINK

The construction of proliferation threats is as much about the construction of the self as the
other—engaging in anti-proliferation discourse reveals a particular understanding of ourselves
and national identity that is based on otherization
Mutimer 2000 [David, associate professor of political science at York University and Deputy Director of the Center for
International and Security Studies, The Weapons State, pg 25]

There is therefore no need to deny the materiality of bodies, or of any other object, to assert that there is nothing olltside of
discourse. Rather, we must recognize that to know an object or to act on it or in relation to it, that object must enter into
discourse. Arguing from a rather different position from that of Campbell or Butler, George Lakoff comes to remarkably similar conclusions in his more
recent work: Categorization is not a matter to be taken lightly. There is nothing more basic than categorization to our thought, perception, action and
speech. Every time we see something as a kind of thing, for example, a tree, we are categorizing. Whenever we reason about kinds of things-chairs,
nations, illnesses, emotions, any kind of thing at all-we are employing categories. Whenever we intentionally perform any kind of action, say something as
mundane as writing with a pencil, hammering with a hammer, or ironing clothes, we are using categories.... Without the ability to categorize, we could not
function at all, either in the physical world or in our social and intellectual lives. 36 It is through this act of categorization, or naming, that an
object is constituted as an object for the purposes of engagement . How we act toward an object depends on what kind of object it is. How
we act in such a relationship also depends on what kind of "we" we are. That is, our identity is crucial to understanding that engagement.
The way in which other discussants will engage with the prime ministerial terrorist will vary just as much by how each
identifies herself as by which epithet is used to characterize the other. It is important to recognize, however, that identity is
also the result of categorization, of grouping those "like" as self and those "different" as other. If we want to understand a
particular form of engagement- for example, international engagement with weapons proliferation- we need to look at the
way the objects and identities of those engaged have been constructed : What kind of thing is weapons proliferation, and what is it not?
Who is involved in the proliferation agenda, and of what kind are they? How are the various elements of the proliferation agenda
referred to, and therefore into what discursive contexts are they set? These are questi~ns I address in the remaining chapters of this book.

Anti-proliferation images construct frames that are the conditions of possibility for the
construction of and response to threats
Mutimer 2000 [David, associate professor of political science at York University and Deputy Director of the Center for
International and Security Studies, The Weapons State, pg 25-26]

This chapter's primary claim is that threats


or policy problems do not dawn on states, as Charles Krauthammer would have us believe
the problem of proliferation dawned on the West. I argue, by contrast, that these problems are produced through acts of
interpretation and, what is more, that the states or other actors that threaten or are threatened are also produced in these same
interpretive acts. To make use of these claims to examine the contemporary concern with weapons proliferation, I argue that social life is
framed in terms of a particular image. That image identifies the objects of social action and the identities of the relevant
actors as objects and subjects of a particular kind. Only in terms of this image can policymakers or analysts know an
international policy problem, and therefore only in terms of this image can action be taken . Therefore, the actions being
taken by states and others in response to the problem of weapons proliferation are founded on an image that has constructed
that problem in the first place. This means, in turn, that practices can be seen to instantiate the images that enable them and
thereby become a central object of study in an analysis such as this, which seeks to reveal the images constitutive of
international life. The images that frame policy problems and thereby produce those problems, the actors and practices of security, draw on discursive
resources to tie the things imagined within the frame to other discursive frames-linking that which is framed to other things we understand in particular
ways. Images therefore tie discourses together; by creating certain links and not others and by creating these links in particular ways, metaphors highlight,
downplay, and hide other images that are operative in any given area of social life. The concepts I have developed in this chapter form the analytic basis
for the rest of the book. I will show the emergence of a relatively coherent discourse centered around the image of "weapons proliferation." The coherence
is revealed across a series of formerly distinct issue areas-the spread of nuclear technology, chemical and biological anus, antipersonnel land mines, major
conventional weapons systems, and nuclear weapons testing, for example. In detailing this image I reveal the resources from which it is built and thus the
discourses it highlights, downplays, and hides. An important means of revealing what is hidden by the emergent proliferation discourse is to tease out the
alternatives that were immanently possible but that were not tried. Counterfactual arguments are always difficult to make, but I hope to show where the
possibilities lay for such alternatives, how they were possible, and how they were made impossible by the proliferation discourse.'7 By detailing what was
highlighted in the proliferation discourse, as well as what was hidden and downplayed, my overarching goal is to show the effects of the proliferation
image. Throughout, it is important to recognize the political nature of the image-making process. As Paul Chilton noted: "Concepts do not exist as
determinate essences, but are produced and contested. The formulation and reformulation of metaphors is a crucial part of the production and contestation
of mental models. This process can occur because image schemas have an inherent 'logic' which provides potential metaphorical entailments that speakers
mayor may not specify."38 The entailments of images can therefore provide resources for the political contestation of
international practices. I intend to show not only the entailments of the "proliferation" image, which are productive of the practices of proliferation
control, but also to draw attention to these discursive re"ourc'" and to the possibilities for contestation.
ENDI 2010 47
Security K Wave 1

PROLIFERATION LINK - STATE

State discourses about proliferation construct it as a threat


Mutimer 2000 [David, associate professor of political science at York University and Deputy Director of the Center for
International and Security Studies, The Weapons State, pg 6]

Following the Security Council's call to arms-or, perhaps more appropriate, its call away from arms-considerable action seems to have beeu taken to
prevent the spread of technology related to research for and production of arms. Proliferation has become a primary security concern of the post-Cold War
era. Proliferation is a threat, however, because it has been stated authoritatively to be a threat. More than that, weapons
proliferation has itself been constructed as a problem of contemporary security . As with the creation of a threat by its being
stated as a threat by the United Nations and other authoritative speakers, proliferation is a construction that begins with
what is said and continues with how that saying is embedded in the actions it makes possible . Because of the
privileged position of the state in international relations, what is stated by the state-or by states when they gather as something like the
Security Council-often has the greatest importance. The weapon state and the state of weaponry, as well as the weapons and
states themselves, are made possible by being stated by the state.
ENDI 2010 48
Security K Wave 1

PROLIFERATION LINK – WEAPON LABEL

Labeling nuclear devices as weapons makes them appear to only be a particular kind of weapon
used in strategic circumstances
Mutimer 2000 [David, associate professor of political science at York University and Deputy Director of the Center for
International and Security Studies, The Weapons State, pg 31-32]

The military developed atomic weapons in the context of a war and with the expressed intention of using them as part of a campaign of strategic
bombardment. Although the remarkable destructive power demonstrated over Japan led to calls for their elimination or for
tight international control, atomic weapons were never removed from the preserve of the militaries in states that developed
them. They were therefore thought of, in the first instance, as weapons .2 Although this may appear little more than a
statement of fact, it is a categorization or a representation, and one with considerable power. By labeling nuclear explosives as
"weapons," those with the initial access to them categorized them as devices of a particular kind.' They understood the
devices in terms of discourses that constituted the range of technologies with which people had killed each other in
certain circumstances. This latter point is important, because not all technologies of killing were discursively constructed as
weapons: exceptions include poisons administered through food, for example, or the hangman's noose. Jonathan Schell appeared
to recognize the contingency of these devices' labeling at the beginning of The Fate of the Earth: "These bombs were built as 'weapons' for 'war,' but their
significance greatly transcends war and all its causes and outcomes. They grew out of history yet threaten to end history. They were made by men, yet they
threaten to annihilate man. They are a pit into which the whole world can fall-a nemesis of all human intentions, actions and hopes."4 The images Schell
suggests, a pit and a nemesis, would entail a very different set of containing practices from those entailed by the image of weapons for war. Both images
suggest dangers that must simply be avoided, not managed so they can somehow be lived with and certainly not put to the kind of use that carries the high
value placed on providers of security or peace. There were possibilities for framings of nuclear weapons less extreme than Schell's but that would still
have denied any notion of their being weapons for use in a practice as instrumental as war. Atomic explosives have been conceived, for example, as
doomsday devices-human creations that allow us to exercise the power of God in ending creation-an image that would still have entailed potential use but
not a military-strategic use. Having been imagined as weapons, however, atomic explosives were further constituted as particular
weapons to fulfill particular functions through their articulation in strategy.

This characterization of nuclear devices as weapons leads to conventionalization and downplays


their massive power
Mutimer 2000 [David, associate professor of political science at York University and Deputy Director of the Center for
International and Security Studies, The Weapons State, pg 33]

A considerable contradiction is implied in the dual framing of nuclear explosives as weapons and as instruments
exclusively for deterrence. This contradiction is seen in a number of ways throughout the strategy debate, most notably in
the odd position of nuclear weapons being accepted as weapons whose only use is in their nonuse. Williams cites a wonderful
example of the result of the contradictions of deterrence. "This is a conclusion that even Hoag himself seems to find difficulty in believing, to say nothing
of justifying: 'Such a policy,' he argues, 'suffers from the political defect, but analytic virtue, of explicit bizarreness.' Explicit bizarreness would seem to be
a peculiar attribute to cons~der as an 'analytic virtue.'''9 the contradiction between weapons and deterrence with regard to nuclear
weapons has also led Robert Jervis to make one of the most trenchant critiques of nuclear strategy: that it suffers from what he
calls "conventionalization." For Jervis, thinking of nuclear weapons in conventional terms;--as weapons-leads to dangerous
strategies because it ignores the lessons of the nuclear revolution, which produced the possibility of strategies of mutual
deterrence in the first place. lO These contradictions, the conventionalization critique in particular, point directly to the
constructed nature of the weapon framing of nuclear weapons. The central characteristic of nuclear explosives--that they are
extremely powerful explosives seems to deny their coherent consideration as weapons at all, yet this is how they are
primarily conceived.
ENDI 2010 49
Security K Wave 1

PROLIFERATION LINK –STABILITY

The emphasis on strategic stability in the face of proliferation reproduces the Cold war us-them
dichotomy in international relations
Mutimer 2000 [David, associate professor of political science at York University and Deputy Director of the Center for
International and Security Studies, The Weapons State, pg 39]

Arms race instabilities resulted from the acquisition of weapons seen to trigger a new round of arms building by the other
side. The U.S. "Star Wars" proposal was arms race destabilizing, for it tended to prompt both an increase in the Soviet arsenal to overwhelm any U.S.
defense and the development of countenneasures. Crisis instabilities were even more dangerous, as they were changes that increased the'likelihood of
nuclear war in the event of a crisis-that is, changes that could lead to the dangerous tumble into war and then to nuclear war. Multiple independently
targetable re-entry vehicle (MIRV) technology, for example, was criticized for introducing crisis instabilities. On the ground, a missile with 10 MIRVed
warheads- the US MX, for example-could be destroyed by a single incoming warhead. Once lannched, however, that single missile could strike 10 targets-
Soviet missile silos, for instance. In a crisis, the argument ran, a greater incentive would exist to launch a MIRVed missile than one with only one warhead
because of this unfavorable ratio. The practice of "arms control" relied heavily on this understanding of balance and the need to
maintain stability. Arms control agreements aimed to create stable balances between the arsenals of the two poles and to establish
mechanisms that would prevent crises from destabilizing the central balance of power. It is no exaggeration, I think, to maintain that without an
understanding of balances privileging stability, the practice of arms control would have been impossible. Their centrality,
both to the theory of deterrence and to the practice of arms control, made the notions of balance and stability important
resources in the reimagination of security following the end of the Cold War. This use of balance and stability has two important
implications. First, relying on these resources when framing other security problems tends to reproduce the dyadic
structure of the Cold War and of balance-of-power theory in international relations. Second, notions of balance and stability
were crucial in differentiating "arms control" as a practice from those practices warranted by the ~'disannament" image.

The affirmative’s representation of a stable world free from nuclear war is rooted in strategic
arms control discourse
Mutimer 2000 [David, associate professor of political science at York University and Deputy Director of the Center for
International and Security Studies, The Weapons State, pg 37]

Finally, underlying all of the practice of arms control is the notiou of strategic stability, another derivative of deterrence
theory. Again. mutual assured destruction ensures that one of the states will avoid launching a nuclear war if it is confronted
with a simple choice: nuclear war or no nuclear war. Wars do not always begin so neatly, however, and some form of
protection from the "back door" into war is needed. The hot line agreement addressed one such back-door route by attempting to prevent a
misunderstanding from resulting in a war neither side wanted. It was also considered possible, however, that a crisis would escalate into a nuclear war-
particularly because nuclear deterrence was supposed to deter all sorts of things other than nuclear war. To prevent a tumble into war, which
could then escalate into a nuclear war, strategic discourse counseled maintaining strategic stability.27 A condition is stable if
changes to that condition are unlikely to cause it to topple into war . The notion of stability, and the consequent need to
maintain balances, is a crucial element of the "arms control" framing, and has important implications for the "proliferation"
image of the post-Cold War era. Although arms control depended very specifically on the understandings of stability in deterrence theory, those
understandings have much deeper roots in international relations.28
ENDI 2010 50
Security K Wave 1

PROLIFERATION LINK – ‗WILDFIRE‘ / CANCER METAPHOR

Metaphors of proliferation as spreading rapidly falsely attribute it to a single source, failing to


understand the dynamics that drive the spread of weapons technology
Mutimer 2000 [David, associate professor of political science at York University and Deputy Director of the Center for
International and Security Studies, The Weapons State, pg 60]

The first implication is that somethingimagined in terms of "proliferation" is seen to grow or multiply from a single source.
Although animal reproduction involves two individuals, the father is quickly forgotten, and it is the mother who is
proliferous. The budding of cells, which gives rise to the proliferation of some plants and, of course, cancers, begins with a
single, or source, cell and spreads from there -in the case of a cancer, both to produce a single tumor and to create a number of separate tumors
throughout the host body. Similarly, the problem of weapons proliferation is one of a source or sources proliferating, that is,
reproducing by supplying the necessary technology to a new site of technological application . This form of imagining
highlights the transmission process from source to recipient. Hence, the dominant response to nuclear proliferation has been
the creation of supplier groups-the Zangger Committee and the NSG- that seek to control the spread of nuclear technology. In other
words, to paraphrase Murray and Hnnt, they attempt to provide the checks and balances that normally ensure orderly transfer and
prevent the spread of nuclear technology resulting in the "cancer" of a prolific number of nuclear weapons .
ENDI 2010 51
Security K Wave 1

PROLIFERATION LINK – WEAPONS SPREAD

The notion of weapons spread perpetuates the notion that the weapons themselves spread and
takes responsible agents out of the analysis of international relations
Mutimer 2000 [David, associate professor of political science at York University and Deputy Director of the Center for
International and Security Studies, The Weapons State, pg 61]

The second implication of the proliferation metaphor for the problem of nuclear weapons spread is an extreme technological
determinism. Animal reproduction is an internally driven phenomenon, and so the metaphor of proliferation applied to the
development of nnclear technology highlights the antonomy in the growth of that technology and its problematic weapons
variant. It is worth recalling Frank Barnaby's words: ~'A country with a nuclear power program will inevitably acquire the techuical knowl- . edge and
expertise, and will accumulate the fissile material necessary to produce nuclear weapons."19 In fact, the text from which this quotation is drawn presents
an interesting example of the autonomy of the proliferation metaphor. The book is entitled How Nuclear Weapons Spread: NuclearWeapon Proliferation
in the 1990s. Notice that the weapons themselves spread; they are not spread by some form of external agent-say, a human
being or a political institution. Under most circumstances such a title would be unnoticed, for the implications are so deeply
ingrained in our conceptnal system that they are not recognized as metaphorical . This image, by highlighting the
technological and autonomous aspects of a process of spread, downplays or even hides important aspects of the
relationship of nuclear weapons to internationaJ security. To begin with, the image hides the fact that nuclear weapons do
not spread but are spread-and, in fact, are spread largely by the Western states. Second, the image downplays-to the point of
hiding-any of the political, social, economic, and structural factors that tend to drive states and other actors both to supply
and to acquire nuclear weapons. Finally, the image downplays the politics of security and threat, naturalizing the security
dilemma to the point that it is considered an automatic dynamic. The image of "proliferation" thus privileges a
technical, apolitical policy by casting the problem as a technical one. The NPT controls and safeguards the movement of the
technology of nuclear energy. The supporting supplier gronps jointly impose controls on the supply-that is, the outward flow-of this same technology. The
goal in both cases is to stem or at least slow the outward movement of material and its attendant techniques.

The notion of spread creates a division between allowable and unallowable weapons use and
development, justifying western countries’ claim to legitimate violence and controlling all others
Mutimer 2000 [David, associate professor of political science at York University and Deputy Director of the Center for
International and Security Studies, The Weapons State, pg 61]

A "proliferation" image produces a particular kind of object. It imagines a technology that reproduces rationally and
autonomously, moving outward from an identifiable origin by relentlessly multiplying. The image imagineses this
technology as essentially benign but with the possibility of excess production is natural, expected, and even
desirable, but prolific reproduction is dangerous. To permit the benign spread of technology while preventing the
dangerous conclusion to that spread, external controls are required. Because the object of "proliferation" is imagined in this
fashion, the forms of control that can be applied are constrained . Put another way, the particular imagination of the object of
"proliferation" enables a specific series of control practices. The reverse is also true: creating given practices will construct the
object of those practices in particular -ways. The result is a neatly closed circle it is simple to reify-we face this particular
problem with these practices; these practices are employed, so we are facing this problem. Read in either direction, the contingent
becomes seen as the natural. What has happened since the late 1980s, particularly following the war in the Gulf, has been
the reimagining of all forms of military technology in terms of the "proliferation" image and the embedding of that image in
a series of control practices. Alternatively, a series of control practices has been established around the range of military
technologies, which has constituted the object of those practices as a "proliferation" problem.
ENDI 2010 52
Security K Wave 1

PROLIFERATION LINK – PEACEFUL/ MILITARY DISTINCTION

The distinction between peaceful and unauthorized use of nuclear power is rooted in the desire
to control and monitor the use of technology by others while justifying military use and
development—the framing of proliferation as a problem in this context is the key representation
Mutimer 2000 [David, associate professor of political science at York University and Deputy Director of the Center for
International and Security Studies, The Weapons State, pg 64-65]

The particular practices of nuclear proliferation control were made possible by the understanding of the problem provided
by the "proliferation" image. By imagining the problem in terms of a largely autonomous, technologically driven process
that would inevitably result in the dissemination of nuclear weapons, "proliferation" privileged a set of practices that
focused on the monitoring and control of the constituent technologies. As a doctor would monitor the growth of cells for
signs of cancerous proliferation and would intervene to regulate healthy growth and, if possible, prevent the cancerous
spread, so the IAEA and nuclear suppliers monitor and intervene to allow for the spread of nuclear energy for peaceful
purposes while attempting to prevent harmful nuclear proliferation. The result is a set of practices that begins with an international
nondissemination norm, together with permission for (even the promotion of) healthy trade in the constituent technology,
underpinned by international monitoring and technology controls by suppliers. These practices, in turn, reproduce the underlying
understandings of the "proliferation" image. This is important for my purposes, for as these practices are transported to other
technologies of concern they tend to constitute their object in the same way nuclear proliferation practices constitute theirs.
As questions of military technologies have come to be reimagined in terms of "proliferation," the practices used to
control nuclear proliferation have been re-created in these other areas. In the process, the images by which the various
military technologies had been or could be imagined have been downplayed . With the interpretation of reported chemical weapons use
by Iraq in its war with Iran as indicative of the weakening of the taboo against CW use, the issue of chemical weapons began to be imagined in terms of
the "proliferation" image. By 1993 this had resulted in the conclusion of the Chemical Weapons Convention, but the immediate response was the 1987
creation of the Australia Group, formed as a supplier group for chemical weapons technology. The group sought to coordinate its members' export control
policies on technology and material related to the creation of chemical weapons in exactly the same way the NSG and the Zangger Committee aim to
control the movement of nuclear technology. The formation of the Australia Group depended on the prior reframing of chemical weapons in terms of
"proliferation." The "taboo" frame in which CW had been imagined constituted a problem of use, not of technological spread. Considering that chemical
weapons use extends at least to the nineteenth century, it might be supposed that the technology was already well spread. Efforts at international
control had therefore involved codifying and strengthening the nonuse norm through legal restrictions on the preparation for
and execution of chemical warfare. In such a context a supplier control group makes little sense. Only when the problem is reimagined
as a "proliferation" problem, and hence a problem of the spread of constituent technologies, would states turn to forming a
supplier group.
ENDI 2010 53
Security K Wave 1

PROLIFERATION LINK – ROGUE STATES/ LOOSE NUKES

The representation of potential nuclear weapons states as rogues incapable of developing their
own weapons is based on Western-centric constructions of subject positions-it is racist and
infantilizes these states while ignoring the West’s complicity in the same practices
Mutimer 2000 [David, associate professor of political science at York University and Deputy Director of the Center for
International and Security Studies, The Weapons State, pg 88-89]

At its 1992 Summit, the UN Security Council determined that the pn'lif'eration of weapons was a threat to international peace and security. There are two
indications in the council's communique of what makes proliferation a threat: first, that the accumulation of (conventional) anns should not be excessive
and, second, that the spread of arms should not disrupt regional or global stability.26 By characterizing the threatening effects of
proliferation those that disrupt stability, the proliferation image constitutes the space for the sort of state behavior that will
cause serious concern for the guardians of international probity. It begins, in other words, to produce important subject
positions within the "proliferation" image. The work of Michel Foucault increasingly defines the way in which we think about
the constitution of the subject in modern society. Through his work, Foucault examines ways in which discourses of normality establishes the
confines in which the subject may operate. Normal behavior is defined largely through identification of the fonns of abnormality
that constitute its limits, which, in turn, are rigidly policed . The proliferation discourse defines normality in terms of
regional and global stability, and hence abnormality (or behavior that causes serious concern) in terms of threats to or disruptions of
that stability. It is a constitution of the normal international subject policed by the UN Security Council and by the
advanced industrial states through their export control regimes. One notable feature of Foucault's accounts of the constitution of the
modern subject is the complicity of various academic disciplines in defining the contours of the normal.27 The idea of stability as the normal condition in
international life also reveals academic complicity, having been produced and reproduced by the discipline of international relations. As I argue in Chapter
3, a particular characterization of balance has been defined in the practice of international relations scholarship largely with
reference to the relationship among the Soviet Union, the United States, and the world order during the Cold War. This
understanding of balance, particularly of balance of power, in turn, gives rise to stability as the normal condition of
international life. Balances need to be maintained; instabilities upset these balances and produce disorder. By extension, those states that act to
upset stable balances can be labeled in some way deviant. I argue in Chapter 3 that the most important implication of framing
security problems in terms of a balance that needs to remain stable is that it highlights dyadic relationships. During the Cold
War the dyadic understandings of balance were reasonably appropriate to the superpower confrontation, as two roughly equivalent superpowers were
anchoring two roughly equivalent alliances. Even then, however, the image downplays and hides those outside the central balance,
rendering non-European states and regions either as invisible or as mere appendages to the superpower confrontation . To
imagine third parties as autonomous would be to introduce problematic third- and higher-order masses into the
metaphorical balance. The regional security systems that today are of greatest "proliferation" concern to those, mainly in the
North and West, who use the image, transgressions thus, for example, the definition of rogue as rascal. Similarly, the outlaw is a
common figure in U.S. romantic Western literature. Outlaws roamed the frontiers of the central United States, at once dangerous and admired for the
rugged individualism they portrayed. Little of this romanticism seems to remain in the use of rogues in official discourse, however. U.S. Secretary of State
Warren Christopher did not seem to admire the rugged individualism of potential rogues , for instance, when he told the Senate
Foreign Relations Committee that "nuclear weapons give rogue states disproportionate power, destabilize entire regions,
and threaten human and environmental disasters. They can turn local conflicts into serious threats to our security. In this era,
weapons of mass destruction more readily available and there are fewer inhibitions on their use."39 Nevertheless, the use of rogue carries with it
marked condescension. Rogues are, as often as not, young men, indeed even little boys. who acting naughtily-in the former
case often in a sexual manner. One of many ironies that emerge in stories of proliferation is that at the same time the primary
international rogue, Iraq, was under intense U.S. pressure because of its refusal to allow UNSCOM unfettered access to its
presidential palaces, the U.S. president was being labeled a rogue for reports he had perhaps allowed too much access to
presidential parts. "Some the President's intimates note his remarkable ability to compartmentalize his life: The policy wonk who genuinely admires
his wife resides in space; the rogue who risks political standing through personal indiscretion occupies another. "40 Put another way, the mature adult
resides on the one side and the rather indiscreet little boy on the other. The use of rogue to label states behaving in ways
deemed unacceptable identifies those states as immature compared with the mature states doing the labeling-foremost
aruong these the United States. Such an entailment fits well with the practices established for proliferation control. The
mature elders gather together to determine which states are sufficiently responsible to be trusted with advalaced technologies
and military equipment-indeed, the practice smacks of Star Trek's Prime Directive. This notion of maturity is then reflected in academic
commentary on temporary security, as Charles Krauthammer's characterization of weapon state threat illustrates: "relatively
small, peripheral and backward states will be able to emerge rapidly as threats not only to regional, but world, security."41
Similarly, a repeated concern in the literature has that new nuclear states would lack the maturity to control their weapons
adequately, unlike the old nuclear states.4'
ENDI 2010 54
Security K Wave 1

PROLIFERATION LINK – ROGUES/ MONITORING

Proliferation discourse draws lines between acceptable and unacceptable proliferators, policing
difference through monitoring and sanctioning supposedly rogue states
Mutimer 2000 [David, associate professor of political science at York University and Deputy Director of the Center for
International and Security Studies, The Weapons State, pg 97]

The proliferation image creates two clear lines of difference. The first marks the distinction between those who can be
trusted to make the must follow the rules signalled by inclusion in the ranks of suppliers-and those who follow the rules-
the recipients. The second line marks those who do follow the rules from those who refuse-the rogues from the herd, the out
from the law-abiding. This second line marks the emergence of an enemy in this discourse of military security, for it is
rogue behavior that a threat, that causes concern to those who make the rules. Thus the recipients' are accepted as part of the
community of the law-abiding and have access to prized technology the suppliers can provide . The recipients, however, are
also potential rogues. Their behavior must be policed through export control and compliance monitoring to ensure that they
conform to the rules and do not become rogues. This policing gives in to the temptation of othering difference Connolly discussed.
Not only are those not included in the supplier groups to be marked as different, but they are to be labeled as potential
enemies and sanctioned as such. The proliferation image constructs states in the Third World as outsiders. Even if they do
not become rogues, they are not permitted inside the privileged Northern club; if they do behave in ways that cause
concern to the privileged, they are labeled enemy and heavily sanctioned. Not surprisingly not all Third World states are entirely happy
with the "proliferation" construction. Iran, for example, finds itself abiding by the rules of NPT membership, rules that are supposed to guarantee its access
to nuclear technology for peaceful purposes. Nevertheless, its behavior-in this case the domestic politics of government---causes concern among members
of the supplier groups, and so it is sanctioned. India has established itself as the preeminent critic of the proliferation discourse in the Third World. India
does not accept the problem as it has been constructed by the insiders and it does not accept the practices to which the construction has given rise. In
Chapter 6 I examine the alternative frarnings produced in this resistance and elsewhere to see the possible objects and identities hidden by the
"proliferation" image that could serve as a basis for political opposition to that image.
ENDI 2010 55
Security K Wave 1

PROLIFERATION DISCOURSE TURNS CASE

The discourse of proliferation reproduces and reinforces the discursive construction of


sovereignty which in turn incentivizes and precipitates the spread of nuclear technologies
Mutimer 2000 [David, associate professor of political science at York University and Deputy Director of the Center for
International and Security Studies, The Weapons State, pg 140]

This connection between sovereignty-statehood and weaponry raises the greatest irony of the proliferation agenda. The
spread of military
technology, which is of such concern to states of the West, is driven largely by iterests found in the representations of state
and sovereignty that circulate throughout the contemporary international system. That circulation takes place through the
practices of states that reproduce the discourses out of which those representations flow. In other words, when the United
Kingdon: "ring-fences" the Trident program in its defense review, it is reproducing the nuclear arsenal as a marker of status
in the international system. Similarly, when the United States revises its military posture in the aftermath of the Cold War on the
basis of a need to maintain a fully functional, high-technology military capable of fighting two or more wars
simultaneously, it strongly reinforces the relationship among statehood, status, and that particular form of military
organization and equipment. To produce that military posture in the post--Cold War world, the United States played a
central role in building proliferation as a primary international security threat and the rogue state as its central villain. In other
words, the very process of developing and responding to a "proliferation" agenda in the past few years has reproduced and
reinforced the discursive construction of what it means to be a sovereign state in the contemporary world, which, in
turn, is central to the spread of advanced weaponry and related military technologies.
ENDI 2010 56
Security K Wave 1

ECONOMIC COMPETITIVENESS LINK

Discourses of economic competitiveness are based on the securitization of financial loss and the
construction of new economic enemies
Lipschutz 1998 [Ronnie, prof of politics at UC Santa Cruz, On Security, ed. Ronnie Lipschutz,
http://www.ciaonet.org/book/lipschutz/index.html]

The ways in which the framing of threats is influenced by a changing global economy is seen nowhere more clearly than in
recent debates over competitiveness and "economic security." What does it mean to be competitive? Is a national industrial
policy consistent with global economic liberalization? How is the security component of this issue socially constructed ?
Beverly Crawford (Chapter 6: "Hawks, Doves, but no Owls: The New Security Dilemma Under International Economic Interdependence") shows how
strategic economic interdependence--a consequence of the growing liberalization of the global economic system, the increasing availability
of advanced technologies through commercial markets, and the ever-increasing velocity of the product cycle--undermines the ability
of states to control those technologies that, it is often argued, are critical to economic strength and military might . Not only
can others acquire these technologies, they might also seek to restrict access to them. Both contingencies could be
threatening. (Note, however, that by and large the only such restrictions that have been imposed in recent years have all come at the behest of the
United States, which is most fearful of its supposed vulnerability in this respect.) What, then, is the solution to this "new security dilemma," as Crawford
has stylized it? According to Crawford, state decisionmakers can respond in three ways. First, they can try to restore state autonomy through self-reliance
although, in doing so, they are likely to undermine state strength via reduced competitiveness. Second, they can try to restrict technology transfer to
potential enemies, or the trading partners of potential enemies, although this begins to include pretty much everybody. It also threatens to limit the market
shares of those corporations that produce the most innovative technologies. Finally, they can enter into co-production projects or encourage strategic
alliances among firms. The former approach may slow down technological development; the latter places control in the hands of actors who are driven by
market, and not military, forces. They are, therefore, potentially unreliable. All else being equal, in all three cases, the state appears to be a net
loser where its security is concerned. But this does not prevent the state from trying to gain. How can a state generate the
conditions for legitimating various forms of intervention into this process? Clearly, it is not enough to invoke the mantra of
"competitiveness"; competition with someone is also critical. In Europe, notwithstanding budgetary stringencies, state sponsorship of
cutting-edge technological R&D retains a certain, albeit declining, legitimacy; in the United States, absent a persuasive threat, this is much less the case
(although the discourse of the Clinton Administration suggests that such ideological restraints could be broken). Thus, it is the hyperrealism of Clyde
Prestowitz, Karel Van Wolferen, and Michael Crichton, imagining a Japan resurgent and bent anew on (non-)Pacific conquest, that
provides the cultural materials for new economic policies. Can new industrialized enemies be conjured into existence so as
to justify new cold wars and the remobilization of capital, under state direction, that must follow ? Or has the world changed too
much for this to happen again?
ENDI 2010 57
Security K Wave 1

HUMANITARIAN ASSISTANCE LINK

Developmental assistance and humanitarian aid are inextricably linked to securitization – the
liberal moralism at the root of the action fuels the threats they seek to solve, leading to a
securitization of society itself
Dillon and Reid 2000 [Michael, Professor of Political Science at Lancaster and internationally renowned author, and
Julian, lecturer on international relations and professor of political Science at King‘s College in London; from Alternatives,
Volume 25, Issue 1: Global Governance, Liberal Peace, and Complex Emergency]

Pursued as a deliberate policy of comprehensive social transformation, and of power projection, development becomes
allied in novel ways via global liberal governance with geopolitical military and economic institutions and interests. The
transformation is therefore to be effected according to the current efficiency and performance criteria of good governance--
economically and politically--set by the varied institutions of global liberal peace. In the proces s, sovereignty, as the traditional
principle of political formation whose science is law, is being supplemented by a network-based account of social organization whose
principle of formation is "emergence" and whose science increasingly is that of complex adaptive systems.[ 5] These ensure
that the political issue posed by Stiglitz rarely progresses beyond an afterthought. This incendiary brew is currently also fueled by a
resurgent liberal moralism. That moralism generates its own peculiar forms of liberal hypocrisy. These include: the calling for
intervention by the international community against Indonesian actions in East Timor while liberal states furnished Indonesian armed forces with the very
means of carrying out those actions; and seeking to proscribe child soldiers while failing to address the global arms economy that furnishes the children
with their weapons. The vexed relation between liberalism and capitalism is also at issue once more since clearly, too, the
globalization of markets and of capitalism is intimately involved in the "complex emergencies" that global liberal
governance seeks to police. While the formula complex emergency arose in the general context of the of the politics of
bipolarity and the advent of liberal peace, it did so in the specific context of the dramatic weakening of state structures and
the exaggerated ideals of sovereign statheood, together with the advent of intractable development problems and civil conflicts as well as
adaptation to the structural-adjustment programs of the 1980s and 1990s, which impacted on the fringes of liberal internationalism. The conflation of
established distinctions between civil and military as well as between the humanitarian and the geopolitical that has taken place as a result has proved
confusing and disturbing to all participants in global governance and liberal peace.

Liberal humanitarian efforts have at their core the desire for security and complicity with the
system that has put it in place
Dillon and Reid 2000 [Michael, Professor of Political Science at Lancaster and internationally renowned author, and
Julian, lecturer on international relations and professor of political Science at King‘s College in London; from Alternatives,
Volume 25, Issue 1: Global Governance, Liberal Peace, and Complex Emergency]

Liberal humanitarians have, for example, become politicized, geopolitically ambitious, and sometimes warlike in pursuit of
liberal peace. They have also found themselves in alliance with the institutions of international political economy and
governance as well as with branches of the military. Increasingly, the policies and practices of "political conditionality" are
also suborning them. Deals and contracts have inevitably to be struck with local political groupings in order that aid might be delivered to the needful
in areas of political turbulence. Political conditionality is, however, more than this local pragmatism. At a policy level, it refers to
the ways in which government and international-aid agencies are increasingly making the delivery of aid conditional on the
recipients meeting the good governance criteria that global liberal politics specifies for them. At a local level, it means
calibrating the delivery of aid to effect the internal politics and maneuvering of warring groups so that political settlements
sought by international coalitions--such as the one, for example, that currently manages Bosnia--might be secured. In order both for policy-
level practices and local political arm twisting to work, governments and international organizations must secure the compliance of the large number of
nongovernmental organizations that populate the zones of "complex emergency." These of course provide many significant conduits
for aid. The vast majority of them are, however, effectively the subcontractors of governmental organizations and of
international agencies. Their prized independence is problematic, and their classification as nongovernmental is sometimes
equally so. Effecting political conditionality requires their participation. To the extent, however, that they comply--and their very capacity to
resource themselves and operate may be intimately dependent upon their good standing with these governmental and international agencies--their
"impartiality" and humanitarian ideals are compromised. In such circumstances, they run the deadly risk of becoming
identified as active participants in conflicts rather than impartial ministers to the needy and afflicted that are created by
them.
ENDI 2010 58
Security K Wave 1

ENVIRONMENTAL SECURITY LINK

Securitization of the environment perpetuates the notion of the nation-state and perpetuates
geopolitical borders
Rasa Ostrauskaite, Third Secretary of Lithuania to the United Nations, December, 2001
Rubikon Environmental Security as an Ambiguous Symbol: Can We Securitize the Environment?,
http://venus.ci.uw.edu.pl/~rubikon/forum/rasa2.htm

In the previous section we have seen that transboundary nature of environmental threats makes it difficult to categorize them. The task
becomes even more complicated when it comes to labeling them ―for securitization‖, especially if we try to do so in the framework of
national security. As societies come to recognize the planetary scale of destruction of the environment, we increasingly realize
that the traditional forms of national sovereignty are challenged by the realities of ecological interdependence . At the same
time, we are not yet ready to sideline the principle of national sovereignty. Despite some calls for a complete rejection of
sovereignty[29] or warnings against the privileging of national security in the face of global problems,[30] we are not ready to give up
conventional political arrangements of nation states. The present-day political map of the world is a map of independent
states. Notwithstanding the fact that they are merely local normative arrangements for promoting the good of humankind in
the area of the world where they are located, nobody wants to challenge the principle of sovereignty - the fundamental
principle on which the rest of international relations is constructed. This view has been so eloquently expressed by R.B.J. Walker,
that despite its length, it is worth quoting:…[E]ven if we admit that we are all now participating in common global structures, that we are all rendered
increasingly vulnerable to processes that are planetary in scale, and that our prost parochial activities are shaped by forces that encompass the world and
not just particular states, it is far from clear what such an admission implies for the way we organize politically. The state is a political category in
a way that the world, or the globe, or the planet, or humanity is not. The security of states is something we can comprehend
in political terms in a way that, at the moment, world security cannot be understood .[31] In this context, security discourse
remains entangled with state politics, and so long as conventional understanding of security prevails, states will remain the
main providers of security. Thus, it seems reasonable to be conservative about national security as the security of the state, since, as Waever rightly
points out, ―neither individual security nor international security exists‖ [32].

Securitizing the environment justifies the state protecting it through military means
Waever 1998 [Ole, professor of International Relations at the Department of Political Science, University of Copenhagen,
―Securitization and Desecuritization,‖ On Security, ed. Ronnie Lipschutz,
http://www.ciaonet.org/book/lipschutz/index.html]

Still, in the final analysis, is it all to the good that problems such as environmental degradation be addressed in terms of security?
After all, in spite of all the changes of the last few years, security, as with any other concept, carries with it a history and a set of
connotations that it cannot escape. At the heart of the concept we still find something to do with defense and the state. As a
result, addressing an issue in security terms still evokes an image of threat-defense, allocating to the state an important role
in addressing it. This is not always an improvement. Why not turn this procedure upside down? In place of accepting
implicitly the meaning of "security" as given and then attempting to broaden its coverage, why not try instead to put a mark
on the concept itself , by entering into and through its core? This means changing the tradition by taking it seriously rather than criticizing it from the
outside. 2 I begin by considering security as a concept and a word. Next, I discuss security as a speech act . In the third part of the essay, I describe four
cases of securitization and de-securitization . Finally, I ask whether we might not want to use "security" as it is classically understood, after all.
ENDI 2010 59
Security K Wave 1

ENVIRON. SECURITY T/O SOLVENCY

Securitization of the environment makes solvency impossible by focusing on the effects as


opposed to the causes of environmental collapse
Rasa Ostrauskaite, EU Political Advisor in the Office of the High Representative for BiH, December, 2001
Environmental Security as an Ambiguous Symbol[1]: Can We Securitize the Environment? Rubikon,
http://venus.ci.uw.edu.pl/~rubikon/forum/rasa2.htm

Having demonstrated the ambiguities of the environmental security discourse, I shall specify the link between environment and security, arguing against
this linkage. According to supporters of the environment-security linkage, environmental degradation is as severe as the military threats and thus deserves
to be lifted to the ―high politics‖; i.e. environmental degradation should be placed under the umbrella of national security. Yet tacking the security
label to environmental issues deserves more than a word of caution. First, in the environmental security discourse, whose
interests should be secured: those of the state, humanity, the future generations or the nature ? As we have seen in the previous sections,
these interests can be and usually are in conflict. It could be pointed out, however, that once a link between environmental degradation and
violent conflict is established, the answer to the question whose interest should be secured becomes self-evident. It is at these crucial junctures of conflict
that the issue of environmental degradation becomes worthy of a ―security‖ label. Nevertheless, the linkage between environmental
degradation and violent conflict could not be easily established, and even those who maintain the existence of such
linkages, albeit indirect, subtle and not always predictable, admit that environmental degradation is not very likely to cause interstate
conflicts[47]. Therefore, it is ―analytically misleading to think of environmental degradation as a national security threat,
because the traditional focus of national security has little in common with either environmental problems or solutions ‖[48].
Second, since one state‘s unilateral efforts may have little effect, if at all, states may choose to cooperate to prevent or
minimize environmental threats for which they share responsibility. To agree upon collective strategies to reduce
environmental vulnerabilities would be easier, however, if decision-makers first desecuritized environmental degradation. As
rightly pointed out by Waever, while to securitize an issue is to declare it being ―off limits‖, to desecuritize an issue is to remove it from the
realm of the politics of survival and to allow for a more open and fruitful debate on it[49]. Thus, desecuritization renders the
issue amenable to more cooperative forms of behavior. And this could be applicable to the logic of international
environmental relations among the states. Moreover, the collective approach frequently entails negotiating treaties that commit states to limit
certain activities within their jurisdiction, which, if the issue is declared to be ―off limits‖, might prove to be more difficult to achieve. The only reason to
feel tempted to keep environment ―off limits‖, however, would be the possibility to have more resources allocated from the state budget, which, unless
environment is securitized, might prove to be a complicated task. Yet the question remains whether the benefits of increased attention of environmental
issues to be gained through association with security are worth the harms caused by negative connotation and effect. It is probably accurate to say that one
of the biggest difficulties for securitization of environment is posed by the fact that causes and effects of environmental issues differ in time and space. If
one of the motives for speaking of environmental degradation as a threat to national security is rhetorical: to make people
respond to environmental threats with a sense of urgency, effects rather than causes tend to be securitized. As Buzan et al. point
out, ―in terms of politicizing causes, much is happening, but most of the threats are too distant to lead to securitization‖[50]. With the exception of already
discussed securitization of the threats posed by nuclear plants in Central and Eastern Europe, which are close in both time and space to the European
states, slow progress has been made towards addressing the causes rather than effects of environmental threats. Even climate change, which is a global
problem that requires a coordinated global response, has recently been defined as ―at least a 100-year problem,‖[51] signaling that, inter alia, asymmetries
in causes and effects might seriously impede securitization moves at the global level. Another motive for securitization of effects rather than causes is a
recognition that crises call for resolution during which the patience of society can be mobilized. Unfortunately, it is very unlikely that
permanent patterns of environmentally sound behavior could be supported for a long time, especially if requires some
personal sacrifice[52]. For this reason, it seems that environmental concerns could be better addressed if they constitute part
of ‗normal politics‘, rather than if enveloped in the national security, since the focus should be on the causes, rather than
effects.
ENDI 2010 60
Security K Wave 1

CRITICAL INEQUALITY LINK

The affirmative’s focus on inequality is narrowly conceived within the confines of democracy or
capital accumulation and anxieties about US hegemony—this reproduces the evocation of danger
at the heart of otherization
Walker 2002 [RBJ, professor of political science at the University of Victoria, ―International/Inequality,‖ International
Studies Review, Vol. 4, No. 2, International Relations and the New Inequality (Summer, 2002), pp. 7-24, jstor]

The relatively simple but often ignored point I want to make to begin with is that the
international is already constituted through the
legitimation of spe- cific forms of inequality. This implies that new forms of inequality might be understood in terms of the
categories through which the international has been constituted historically, or they might be understood as challenges to the legit-
imacy, and perhaps even the possibility, of the international. The first option permits some familiar accounts of what it means to identify and respond to
inequality in an international context, and I will not say much about them. The second option opens out a range of questions that are much more difficult
to evaluate, and I will only seek to suggest a few lines of thinking about what these questions might imply. First , we might focus on worldwide
patterns of global economic inequality, patterns that while difficult to delineate or evaluate in precise detail are clearly at odds with the claims to
formal equality in the system of modern sovereign states which constitutes the primary ground of modern political life. In this context we are likely to
engage with various traditions of international or global political economy, and thus with questions about how we are supposed to dis- tinguish both
between the international and the global and between the eco- nomic and the political. These questions usually lead us to engage with various legacies of
liberal, Marxist and other traditions that have sought to reconcile tendencies toward unequal economic accumulation and distribution with the normative
ambition for political equality that has been a defining feature of most modern political ideologies. The prevailing tendency here has been to
define equality in rather narrowly conceived political terms, usually of various mechanisms of democratic representation ,
and to legitimize greater or lesser degrees of economic inequality as a necessary aspect of social life under explic- itly capitalist conditions. Moreover,
claims about political equality have been articulated primarily in relation to the domestic communities of sovereign states
whereas processes of economic accumulation and distribution have increas- ingly come to be understood in relation to the
globalizing dynamics of contem- porary capitalism, dynamics that are in uneasy tension with the political authority of sovereign states.
Hence all the long-standing uncertainties about the relation between the determining logics of the modern states system and
a globalizing system of capitalism, as well as about the capacity of modem states, under distinctly uneven conditions, to
sustain their functional capacities and legiti- mate authorities given the contradictions and convergences between these
logics. Second, we might focus on accounts of the principle and historical experi- ence of the "great powers" as a guarantee
of international "order," again in relation to the formal claims to equality expressed in the system of sovereign states. Here we might engage with the
institutional distinction between the Security Council and the General Assembly of the United Nations, with broad readings of Pax Britannica and Pax
Americana, and with the strange duopoly of the Cold War and the even stranger unipolarity that followed, or with the conceptual ambiguities
buried in claims about "hegemony." In the background, always implicit but now again increasingly explicit, is the worry
that while hegemony might indeed be necessary for the maintenance of interstate order, it simultaneously threatens to
undermine the most fundamental principles of the international system by crossing the vague but crucial line that distinguishes a great
power from an empire. Hence, to be specific, a broad range of contem- porary anxieties about the degree to which U.S. military
force and the pursuit of various unilateralist policies by the second Bush administration signal less a quantitative adjustment
in a familiar world of hegemons in a states system than a qualitative shift to structures of global power and authority
fundamentally at odds with a system of sovereign states. Third, we might focus on the capacities of specific kinds of political
com- munity to participate in anything more than a formal or even token way in the modern system of sovereign states. Here we might look
at the historical pro- cesses of colonization that accompanied the construction of a modern system of sovereign states first in Europe and later, largely in
the mid-twentieth century, in most of the world. In this way we might be drawn to look at how the process of internationalization worked as both a form of
inclusion and a form of exclu- sion, thereby enabling consequent distinctions between North and South, devel- oped and underdeveloped, properly
democratic states and failed states and all the other tropes through which we have been encouraged to read the process of internationalization
simultaneously as a process of modernization. It is in this context that claims about inequality are likely to refer less to the principle
of modern political life expressed in claims about state sovereignty, or the Treaty of Westphalia in 1648, than to various claims
about "civilization" and its absence that might be traced back to the Crusades or the voyages of Christopher Colum- bus and Vasco da
Gama and forward to contemporary predictions of civiliza- tional conflict and the so-called war against terror. Fourth, we might
focus on the constitutive value field in which the inter- national is judged as the negation of the positive values ascribed to
statist forms of political community. Here we would have to engage with the core principles expressed in what has come to
be known as the theory of international relations, a discourse that draws its primary conceptual resources from an insis-
tence that modem politics is organized around a capacity to distinguish between competing sovereignties, and thus between
friends and enemies. Here claims about inequality are always in danger of reducing to an absolutist and radically
dualist account of all values, an account of those who are to count as proper human beings and those who are not-
and of the sovereign capacity to decide who gets to count as the former rather than the latter, the "Other ."
ENDI 2010 61
Security K Wave 1

INEQUALITY LINK

Claims about addressing international inequalities assume a relationship between equality and
inequality that is rooted in modern sovereignty and capitalism
Walker 2002 [RBJ, professor of political science at the University of Victoria, ―International/Inequality,‖ International
Studies Review, Vol. 4, No. 2, International Relations and the New Inequality (Summer, 2002), pp. 7-24, jstor]

Obviously much can and has been said about all these ways of beginning to think about (in)equality and the international. Such a brief and broad sketch
necessarily effaces many indispensable but also problematic categories of class, race, gender, culture and technology, and obscures most of the crucial
debates among different theoretical traditions that are at play in each of these four contexts. Nevertheless, such a sketch is sufficient to underline several
impor- tant points. Again, four seem fairly obvious. First, before trying to make sense of claims about new forms of inequality in an
international context, it is necessary to understand the specific understand- ings of the legitimate relationship between
equality and inequality that is already expressed in the principles of a political life that is somehow international . This might
take us initially into discussions about the different meanings of equality- empirical, philosophical, legal, opportunity-oriented or outcome-oriented, indi-
vidual or collective-that are at play in the practices of modern states. It would certainly take us eventually into dense debates about the
practices of modern sovereignty. Second, while it is often tempting to assume that modern politics is pred- icated on
assumptions about equality as a regulative norm, and thus to under- stand inequalities as aberrations from that norm that
might somehow be eradicated in order to bring us back to our proper normative principles, it is necessary to understand
how "the international" is already constituted, in prin- ciple, as a normative account of the proper relationship between
equality and inequality. It expresses this relationship in the various ways politics and the state are distinguished from
economics or "the market"; in the ways principles of sovereign equality and "domestic jurisdiction" are reconciled with
princi- ples of great-power hegemony and various kinds of "intervention"; in the in- corporation of a linear account of
history as modernization and "development" into a structural account of international order that depends on various histor-
ical practices of exclusion that render specific practices of inclusion necessary and natural; and in the way the most basic
premises of the states system affirm the primacy of the internal as the ground on which to affirm the inferiority of the
external. Third, there is no single ground on which we can speak about inequality, no universal standard against which to measure the
equal and the unequal, even though each of the four themes I have identified expresses very powerful assumptions about what equality must involve. On
the contrary, we have a range of accounts of how the relationship between equality and inequality ought to be expressed in the primary constitutive
practices of modern life, and especially in the practices of the modern state. Finally, in order to engage with claims about novel forms of inequality in an
international context it might be possible, though necessarily misleading, to initiate analysis on any one of the specific grounds identified here, but quite
difficult to take them all into account at once.
ENDI 2010 62
Security K Wave 1

INEQUALITY LINK

Their criticism of inequality overlooks the intrinsicness of hierarchy to the modern political
order—the ability to distinguish between legitimate nad illegitimate discrimination allows
sovereigns to declare states of exception and go to war to eliminate enemies
Walker 2002 [RBJ, professor of political science at the University of Victoria, ―International/Inequality,‖ International
Studies Review, Vol. 4, No. 2, International Relations and the New Inequality (Summer, 2002), pp. 7-24, jstor]

To think about the practices of discrimination and the ways in which these practices have been authorized in specific ways under specific conditions is to
get some sense both of the practices through which claims about inequality and the international have been constituted historically and of how these claims
can be understood in relation to what seem like four quite distinctive framings of how the relationship between inequality and the international must be
under- stood. Not surprisingly, perhaps, we are led to think about ways of dividing up the world which have been expressed partly
in a vertical plane (the inequalities of economic accumulation and class antagonisms, the discrepancy between sov- ereign
equality and great-power hegemony) and partly in a horizontal plane (the inclusionary/exclusionary logics of the system of
territorial states and prac- tices of alterity associated with categories of race, gender, culture, colony, civ- ilization and so
on), and thus to various puzzles about how these different expressions might be related or reconciled. Modern political life
tends to affirm the natural necessity of a world that can be organized partly as a system of horizontal inclusions/exclusions
and partly as a set of vertical hierarchies within these inclusions/exclusions; that is, to affirm a specific set of practices of dis- crimination.
Modern accounts of what it must mean to invoke categories of equality and inequality also express these practices, though they do so mainly by focusing
on hierarchies in a vertical plane rather than the horizontal inclusions/ exclusions that make these hierarchies possible. The key difficulty posed by
contemporary accounts of inequality is that they tend to suggest that the dis- criminations that enable modern politics to
sustain its characteristic inclusions and hierarchies seem less and less persuasive to the contemporary political imagination .
Let me work backwards. Modern politics is conventionally understood in relation to some version of a story about the gradual dissolution of medieval, or
feudal, or theological forms of social organization and obligation and its replacement by, or mutation into, the flat territorial spaces expressed by modern
states and modern subjec- tivities. Sovereigns came to monopolize authority in their own territory. Indi- vidual subjects came to aspire to autonomy, to a
capacity to internalize universal reason within their own particularities. At least this is the story that stands as the regulative ambition affirmed in the
standard claims of modern states and the most refined (Kantian) accounts of what it means to be a modern subject free from dependence on other subjects.
Within modern states politics came to be organized around a series of prob- lematic relations between the sovereign and the
subject, a relationship that could be constructed in some now familiar ways, especially by privileging the sovereign
authority to deny or permit the freedom of particular subjects or by privileging the capacity of particular subjects to
constitute a sovereign author- ity to limit particular freedoms. Hierarchy was thus rewritten in a modern abstract scale
reaching from the many individual subjects to the unitary sovereign that both constituted and was constituted by many
abstract subjects. Other forms of hierarchy, sometimes feudal, or aristocratic, or theological, sometimes the prod- uct of new
relations of production distribution and exchange, were largely orga- nized so as to fit in with this new abstract relation
between sovereigns and subjects, an organization understood primarily in terms of a story about democ- racy and the mechanisms through which
the (particular) many might be repre- sented in the deliberations and decisions of the unitary sovereign that successfully claim to speak for the (particular)
many. Between states, sovereigns had rela- tions with other sovereigns, sometimes coexisting, sometimes exercising sov-
ereign capacities to declare an exception to normal life and go to war, and always fearful that the system of states that
enabled them to sustain their claims to universality in a field of particularities would collapse . Hierarchy emerged here
precisely because sovereigns were not as equal in practice as they might have claimed to be in principle . The hegemonic
role of particular states, or coalitions of states, or even quasi-institutional regimes generated by coalitions of states, could
therefore be tolerated so as to permit some kind of stability and order in the system despite the dangers associated with
competition for heg- emonic status and the possibility that hegemons would start acting like "world powers," to use Martin
Wight's term,7 "hyperpowers" to use a term of current European diplomatic usage, or, the most worrying case, emperors. This modern world of
sovereign subjects generated new problems, not the least of which being how to reconcile claims about freedom with claims
about equality. Both claims came to have foundational status as definitive principles of modern political life. At least from Hobbes's account of the
anarchical con- sequences of putting these two principles into motion as a competition for desire after desire, and not least from Marx's analysis of the
inequalities deriv- ing from the necessary contradiction between the value of human labor and the value of exchange in a capitalist market, these definitive
principles of modern political life have been understood to be radically at odds with each other. This has generated much of the dynamism of modern
democratic societies, as well as the characteristic puzzles of liberal political theory. In relations between states, however, the contradiction is rather more
straightforward. Although some people with an excessive fondness for the elegant models (and models of elegance) favored by certain forms of economics
often speak of an inter- national anarchy, hardly anyone imagines that they are thereby referring to two hundred or so agents acting in the way portrayed in
Hobbes's account of atom- istic individuals. Whether read as anarchy or as hegemony, however, the international is under- stood to
be radically different from, and in a crucial sense inferior to, the world of a proper politics within states. In a tradition that might
be taken back to the classical polis, it is only within a defined community that it is possible to live a properly political life, or, in a more modern format, a
life in which democratic practices of some sort make it possible to reconcile freedom and equality, among other things, in a responsible and authoritative
manner. Between states, in the system with no overarching authority, war remains the final resort . The limit condition of
modern political life is marked by a capacity to declare an excep- tion to all domestic norms and to inscribe an absolutist
discrimination between friend and enemy on a spatial ground of statist inclusions and exclusions. This limit condition is widely
felt to be unacceptable.
ENDI 2010 63
Security K Wave 1

INEQUALITY LINK - DISCRIMINATION

The affirmative buys into a modern sovereign determination of establishing limits to


discrimination, which legitimizes ―justifiable‖ discrimination and draws boundaries between
deserving and undeserving subjects
Walker 2002 [RBJ, professor of political science at the University of Victoria, ―International/Inequality,‖ International
Studies Review, Vol. 4, No. 2, International Relations and the New Inequality (Summer, 2002), pp. 7-24, jstor]

Claims about (in)equality are simi- larly vulnerable to suspicions about the ways in which modern political dis- courses work
with questionable concepts of discrimination. Modern politics-that is, the kind of politics we associate with the forms of statist political
community in a system of statist communities that took shape in post-Renaissance Europe-is explicitly constructed as a precise and intricately articulated
system of discriminations. It works especially on the basis of constitutive distinctions between legitimacy and illegitimacy, rulers
and unruly, insiders and outsiders. Among many other things, it is, or has come to be, a practice that tells us what politics is, or is not,
and thus a practice that has to be understood in terms of how it has come to be defined rather than in terms of stipulative
definitions of what it is. As such, modern politics enacts systems of discriminations that in turn enable sequences of
subsequent discriminations, including those that tell us when we should and should not discriminate, or under what
conditions we might consider ourselves to be discriminating, or when we should and should not challenge the
discriminations that make all subsequent discriminations possible. Here one might want to privilege Locke's fateful distinction between
"the world that God gave to man in common" and the right to private property, as well as the subsequent reworking of this theme in Marx's theory of value
on the one hand and the marginalist school of market economics on the other. Or one might prefer to privilege the struggles between sacred and secular
authority that continue despite the stakes buried in God's long dead ghost; or between public and private ethics despite the stubborn elusiveness of any
clear distinction between public and private. Or perhaps one might begin to unravel the reified dualisms of male and female, the West and the rest, the
human and the animal and so on, through which characteristically modern accounts of political iden- tity and subjectivity are enacted, legitimized and
sometimes subverted. Or we might think about the ways in which the discriminations of modern politics enact novel ways of
making discriminations, a modem challenge to a world that could envisage Plato carving nature at the joints or Saint Thomas filling in the gaps
between time and eternity articulated by Saint Augustine, and an affir- mation of the need to impose names and definitions on a phenomenal world that
does not conveniently disclose itself through a name. All these established entries to the politics of modern discriminations and the
discriminations of modern politics are undoubtedly important. They enable and are enabled by specific political practices.
The capacity to make a differ- ence, to act in such a way that differences are made, is crucial to the mysteries of political power. The capacity to act
on the basis of differences made, of discriminations enacted between the legitimate and the illegitimate, between the
properly political and everything else, is crucial to the mysteries of political authority. Thinking about modern practices of
discrimination can take us in many directions and to many engagements with many different forms of analy- sis across
many scholarly disciplines and traditions. In whichever direction we go, however, we are likely to become aware very quickly that modern
politics expresses the demand for some sovereign power/authority to distinguish that which is properly political and that which is not, or, to follow Carl
Schmitt's awesome summation of our Hobbesian legacy in this respect, that which is the general rule and that which is the exception to the general rule.3
For all that accounts of political power might be informed by the capacity of states to make a difference, to mobilize
physical force and administer the differences made, and for all that accounts of legitimate authority might be informed by
claims about the ethical, cultural or democratic sources of legitimation, modern poli- tics ultimately affirms a claim that the
capacity to act, and to claim to act legitimately, rests with the sovereign, conceived largely as a constitutional, and
constitutive, abstraction. It was perhaps Hobbes who most clearly articulated the distinction between the political and the nonpolitical as an
achievement of modern sovereignty; a sovereignty, that is, expressed not in the body of any particular being but in the abstract embodiment of a particular
ensemble of beings. It is this that marks Hobbes as the most important textual expression of the convergence between specifically modern forms of
discrimination and the practices of modern polit- ical authority, and that ensures his centrality to contemporary attempts to come to terms with the limits of
modem politics.
ENDI 2010 64
Security K Wave 1

INEQUALITY LINK - DISCRIMINATION

The affirmative establishes a teleological narrative about discrimination in which the


advancement of modern states depends on who they bring under their protection—this
exaserbates the divide between inside and outside
Walker 2002 [RBJ, professor of political science at the University of Victoria, ―International/Inequality,‖ International
Studies Review, Vol. 4, No. 2, International Relations and the New Inequality (Summer, 2002), pp. 7-24, jstor]

Human life, it is not difficult to argue, especially on the basis of a rich tradition of liberalism and humanism, cannot be reduced to a simple
binary of friend and enemy. Yet what is widely considered to be unacceptable is in fact the constitutive condition enabling
the modern account of a political community within a sovereign state. Many con- tradictions are at play here. Many people wish
they would simply go away, or simply assume that the limits of the modern state are not the limits of modern politics, or at
least of modern liberalism. Many others wish that modern polit- ical analysis, and especially modern liberalism, would confront
the statist lim- its of modern politics in a more serious way and recognize the fundamental antagonism, and valorization,
that enables us to speak of equality on a political ground that insists on the priority of the particular (even if universalizing)
state/community/friend over all other states/anarchies/enemies. Some of the most powerful modern stories about equality and
inequality start here. The regulative antagonism between friends and enemies that both defines and limits, enables and threatens,
political life in the modern state is itself shadowed and enabled by a constitutive antagonism at the limit of the modern system of states. This antagonism
is usually framed in temporal rather than spatial terms, as the necessity for modernization as a process that will bring all
societies into the system of sovereign states that guarantees their membership in a particular sort of community of humankind.
The most influential narratives here derive from Kant's articulation of the possibility of a cosmopolitan his- tory, a temporal shift to a condition of maturity
and independence, the shift that expresses all the most seductive rhetorics about progress and growing up, about the potential unity of humanity to be
realized in a universalizing History. Told from the inside, this is always a beguiling story, unless one is suspicious of a human society that is afraid of
dependence on others and celebrates a conform- ist egoism as a model of autonomy. Told from the outside, it offers a paradig- matic
expression of modern racism, and an open door to rabble-rousing cliches about a "coming anarchy" or a "clash of civilizations." 8
The key difficulty here, however, is less the genealogy of a set of categories than the historical practices through which
these categories have been instan- tiated within structures in which the "North" and the "developed" have indeed been
distinguishable from the "South" and the "undeveloped." Various tradi- tions have tried to explain these structures-accounts
of colonialism, neocolo- nialism, imperialism, dependency, center-periphery relations, the rearticulation of capital-state relations and
so on. In whatever form, however, claims about inequality have been framed largely as a pattern of spatial exclusion that
expresses a quasi-Kantian narrative about the necessary process of inclusion into the mod- ern states system as its key
legitimation. The relative ease with which various kinds of liberalism have converged with ostensibly illiberal invocations of ter- ror or Islam after the
violence of September 11, 2001 illustrates the tenacity of modern theories of international relations as a set of claims about the temporal incorporation of
all others into modernity quite as much as a set of claims about the consequences of sovereignty in a system of sovereign states. Indeed, the temporal
story, the bringing in of an historically excluded outside into a system of states that is itself constituted as a structure
of insides and outsides, is a condition of the possibility of the modern international. Kant simply affirms Hobbes in this respect.
A story about time enables a story about space that in turn enables a legitimation of inequality . Entry into the modern
system of states enables any particular sovereign to decide on an exception to the norms of human conduct within a
particular territory.
ENDI 2010 65
Security K Wave 1

INEQUALITY LINK – INTERNATIONAL / CAPITALISM

Claims about international inequality occur within a specificially modern discursive field that
naturalizes modern binaries between friend/enemy and savage/modern
Ashley 1984 [Richard, professor of political science at Arizona State University, ―The Poverty of Neorealism,‖
International Organization, Vol. 38, No. 2 (Spring, 1984), pp. 225-286, jstor]

First, to
use concepts like equality and inequality, and to treat inequality as a problem, is to work with a specifically modern
discursive field. In part, this leads us to think about the shift to a modern quantitative understanding of scale, and the degree to which fundamental
notions of higher and lower have largely been recast as a problem rather than as a condition of a properly ordered (aris- tocratic) society. In part it leads us
to think about the ways in which this prob- lematic relationship between higher and lower was articulated symbiotically with two crucial framings of
inclusion and exclusion, one involving the spatial limits of the modern state, one involving the spatial limits of the modern system of states. Modern
accounts of equality and inequality assume a common ground on which something can be measured, and compared across jurisdictions. But this common
ground is traversed by sharp limits, by massively institutionalized capacities to discriminate between the inside and the outside, whether between friends
and enemies within and between sovereign states or between those who can be brought into the system of states and those who cannot or will not. Part of
the brilliance of Hobbes's story of the shift from state of nature to a state of society is that it conflates these two very different narratives of inclusion/
exclusion into a single story about before and after, anarchy and community, the primitive and the modern. Theories of international relations are
thereby enabled to treat the international as synonymous with the world as such, to read all forms of inclusion and
exclusion as a relation between friends and enemies, and either to leave the temporal process of modernization, of bringing
the unmod- ern into modernity, to some other discourse, or to use all the half-buried tropes of civilizational difference as a
way of building up a statist logic of friends and enemies into a battle between the civilized and the damned, those who are
capable of becoming equal because they are unequal and those who seem to be incapable of becoming equal because they
are unequal.
ENDI 2010 66
Security K Wave 1

LINK HELPERS

Securitizing minority groups leads to their exclusion and violent reaction against dominant
groups
Cannetti-Nism et. al, 2008 (Daphna, a assistant professor of political science at the University of Haifa, Gal Ariely at the National Security
Studies Center, and Eran Halperin, the Deputy Director of Political Psychology at IDC with a PhD in political science from the University of Haifa, Life,
Pocketbook or Culture, the Role of Percieved Security Threats in Promoting Exclusionist Politics, Political Research Quarterly, Vol 61 Number 1 March
2008pp. 90-103]

To recapitulate existing claims and findings, economic (realistic) and symbolic threats are quite common explanations for exclusionist attitudes toward
minorities. Following recent post-9/11 political studies (e.g., Huddy et al. 2002; Huddy 2003; Sniderman, Hagendoorn, and Prior 2004), we
contend that perceived threat to the security of one's reference group might have a significant impact on attitudes
toward out-groups. In such cases, the rationalization under lying the perceived threat is based on fear stemming from
involvement of members of a minority group in actions (violent and nonviolent) undermining the existence of the host
state, terror attacks included (Huddy et al. 2002). Intense fear may induce generalized attribution to the minority group,
delegitimization of the out-group members, hatred, and eventually exclusionist and other aggressive political attitudes
toward them (for a literature review, see Jarymowicz and Bar-Tal 2006). We emphasize that economic and security threats are different facets of
realistic threat, based on a challenge to the group's economic or physical well-being, and distinct from symbolic threats, which challenge the in-group's
norms and values. Thus, the question we raise is, In a post-9/11 world, what is the relative impact of each of the three sources of threat on political
exclusionist attitudes toward different types of minorities? We assume that because security threats are threats to life itself, they will have a prominent
impact on exclusionist attitudes. It should be noted that this study focuses on collective security threats (e.g., political violence), rather than personal threat
(e.g., crime). We further assume that the role of economic versus symbolic threat is dependent on the specific contexts of the relationships between the
out-group and the in-group, a topic with which we deal in the fol lowing section.1

Dominant groups create perceived threats and prejudice towards others in order to maintain
control
Stephan and Stephan 2000 (Walter G- professor of psychology at the University of New Mexico, and Cookie
White- a professor of Sociology at the University of New Mexico, Reducing Prejudice and Discrimination, An Integrated
Threat Theory of Prejudice]

The first types of threats included in our model consist of realistic threats composed of the outgroup .
Realistic threats are threats to the very
existence of the ingroup ( e.g through welfare), threats to the political and economic power of the ingourp, and threats to
the physical or material well-being of the ingroup or its members (e.g their health). The concept of realistic threats has its origins in
realistic group conflict theory. This theory, as developed by Levine and Campbell and by Sherif, was primarily concerned with conflict for scarce
resources, such as territory, wealth, or natural resources (Bobo 1988; Levine & Campbell, 1972; Sherif 1966). Our concept of realistic threat
differs from the idea of threat emphasized in realistic group conflict theory in two ways. First, our focis is broader,
encom[assign any threat to the welfare of the group or its members. Second, we focus on subjectively perceived conflict
between groups. We emphasize perceived realistic threats because the perception of threat can lead to prejudice,
regardless of whether or not the threat is ―real‖.
ENDI 2010 67
Security K Wave 1

SECURITIZATION BAD – KILLS CRITICISM

Securitization closes off and depoliticizes criticism


Jenny Edkins, Senior Lecturer in International Politics at the University of Wales Aberystwyth, Postructuralism and
International Relations: Bringing the Political Back In, 1999, p. 10-11
A second example in the field of international politics is the process of securitization.54 Securitization, or claiming that something is an issue
of national security, removes it from one arena within which it is debated or contested in a certain way and takes it to another,
where the priorities are different. Once something has been "securitized," this changes the terms of the debate. Certain questions
can no longer be asked. In the security studies literature, securitization is seen as a further step beyond what is called there "politicization." Barry
Buzan, Ole Waever, and Jaap de Wilde explain how they see "securitization": "Security" is the move that takes politics beyond the established rules of the
game and frames the issue either as a special kind of politics or as above politics. Securitization can thus be seen as a more extreme version of
politicization. In theory any public issue can be located on the spectrum ranging from nonpoliticized meaning the state does not deal with it and it is not in
any other way made an issue of public debate and decision) through politicized (meaning the issue is part of public policy, requiring government decision
and resource allocations or, more rarely, some other form of communal governance) to securitized (meaning the issue is presented as an existential threat,
requiring emergency measures and justifying actions outside the normal bounds of political procedure).55 Buzan, Waever, and de Wilde's use of
"politicized" is quite distinct from what mine would be.56 What they call "politicization" I would call "depoliticization": When an issue
becomes, as they say, "part of public policy, requiring government decision and resource allocations," it becomes for me part
of "politics" and hence, as I have argued above, "depoliticized." I would agree that securitization is a further step in the same direction,
but for me that direction is one of depoliticization. When issues are "securitized," they are even more firmly constrained within the
already accepted criteria of a specific social form. And that constraint is even more firmly denied. The state as a form of
society has defined itself in large part around what it will consider as "security threat" and what mechanisms it will adopt
for dealing with it. Issues of "security" are more removed from public debate and decision than issues of "politics"; in most
cases these issues are secret, and even the existence of such matters is concealed. Decisions about them are taken in
technical terms, following the advice of experts in military affairs or defense. Securitization is technologization par
excellence.
ENDI 2010 68
Security K Wave 1

SECURITIZATION BAD – VIOLENCE

The war logic of securitization leads to the violent elimination of enemies


Aradau, 2001 [Claudia, Research Associate in the Centre of International Relations, Department of War Studies, King‘s
College London, December. RUBIKON, Date accessed May 10, 2007,
http://venus.ci.uw.edu.pl/~rubikon/forum/claudia2.htm]
Survival refers not only to the fear of death, but implies countermeasures, the ‗extraordinary measures‘ of the CoS. Michael Dillon
has formulated the appeal to security as necessarily implying a ‗specification, no matter how inchoate, of the fear which
engenders it‘ and hence calls for ‗counter-measures to deal with the danger which initiates fear, and for the neutralization,
elimination or constraint of that person, group, object or condition which engenders fear‘ .[13] These countermeasures are directed at
the other, the enemy to be eliminated. Or in metaphoric terms, to use Jef Huysmans‘ favorite analogy society-garden, counter-measures refer to
unearthing the weeds threatening the harmonious growth of the garden .[14] The metaphor of war is constitutive of what both Ashley and
Campbell have called the ‗paradigm of sovereignty‘.[15] In Campbell‘s formulation, sovereignty signifies ‗a center of decision presiding
over a self that is to be valued and demarcated from an external domain that cannot and will not be assimilated to the
identity of the sovereign domain‘.[16] This process of demarcation of friends and enemies, delineation of boundaries of
order versus disorder has been the prerogative of the sovereign state, provider of security within its boundaries and
preserver of ‗law and order‘. The injunction to preserve the internal order of the modern harmonious garden has targeted
both internal and external enemies, the weeds that need to be rooted out for the benefit of the political community. As the
‗war on drugs‘ will clearly illustrate, this approach is highly ambiguous for a political community predicated upon the
friend/enemy differentiation. In this political community constituted upon the logic of war, securitizing moves are only
liable to breed insecurity. Elimination of enemies or their circumscribing is the ultimate goal of war. Thus the sovereign logic
of security ultimately endangers, threatens those who threaten us and in this sense it has disquieting effects on the political
community. Moreover, the mutual constitutiveness of threats and threatened objects leads to a spiral of enemy constructions. The enemy needs to be
eliminated and at the same time the very identity of society, for example, depends on enemy construction. The war logic of security is thus likely
to lead to a paradoxical story, in which security is only likely to breed more insecurity and eventually violence.

Their realist conception of international relations guarantees extinction--it fails to encompass the
reality of the political subject of violence. Our critique turns all their practicality claims.
David Campbell\Michael Dillon, ―The end of philosophy and the end of international relations,‖ The Political Subject
of Violence, 1993, pp. 17-18
To broach this task anew, however, we have briefly to re-visit again an aspect of the early formation of the terminus in which we are located. Should the
old objection be advanced that a return to the ethical represents a retreat from the hard violent choices entailed in the political, the reply before we proceed
should be brief and 'hard-nosed' enough to match any realist. Violence may be the ultima ratio of politics, but it has never been the only
ratio; and in a life that now has to be lived with a proliferating array of devices capable of threatening lethal global consequences it simply cannot
be allowed to enjoy the practical, intellectual and moral licence once extended to it in our political discourses. Neither is
there anything in the history of the technology of political violence to warrant the claim that the political rationalisation of violence diminishes its sway.
Monopolistic control and attempted rational deployment of the legitimate use of force by modern political authorities has
helped bring human being to the threshold of planetary survival. The technology of modernity's political settlement
realises its end in the real prospect ultimately not only of genocidal but also of species extinction. Human perdurance cannot afford
the cost of the politics of political and ethical forgetting charged by the technologising of the political as violence. Realist
and neo-realist answers not only fail intellectually - in a way that would not matter very much if they did not so impoverish our political
imagination - they fail most because they are not good enough practically to match our circumstances. It is not a matter of
getting knowledge 'to represent reality truly' (we shall see later how modern reality has become a function of its technologies of representation),
but of acquiring 'habits of action for coping with reality';" a reality which always exceeds the realist representation of it,
and whose unprecedented finitudes now define the horizon of life in novel ways.
ENDI 2010 69
Security K Wave 1

SECURITIZATION BAD - RESENTMENT

Securitization produces resentment of difference and constructs scenarios through causality and
rationality
Der Derian 1998 [James, prof of political science at Brown, ―The Value of Security: Hobbes, Marx, Nietzsche, and
Baudrillard On Security,‖ ed. Ronnie Lipschutz, http://www.ciaonet.org/book/lipschutz/index.html]

The desire for security is manifested as a collective resentment of difference--that which is not us, not certain, not
predictable. Complicit with a negative will to power is the fear-driven desire for protection from the unknown. Unlike the
positive will to power, which produces an aesthetic affirmation of difference, the search for truth produces a truncated life
which conforms to the rationally knowable, to the causally sustainable . In The Gay Science , Nietzsche asks of the reader: "Look, isn't
our need for knowledge precisely this need for the familiar, the will to uncover everything strange, unusual, and questionable, something that no longer
disturbs us? Is it not the instinct of fear that bids us to know? And is the jubilation of those who obtain knowledge not the jubilation over the restoration of
a sense of security?" 37 The fear of the unknown and the desire for certainty combine to produce a domesticated life, in which
causality and rationality become the highest sign of a sovereign self , the surest protection against contingent forces. The
fear of fate assures a belief that everything reasonable is true, and everything true, reasonable . In short, the security
imperative produces, and is sustained by, the strategies of knowledge which seek to explain it. Nietzsche elucidates the nature of
this generative relationship in The Twilight of the Idols : The causal instinct is thus conditional upon, and excited by, the feeling of fear. The "why?"
shall, if at all possible, not give the cause for its own sake so much as for a particular kind of cause --a cause that is comforting, liberating and relieving. . .
. That which is new and strange and has not been experienced before, is excluded as a cause. Thus one not only searches for some kind of explanation, to
serve as a cause, but for a particularly selected and preferred kind of explanation--that which most quickly and frequently abolished the feeling of the
strange, new and hitherto unexperienced: the most habitual explanations. 38 A safe life requires safe truths. The strange and the alien remain
unexamined, the unknown becomes identified as evil, and evil provokes hostility--recycling the desire for security. The "influence of timidity," as
Nietzsche puts it, creates a people who are willing to subordinate affirmative values to the "necessities" of security: "they fear change, transitoriness: this
expresses a straitened soul, full of mistrust and evil experiences." 39 The unknowable which cannot be contained by force or explained
by reason is relegated to the off-world. "Trust," the "good," and other common values come to rely upon an "artificial strength": "the feeling of
security such as the Christian possesses; he feels strong in being able to trust, to be patient and composed: he owes this artificial strength to the illusion of
being protected by a god." 40 For Nietzsche, of course, only a false sense of security can come from false gods: "Morality and religion belong altogether
to the psychology of error : in every single case, cause and effect are confused; or truth is confused with the effects of believing something to be true; or a
state of consciousness is confused with its causes." 41
ENDI 2010 70
Security K Wave 1

HEG BAD-IMPERIALISM

US hegemonic imperialism will cause devastation on an unprecedented scale, causing global war
and unleashing new global holocausts. We must reject this endless cycle of violence. Vote
negative to resist imperialism
Foster 2003 [John Bellamy Foster is co-editor of Monthly Review, professor of sociology at the University of Oregon,
―The new Age of Imperialism,‖ Monthly Review 55.3]

At the same time, it


is clear that in the present period of global hegemonic imperialism the United States is geared above all to
expanding its imperial power to whatever extent possible and subordinating the rest of the capitalist world to its interests .
The Persian Gulf and the Caspian Sea Basin represent not only the bulk of world petroleum reserves, but also a rapidly increasing proportion of total
reserves, as high production rates diminish reserves elsewhere. This has provided much of the stimulus for the United States to gain greater control of
these resources--at the expense of its present and potential rivals. But U.S. imperial ambitions do not end there, since they are driven by economic
ambitions that know no bounds. As Harry Magdoff noted in the closing pages of The Age of Imperialism in 1969, "it is the professed goal" of U.S.
multinational corporations "to control as large a share of the world market as they do of the United States market," and this hunger for foreign markets
persists today. Flo rida-based Wackenhut Corrections Corporation has won prison privatization contracts in Australia, the United Kingdom, South Africa,
Canada, New Zealand, and the Netherlands Antilles ("Prison Industry Goes Global," www.futurenet.org, fall 2000). Promotion of U.S.
corporate interests abroad is one of the primary responsibilities of the U.S. state . Consider the cases of Monsanto and genetically
modified food, Microsoft and intellectual property, Bechtel and the war on Iraq. It would be impossible to exaggerate how dangerous this
dual expansionism of U.S. corporations and the U.S. state is to the world at large. As Istvan Meszaros observed in 2001 in Socialism
or Barbarism, the U.S. attempt to seize global control, which is inherent in the workings of capitalism and imperialism, is now threatening
humanity with the "extreme violent rule of the whole world by one hegemonic imperialist country on a permanent basis...an
absurd and unsustainable way of running the world order." * This new age of U.S. imperialism will generate its own
contradictions, amongst them attempts by other major powers to assert their influence, resorting to similar belligerent means, and
all sorts of strategies by weaker states and non-state actors to engage in "asymmetric" forms of warfare. Given the unprecedented
destructiveness of contemporary weapons, which are diffused ever more widely, the consequences for the population of the world
could well be devastating beyond anything ever before witnessed. Rather than generating a new "Pax Americana" the United
States may be paving the way to new global holocausts. The greatest hope in these dire circumstances lies in a rising tide
of revolt from below, both in the United States and globally. The growth of the antiglobalization movement, which dominated the world stage for
nearly two years following the events in Seattle in November 1999, was succeeded in February 2003 by the largest global wave of antiwar protests in
human history. Never before has the world's population risen up so quickly and in such massive numbers in the attempt to stop an imperialist war. The
new age of imperialism is also a new age of revolt. The Vietnam Syndrome, which has so worried the strategic planners of the imperial order
for decades, now seems not only to have left a deep legacy within the United States but also to have been coupled this time around with an Empire
Syndrome on a much more global scale--something that no one really expected. This more than anything else makes it clear that the
strategy of the American ruling class to expand the American Empire cannot possibly succeed in the long run, and will
prove to be its own--we hope not the world's--undoing.
ENDI 2010 71
Security K Wave 1

MYTHS IMPACT

Foundational myths naturalize state authority and violence as teleological outcomes of history
and constrain subjectivity
Edkins 1999 [Jenny, Senior Lecturer in International Politics at the University of Wales Aberystwyth, Postructuralism and
International Relations: Bringing the Political Back In, p. 8-9]

Thus, moments of transition, where there is a sense of openness, of decision, are both moments of the political and moments in which subjectivity is called
into play. They are also moments that constitute the social or symbolic order. Or rather, moments at which, through the presupposition of the
existence of a new social system, such a system is brought into being. Not only is the new society founded, but it is
produced as inevitable, authoritative, and legitimate: as if it has always already existed or been prophesied. The
contingency of its origin is concealed. At that moment, once the foundational myth of the new social or symbolic order is
(re)instated, the subject as such disappears, and with it the "political"-to be replaced by "politics." What is more, the interregnum,
where there was a brief openness, is forgotten: de-scribed or un-written by the "writing" of the history of the new state. The
act of the subject "succeeds by becoming invisible-by 'positivising' itself in a new symbolic network wherein it locates and
explains itself as a result of historical process, thus reducing itself to a mere moment of the totality engendered by its own
act."38 This happens when events are "read" backward or retroactively: at that point it is easy to explain "objectively" why certain forces were effective
and how particular tendencies "won." Indeed the Lacanian definition of "act" is just this: "a move that, so to speak, defines its own conditions;
retroactively produces the grounds which justify it."39 This is where the notion of ideology as social fantasy, which I discuss in detail in Chapter 6, comes
in. Once the new symbolic order is in place, the contingencies that gave rise to it are obliterated-they disappear-and a new
version of social reality is established. The role of ideology here is to conceal the illegitimate, unfounded nature of what we
call social reality, what Ziiek calls "social fantasy." Ideology supports the principle of legitimacy upon which the new state is "founded" and conceals
its "impossibility." It does this in part by defining "politics" as a subsystem of the social order and obliterating "the political"-its unfounded founding
moment: "'Politics' as 'subsystem,' as a separate sphere of society, represents within society its own forgotten foundation, its
genesis in a violent abyssal act-it represents, within the social space, what mustfall out if this space is to constitute itself."40
Or as 2iiek expresses it more provocatively, "Politics as subsystem is a metaphor of the political subject, of the Political as subject."41 In other words, it
is "politics," viewed as one of the subsystems of all the systems that go to make up the social order, that enables us to escape or forget the
lack of "the political" and the absence of the possibility of any political action. We are confined by this process to activity
within the boundaries set by existing social and international orders, and our criticism is restricted to the technical
arrangements that make up the "politics" within which we exist as "subjects" of the state. The political subject and the
international subject, too, are safely caged and their teeth pulled.
ENDI 2010 72
Security K Wave 1

KRITIK TURNS CASE-WAR

The kritik turns the case – nation-state securitization perpetuates the violence and wars the aff
attempts to solve.
Ukeje, 2005 [Charles Professor of International Relations at Obafemi Awolowo and scholar at the Centre for African
Studies, Submitted at 11th CODESRIA General Assembly, Rethinking Africa‘s Security in the Age of Uncertain
Globalisation: NEPAD and Human Security in the 21st Century,
http://codesria.org/Links/conferences/general_assembly11/papers/ukeje.pdf]

Howbeit, the quest to redirect security towards human centred concerns raises several problems. In the first instance, ―human security‖ is still heavily
contested in its definition, scope and utility. The concept is criticised for overstretching the traditional notion of security- much the same way that
environmental security did over the last decade. Another criticism is that ‗human security‘ is far too universalistic, containing
―conceptual flaws‖ that raises false priorities and hopes regarding the securitization of human beings. The orthodox
conception of security, either focusing on the internal or external dimensions to insecurity, tend to restrict the concept to the
political survivability and effectiveness of states and regimes, and in doing so, excluded economic, environmental, cultural
and other non-political threats. It puts the state (and politics) at the centre of the conceptualisation of security, suggesting that non-political
threats ―become integral components of our definition of security only if they become acute enough to acquire political dimensions and threaten state
boundaries, state institutions, and regime survival‖ (Vayrynen, 1995: 260). Another limitation of the concept of human security is that it
cannot be fully consummated for as long as the quest for peace and security remains tied to the authoritarian values and
motivations of those in power, human security would continue to suffer breaches and abuses as regime/ state security
further allows official violence to multiply (Sabelo, 2003: 306; Niukerk, 2004). Adele Jinadu (2000) offered further perspectives on how
human security suffers in the attempts by custodians of the state to retain and extract compliance through the instrumentality of force and coercion. He
explained that the problematic of peace and security is ―intrinsically bound up with human nature, especially the dialectics of the social psychology of
human interactions, under conditions of scarcity and choice‖. Accordingly, the problem of peace and security ―cannot and should not be divorced from the
dialectics of domination and subjection, in order words from considerations of superordinate/ subordinate relations at the community, national and global
levels‖ (Jinadu, 2000: 1). The crucial question, as he pointed out is ―[If] humankind cannot create a perfect society, given human nature and the reality of
scarcity, as well as the difficult and contentious questions of choice which scarcity poses, what needs to be done to create a less imperfect society? Under
what conditions can such a less imperfect society expected to emerge and thrive? He argued that the ―modern state…continues to be the pre-
eminently contested terrain of hegemonic groups in national and international society, serving predatory group interests,
and itself becoming part of the problem, the core avenue of contention and conflict, a major impediment to structural
reform and, therefore, a major obstacle to peace and security, which requires in many cases, reconstitution and
reconstruction as a necessary condition for the enthronement and durability of peace and security‖ (Jinadu, 2000: 2-3). As shall be
discussed in the next section, what the above implies, in part, is that NEPAD must first resolve the underpinning motivation of power and militarism; of
superordinate and subordinate.
ENDI 2010 73
Security K Wave 1

2NC ALT SOLVES


Rejecting securitization opens up space for emancipatory political engagement more likely to
deal with real world problems
Neocleous, 2008 [Mark, Professor of the Critique of Political Economy; Head of Department of Politics
& History Brunel Univ, Critique of Security, 185-6]

The only way out of such a dilemma, to escape the fetish, is perhaps to eschew the logic of security
altogether - to reject it as so ideologically loaded in favour of the state that any real political
thought other than the authoritarian and reactionary should be pressed to give it up. That is
clearly something that can not be achieved within the limits of bourgeois thought and thus could never even begin to be
imagined by the security intellectual. It is also something that the constant iteration of the refrain 'this is
an insecure world' and reiteration of one fear, anxiety and insecurity after another will also make it hard to do. But it
is something that the critique of security suggests we may have to consider if we want a political way
out of the impasse of security. This impasse exists because security has now become so all-encompassing that
it marginalises all else, most notably the constructive conflicts, debates and discussions that
animate political life. The constant prioritising of a mythical security as a political end - as the
political end constitutes a rejection of politics in any meaningful sense of the term. That is, as a
mode of action in which differences can be articulated, in which the conflicts and struggles that arise
from such differences can be fought for and negotiated, in which people might come to believe that another world

is possible - that they might transform the world and in turn be transformed. Security politics simply removes this; worse, it remoeves it while purportedly addressing it.
In so doing it suppresses all issues of power and turns political questions into debates about the most

efficient way to achieve 'security', despite the fact that we are never quite told - never could be told - what might count as having achieved it.
Security politics is, in this sense, an anti-politics,"' dominating political discourse in much the same manner as the security state tries to
dominate human beings, reinforcing security fetishism and the monopolistic character of security on the political

imagination. We therefore need to get beyond security politics, not add yet more 'sectors' to it in a
way that simply expands the scope of the state and legitimises state intervention in yet more and more areas of our lives. Simon Dalby reports a personal communication with
you take away security, what do you put in
Michael Williams, co-editor of the important text Critical Security Studies, in which the latter asks: if

the hole that's left behind? But I'm inclined to agree with Dalby: maybe there is no hole."' The mistake has been to think that there is a
hole and that this hole needs to be filled with a new vision or revision of security in which it is re-mapped or civilised or gendered or humanised or expanded or whatever. All of
these ultimately remain within the statist political imaginary, and consequently end up reaffirming the state as the terrain of modern politics, the grounds of security. The
real task is not to fill the supposed hole with yet another vision of security, but to fight for an alternative political
language which takes us beyond the narrow horizon of bourgeois security and which therefore does not constantly throw us into the
arms of the state. That's the point of critical politics: to develop a new political language more adequate to the kind of society we want. Thus while much of what I
have said here has been of a negative order, part of the tradition of critical theory is that the negative may be as significant as the positive

in setting thought on new paths. For if security really is the supreme concept of bourgeois society and the fundamental thematic of liberalism, then to
keep harping on about insecurity and to keep demanding 'more security' (while meekly hoping that this increased
security doesn't damage our liberty) is to blind ourselves to the possibility of building real alternatives to the

authoritarian tendencies in contemporary politics. To situate ourselves against security politics


would allow us to circumvent the debilitating effect achieved through the constant securitising of social and political issues, debilitating in the sense
that 'security' helps consolidate the power of the existing forms of social domination and justifies the short-circuiting of
even the most democratic forms. It would also allow us to forge another kind of politics centred on a different

conception of the good. We need a new way of thinking and talking about social being and politics that moves us beyond security. This would
perhaps be emancipatory in the true sense of the word. What this might mean, precisely, must be open to debate.
But it certainly requires recognising that security is an illusion that has forgotten it is an illusion; it requires recognising that security is not the same as solidarity; it

requires accepting that insecurity is part of the human condition, and thus giving up the search for the certainty of
security and instead learning to tolerate the uncertainties, ambiguities and 'insecurities' that come with

being human; it requires accepting that 'securitizing' an issue does not mean dealing with it politically, but
bracketing it out and handing it to the state; it requires us to be brave enough to return the gift."'
ENDI 2010 74
Security K Wave 1

ALT SOLVES - VIOLENCE

Reject the affirmative to expose the their role in the genealogy of securitization. Problematizing
the aff allows new conceptions of security that do not rely on the elimination of difference
Der Derian 1998 [James, prof of political science at Brown, ―The Value of Security: Hobbes, Marx, Nietzsche, and
Baudrillard On Security,‖ ed. Ronnie Lipschutz, http://www.ciaonet.org/book/lipschutz/index.html]

What if we leave the desire for mastery to the insecure and instead imagine a new dialogue of security, not in the pursuit of
a utopian end but in recognition of the world as it is, other than us ? What might such a dialogue sound like? Any attempt
at an answer requires a genealogy: to understand the discursive power of the concept, to remember its forgotten
meanings, to assess its economy of use in the present, to reinterpret --and possibly construct through the reinterpretation--a late modern
security comfortable with a plurality of centers, multiple meanings, and fluid identities. The steps I take here in this direction are
tentative and preliminary. I first undertake a brief history of the concept itself. Second, I present the "originary" form of security that has so dominated our
conception of international relations, the Hobbesian episteme of realism. Third, I consider the impact of two major challenges to the Hobbesian episteme,
that of Marx and Nietzsche. And finally, I suggest that Baudrillard provides the best, if most nullifying, analysis of security in late modernity. In short, I
retell the story of realism as an historic encounter of fear and danger with power and order that produced four realist forms of security: epistemic, social,
interpretive, and hyperreal. To preempt a predictable criticism, I wish to make it clear that I am not in search of an "alternative security." An easy defense
is to invoke Heidegger, who declared that "questioning is the piety of thought." 9 Foucault, however, gives the more powerful reason for a
genealogy of security: I am not looking for an alternative; you can't find the solution of a problem in the solution of
another problem raised at another moment by other people. You see, what I want to do is not the history of solutions, and
that's the reason why I don't accept the word alternative . My point is not that everything is bad, but that everything is
dangerous, then we always have something to do . 10 The hope is that in the interpretation of the most pressing dangers of
late modernity we might be able to construct a form of security based on the appreciation and articulation rather than the
normalization or extirpation of difference.
ENDI 2010 75
Security K Wave 1

ALT SOLVES - SOVEREIGNTY

Rethinking the sovereign individual as the political subject unsettles the foundational claims of
IR’s security constructs. Discursive analysis is necessary to prevent a depoliticized technocracy.
Edkins 1999 [Jenny, Senior Lecturer in International Politics at the University of Wales Aberystwyth, Postructuralism and
International Relations: Bringing the Political Back In, p. xi-xii]

The rethinking of the political that is taking place in contemporary theory (and that has indeed been taking place for some time) involves an
unsettling of the view of the "subject" of politics. At one time the political subject was assumed to be the sovereign
individual, preexisting politics itself. This concept of the subject has been decentered and the notions of existence and temporality on which it
was founded problematized. The unsettling of the subject (of theory as well as of politics) has taken place in parallel with a freeing of the colonized
subject, albeit still within a postcolonial world, and a reexamination of boundaries of various kinds constructed to keep subjects in their place. The
challenge to international relations comes not only from a realignment and reexamination of subjectivity that leads to a rearticulation of fundamental
political questions but also from a reassessment of "the political" itself. If the unsettled subject can no longer be seen simply as friend or
enemy, what is "the political" about? If the boundary between the international and the domestic is insecure in more than
the traditional sense, can we still draw the line between politics within and anarchy without? Or is the political moment over once
the frontier is in place? As we shall see in Chapter 1, a reassessment of what we might mean by these terms leads a number of writers to make a
distinction between "politics" and "the political." It also leads to an analysis that acknowledges the importance of questions
of language, discourse, and ideology to a consideration of the political. Much of what we call "politics" is in many senses
"depoliticized" or technologized: the room for real political change has been displaced by a technology of expertise or the
rule of bureaucracy.
ENDI 2010 76
Security K Wave 1

ALT SOLVES - EPISTEMOLOGY

The alternative provokes an epistemological crisis within IR studies that puts the boundaries the
affirmative falsely draws in question
Ashley and Walker 1990 [Richard, Associate Professor of Political Science, Department of Political. Science, Arizona
State University, Tempe, RBJ, professor of political science at the University of Victoria, ―Conclusion: Reading
Dissidence/Writing the Discipline: Crisis and the Question of Sovereignty in International Studies,‖ International Studies
Quarterly, Vol. 34, No. 3, (Sep., 1990), pp. 367-416, jstor]

Our first step is to attend to the circumstances in which these critical readings occur. What occasions these critical readings? What prompts into action the
aesthetic labors they exemplify? Only by attending to this question can we render intelligible the problem of sovereignty to which these readings reply.
Only thus can we understand their strategic situation and what, as strategies, their aesthetic practices labor to do. At first the answer to these questions
would seem to be obvious. What prompts these critical readings is dissident works of thought, like those reflected in the present essays, issuing from the
margins of the discipline. This obvious answer, though, is insufficient. We need to know what it is about dissident works that prompts attention to them.
Why, put simply, should critical readers even care? The answer cannot be that dissident works of thought promise to provide a better
method, a superior framework, a more powerful way of producing more convincing answers and more certain solutions to
questions and problems that a discipline readily poses. These works eschew heroic promises such as these, and as we have seen,
their critics often indict them for the eschewal. There must be another answer. Our answer can be baldly stated: dissident works of thought
elicit attention and prompt critical readings because these works accentuate and make more evident a sense of crisis, what
one might call a crisis of the discipline of international studies. They put the discipline's institutional boundaries in question and put its familiar
modes of subjectivity, objectivity, and conduct in doubt; they render its once seem- ingly self-evident notions of space, time, and progress uncertain; and
they thereby make it possible to traverse institutional limitations, expose questions and difficulties, and explore political
and theoretical possibilities hitherto forgotten or deferred . In short, dissident works of thought help to accentuate a
disciplinary crisis whose single most pronounced symptom is that the very idea of "the discipline" enters thought as a
question, a problem, a matter of uncertainty.
ENDI 2010 77
Security K Wave 1

ALT SOLVES – METANARRATIVES

We should bring disorder to the concept of security in order to reject universalizing


metanarratives or single flawed epistemologies
Der Derian 1998 [James, prof of political science at Brown, ―The Value of Security: Hobbes, Marx, Nietzsche, and
Baudrillard On Security,‖ ed. Ronnie Lipschutz, http://www.ciaonet.org/book/lipschutz/index.html]

If security is to have any significance for the future, it must find a home in the new disorder through a commensurate
deterritorialization of theory. We can no longer reconstitute a single Hobbesian site of meaning or reconstruct some Marxist
or even neo-Kantian cosmopolitan community; that would require a moment of enlightened universal certainty that
crumbled long before the Berlin Wall fell. Nor can we depend on or believe in some spiritual, dialectical or scientific
process to overcome or transcend the domestic and international divisions , ambiguities, and uncertainties that mark the age of speed,
surveillance and simulation. This is why I believe the philosophical depth of Nietzsche has more to offer than the hyperbolic flash of Baudrillard. Can we
not interpret our own foreign policy in the light of Nietzsche's critique of security? As was the case with the origins of an ontotheological security, did not
our debt to the Founding Fathers grow "to monstrous dimensions" with our "sacrifices"--many noble, some not--in two World Wars? Did not our
collective identity, once isolationist, neutralist and patriotic, become transfigured into a new god, that was born and fearful of a nuclear, internationalist,
interventionist power? The evidence is in the reconceptualization: as distance, oceans and borders became less of a protective barrier to
alien identities, and a new international economy required penetration into other worlds, national interest became too weak
a semantic guide. We found a stronger one in national security , as embodied and institutionalized in the National Security Act of 1947, as protected
by the McCarran-Walter Act of 1952, and as reconstructed by the first, and subsequent National Security Council meetings of the second, cold war.
ENDI 2010 78
Security K Wave 1

ENVIRON SECURITY ALT - EXCLUDE SECURITIZATION

We can take action without securitizing non-military issues—the plan and the permutation
legitimize the social construction of danger
Waever 1998 [Ole, professor of International Relations at the Department of Political Science, University of Copenhagen,
―Securitization and Desecuritization,‖ On Security, ed. Ronnie Lipschutz,
http://www.ciaonet.org/book/lipschutz/index.html]

Finally, the approach I have proposed above points toward a study of the mechanisms leading to securitization of certain issues related to identity,
especially when and how these problems are handled, by society , in security terms. Such an approach implies that we have to take seriously
concerns about identity, but have also to study the specific and often problematic effects of their being framed as security
issues. We also have to look at the possibilities of handling some of these problems in nonsecurity terms, that is, to take on
the problems, but leave them unsecuritized. This latter approach recognizes that social processes are already under way
whereby societies have begun to thematize themselves as security agents that are under threat . This process of social
construction can be studied, and the security quality of the phenomenon understood, without thereby actually legitimizing
it. With the "as much security as possible" approach, this is hard to handle: one will have either to denounce such issues as not being security phenomena
("misperceptions"), or one will be pulled into the process as co-securitizer.
ENDI 2010 79
Security K Wave 1

FRAMEWORK: DISCOURSE SHAPES REALITY/POLICY

Security discourse shapes reality and policy


Lipschutz 1998 [Ronnie, prof of politics at UC Santa Cruz, On Security, ed. Ronnie Lipschutz,
http://www.ciaonet.org/book/lipschutz/index.html]

Conceptualizations of security--from which follow policy and practice--are to be found in discourses of security . These
are neither strictly objective assessments nor analytical constructs of threat, but rather the products of historical structures
and processes, of struggles for power within the state, of conflicts between the societal groupings that inhabit states and the
interests that besiege them. Hence, there are not only struggles over security among nations , but also struggles over
security among notions . Winning the right to define security provides not just access to resources but also the authority to
articulate new definitions and discourses of security, as well. As Karen Litfin points out, "As determinants of what can and
cannot be thought, discourses delimit the range of policy options, thereby functioning as precursors to policy outcomes. . .
. The supreme power is the power to delineate the boundaries of thought--an attribute not so much of specific agents as it is
of discursive practices." 15 These discourses of security, however clearly articulated, nonetheless remain fraught with
contradictions, as the chapters in this volume make clear.

AND, this outweighs their policy impacts-securitization happens within state discourses, not
between them—it is a precursor to the legitimation of state violence
Lipschutz 1998 [Ronnie, prof of politics at UC Santa Cruz, On Security, ed. Ronnie Lipschutz,
http://www.ciaonet.org/book/lipschutz/index.html]

What this process suggests is that concepts of security arise, to a great degree, out of discursive practices within states and, only
secondarily, among states. 17 Ole Wæver (Chapter 3: "Securitization and Desecuritization") illuminates this aspect of security, framing it not as an
objective or material condition, but as a "speech act," enunciated by elites in order to securitize issues or "fields," thereby helping to reproduce the
hierarchical conditions that characterize security practices. Thus, according to Wæver, much of the agenda of "redefining security" is a
process of bringing into the field of security those things that, perhaps, should remain outside (but this struggle to redefine a
concept can also be seen as an effort by heretofore-excluded elites to enter the security discourse). He warns, therefore, that redefining security in a
conventional sense, either to encompass new sources of threat or specify new referent objects, risks applying the
traditional logic of military behavior to nonmilitary problems. This process can also expand the jurisdiction of already-
expansive states as well. As Wæver puts it, "By naming a certain development a security problem, the `state' [claims] . .
. a special right [to intervene]." In intervening, the tools applied by the state would look very much like those used
during the wars the state might launch if it chose to do so. This contradiction was apparent in the initial landing of U.S. Marines in
Somalia in December, 1992. Demonstrably, there was a question of matching force to force in this case, but the ostensible goal of humanitarian assistance
took on the appearance of a military invasion (with the added hyperreality of resistance offered only by the mass(ed) media waiting on shore). This does
not mean that Wæver thinks that "security as a speech act" should not be applied to anything at all; only that it is necessary to consider with care what is
implied or involved if we are indiscriminate in doing so.

Discursive contexts determine the outcomes of policies


Lipschutz 1998 [Ronnie, prof of politics at UC Santa Cruz, On Security, ed. Ronnie Lipschutz,
http://www.ciaonet.org/book/lipschutz/index.html]

Security is, to put Wæver's argument in other words, a socially constructed concept: It has a specific meaning only within a specific social context. 18 It
emerges and changes as a result of discourses and discursive actions intended to reproduce historical structures and subjects
within states and among them. 19 To be sure, policymakers define security on the basis of a set of assumptions regarding vital
interests, plausible enemies, and possible scenarios, all of which grow, to a not-insignificant extent, out of the specific historical and
social context of a particular country and some understanding of what is "out there." 20 But, while these interests, enemies,
and scenarios have a material existence and, presumably, a real import for state security, they cannot be regarded simply as
having some sort of "objective" reality independent of these constructions . 21 That security is socially constructed does not
mean that there are not to be found real, material conditions that help to create particular interpretations of threats, or that such
conditions are irrelevant to either the creation or undermining of the assumptions underlying security policy. Enemies, in part, "create"
each other, via the projections of their worst fears onto the other; in this respect, their relationship is intersubjective. To the extent that they act
on these projections, threats to each other acquire a material character. In other words, nuclear-tipped ICBMs are not mere
figments of our imagination, but their targeting is a function of what we imagine the possessors of other missiles might do
to us with theirs . 22
ENDI 2010 80
Security K Wave 1

FRAMEWORK: DISCOURSE SHAPES REALITY

Our dissident analysis of the affirmative affects all political life


Ashley and Walker 1990 [Richard, Associate Professor of Political Science, Department of Political. Science, Arizona
State University, Tempe, RBJ, professor of political science at the University of Victoria, ―Conclusion: Reading
Dissidence/Writing the Discipline: Crisis and the Question of Sovereignty in International Studies,‖ International Studies
Quarterly, Vol. 34, No. 3, (Sep., 1990), pp. 367-416, jstor]

Introduction The essays in this collection, like so many of the texts that sustain them, speak from the margins. They are instances of increasingly visible
works of dissident thought proliferating in international studies today. Yet marginality and dissidence, upon being brought to the attention of a
discipline, invite a tried and all-too-familiar mode of interrogation and interpretation. They invite a strategy of reading and response that would assign to
these and other marginal works a location in a scholarly culture, a place and function in political life, a range of possibilities allowably explored, and a set
of standards by which their merits and claims of seriousness must be proven or shown to be lacking. This interpretive strategy deserves attention. For
what is at stake is not just the way in which the discipline receives these dissident works of thought-or puts them in their
place. Far beyond the matter of academic privilege, there is a question of considerable theoretical and practical import
involved. It is a question that the present essays take very seriously. It is also a question that resonates in all the far reaches of global
political life today, wherever and whenever time, space, and politi- cal identity are put in doubt and the territoriality of
modern being is uncertain. And yet it is a question to which the discipline must turn a deaf ear-which it must presume to be already answered-so
long as the interrogation of marginal and dissident events, including the present essays, is controlled by a certain strategy of reading and response. In a
word, it is the question of sovereignty. In offering an essay to close this issue, we do not try to bring the question of sovereignty to a close. We do
not pretend to gather up and express an implicit consensus among contributors as to how the question of sovereignty might or should be answered. No
such consensus exists. Indeed, if the present essays exhibit anything resembling agreement on the question of sovereignty, it is only that it must be
regarded as just that, a question. In contrast to the vast preponderance of writings appearing in the Quarterly over the years, the essays in these pages share
a suspicion of all assertions of sovereign privilege, and they assert none of their own. These essays do not presume to speak a sovereign voice, a voice
beyond politics and beyond doubt, a voice of interpretation and judgment from which truth and power are thought to emanate as one. Instead, their
marginality consists in their disposition to maintain their distance from all presumptively sovereign centers of interpretation and judgment. Their
dissidence consists in their readiness to regard every historical figuration of sovereign presence-be it God, nature, dynasty, citizen, nation, history,
modernity, the West, the market's impartial spectator, reason, science, paradigm, tradition, man of faith in the possibility of universal human community,
common sense, or any other-as precisely a question, a problem, a contingent political effect whose production, variations, and possible undoing merit the
most rigorous analysis. In this concluding essay, we shall explore some of the implications of this dissident attitude, this insistence on regarding
sovereignty as a question. These implications are far-reaching, connecting the immediate reality of disciplinary crisis not only to
all the unsettled zones of global political life today but also to all those historical in- stances of cultural crisis to which the
"great texts" of the discipline have replied. There is, though, another task. If we are to explore the implications of the present essays'
attitudes toward the question of sovereignty, we need to address ourselves to the strategy of interpretation invited by dissident
works of thought at the margins of the discipline. We need to understand how this strategy works and what it does. In
particular, we need to understand that it labors to produce a silence on the question of sovereignty-a silence that always marks the time and place that
sovereignty would be.
ENDI 2010 81
Security K Wave 1

DISCOURSE SHAPES REALITY – METAPHOR

Security metaphors shape reality- they determine how we structure images of threats and take
actions to eliminate them
Mutimer 2000 [David, associate professor of political science at York University and Deputy Director of the Center for
International and Security Studies, The Weapons State, pg 19-20]

It is not entirely common to think that metaphor has much to do with the making of policy in general and of security policy
in particular. Security policy concerns the serious matter of war; its subject is troops, not tropes. Nevertheless, it would seem
even policymakers bent on waging war recognize the occasional utility of an apt metaphor . Hidden in a footnote is a report by Chris
Hables Gray on a small change in the language surrounding the war in the Gulf: "Originally, the attack on Iraq and occupied Kuwait was to be called
Desert Sword, but it was decided to portray the war as more of a natural force."22 Gray's contention rings true, as Desert Sword fits more obviously with
the prior operation, Desert Shield, than does Desert Storm. Somebody in the Pentagon, however, recognized that swords are wielded by hands whose
owners can then be held responsible; storms are acts of nature or of God, not of people. Although the clear intention of this use of metaphor is political in
the narrowest sense-we might even say it is meant as public relations-the means by which metaphors function is independent of such intention. Swords and
storms carry different meanings; that is, they have different entailments and as such shape a labeled object, such as a military action, in different ways.23
Paul Chilton recently used metaphor as an analytic starting point to examine the heart of Cold War security discourse. In the conclusion to Security
Metaphors, Chilton explains how metaphor relates to policy: Metaphor is an element in the discourse of policymaking; it does
not drive policy.... It would be absurd to reduce the Cold War to the influence of metaphor. However, both cognitive
analysts of policymaking and historians of the Cold War have noted the part played by analogical reasoning and by
metaphor. Whatever distinctions might be drawn between the two terms "analogy" and "metaphor," they can both be treated as manifestations of the
cognitive process whereby one thing is seen in terms of another.24 The common understanding of metaphor is that it is a literary
technique, allowing an author to provide descriptive depth and allegorical commentary by establishing a relationship between two separate objects' or
ideas. Chilton argues that metaphors are much more than this, that metaphor is "an indispensable ingredient of thought
itself."25 Policymakers address problems by means of what I have called images-that is, the student or policymaker constructs
an image of a problem, of an issue, or even of other actors. This image relates the thing imagined to another, in terms of
which the first is understood. This act of relation is crucial both to understanding and to the scholarly act of interpretation that follows. Metaphors
compose the images used to structure and support our understanding of a problem and therefore our respouse to that problem. The choice of Desert Storm
over Desert Sword is designed to foster political support for a policy problem by imagining the operation in terms of a force of nature it would be
nonsensical to oppose. We might decry the devastation caused by weather, but we would look a bit foolish marching on Washington to bring an end to
hurricanes. The general relationship~among the image of a policy problem, the condition of the problem itself, and the policy solution to that problem-
however, allow these ideas to be given a much wider scope than they would receive as a form of public relations. In Security Metaphors Chilton provides a
detailed and rigorous examination of the role of metaphor in Cold War security. Specifically, he explores the way in which three metaphors were central to
the understandings that gave rise first to the Cold War and later to its end. He looks first at how the metaphor of security and then the related metaphor of
containment emerged from attempts within the U.S. state to make sense of the postwar era. In the final part of his book, Chilton turns to the end of the
Cold War and to the place of architectural metaphors, particularly the common house, in producing the Cold War's end. The metaphors of security,
containment, and the common house did more than simply support a policy choice; they structured the way in which we
can think about problems and thus shape that choice in the first place.
ENDI 2010 82
Security K Wave 1

FRAMEWORK: SECURITY = SPEECH ACT

Security is a speech act that shapes reality—its evocation gives the state unending permission to
secure itself against existential challenges
Waever 1998 [Ole, professor of International Relations at the Department of Political Science, University of Copenhagen,
―Securitization and Desecuritization,‖ On Security, ed. Ronnie Lipschutz,
http://www.ciaonet.org/book/lipschutz/index.html]

we can regard "security" as a speech act . In this usage, security is not of interest
What then is security? With the help of language theory,
as a sign that refers to something more real; the utterance itself is the act. By saying it, something is done (as in betting,
giving a promise, naming a ship). 23 By uttering "security," a state-representative moves a particular development into a specific
area, and thereby claims a special right to use whatever means are necessary to block it. 24 The clearest illustration of this
phenomenon--on which I will elaborate below--occurred in Central and Eastern Europe during the Cold War , where "order" was
clearly, systematically, and institutionally linked to the survival of the system and its elites. Thinking about change in East-
West relations and/or in Eastern Europe throughout this period meant, therefore, trying to bring about change without
generating a "securitization" response by elites, which would have provided the pretext for acting against those who had
overstepped the boundaries of the permitted. Consequently, to ensure that this mechanism would not be triggered, actors had
to keep their challenges below a certain threshold and/or through the political process--whether national or international--have the threshold
negotiated upward. As Egbert Jahn put it, the task was to turn threats into challenges; to move developments from the sphere of existential fear to one
where they could be handled by ordinary means, as politics, economy, culture, and so on. As part of this exercise, a crucial political and theoretical issue
became the definition of "intervention" or "interference in domestic affairs," whereby change-oriented agents tried, through international law, diplomacy,
and various kinds of politics, to raise the threshold and make more interaction possible. Through this process, two things became very clear. First,
the word "security" is the act ; the utterance is the primary reality. Second, the most radical and transformational perspective-
-which nonetheless remained realist--was one of minimizing "security" by narrowing the field to which the security act was
applied (as with the European détente policies of the 1970s and 1980s). After a certain point, the process took a different form and the aim became to
create a speech act failure (as in Eastern Europe in 1989). Thus, the trick was and is to move from a positive to a negative meaning:
Security is the conservative mechanism--but we want less security! Under the circumstances then existing in Eastern
Europe, the power holders had among their instruments the speech act "security." The use of this speech act had the effect
of raising a specific challenge to a principled level, thereby implying that all necessary means would be used to block that
challenge. And, because such a threat would be defined as existential and a challenge to sovereignty, the state would
not be limited in what it could or might do . Under these circumstances, a problem would become a security issue whenever so
defined by the power holders. Unless or until this operation were to be brought to the point of failure--which nuclear conditions made rather
difficult to imagine 25 --available avenues of change would take the form of negotiated limitations on the use of the "speech act security." Improved
conditions would, consequently, hinge on a process implying "less security, more politics!"
ENDI 2010 83
Security K Wave 1

FRAMEWORK: SECURITY = SPEECH ACT

Securitization is a speech act--acceptance maintains the politics securitization – rejection allows


for a deconstruction
Ivarsson, 2006 [Niclas, Head of Department of Political Science: Peace and Conflict Studies at Lund Univerity. Health and
Security: HIV/AIDS in Post-Apartheid South Africa, p.6-7, http://theses.lub.lu.se/archive/2007/05/30/1180512652-29760-
128/Uppsatsen.PDF]

The process of securitization is often compared by Buzan et al. (1998) to the speech act of theoretical linguistics. A speech act is not
referring to an actual event. Instead it is the speech act itself that is the event. This can be compared to giving a promise to someone. Unless
you say it there is no promise. It is the act of giving the promise that is the actual event or act. For something to be security,
uttering the issue and the word security in the same sentence will simply not do (Buzan et al. 1998 s 26f). It is only when an
issue is presented with the logic and grammar of the speech act that we can talk about securitization. For this we need four
components ―(i) securitizing actors (such as political leaders, intelligence experts, etc.), declaring (ii) a referent object (such
as a state) to be (iii)existentially threatened (e.g., by an imminent invasion), and who make a persuasive call for the
adoption of (iv) emergency measures to counter this threat(e.g., declare war or impose a curfew)‖ (Elbe 2006 p 125f).Buzan et al.
claim that security is always about the future and therefore hypothetical and about counterfactuals. What will happen if we take action and what will
happen if we do not? Security is an arena were objective standards are practically impossible to apply. Even if tanks were to
rush over your border you cannot be sure if it is a threat or not unless you know the socially constituted relationship
between the tank and the referent object. Example, the tanks can be hostile but also part of peace-keeping force. For a
securitization to take place the emergency measures adopted to counter the existential threat must be accepted by the
referent object. If it (read they) do not accept and tolerate these emergency measures that under normal circumstances would
be illegitimate there is no securitization. The final component that is needed for a successful securitization is an acceptance
from the people. Securitization is therefore essentially an intersubjective process ―as with all politics‖ (Buzan et al. 1998 p 30f). The rhetoric of
securitization would therefore sound something like ―if we don‘t take action against this threat now everything else will become unimportant‖. Obviously
meaning that there will not be anything left as this threat threatens our very existence. It is important to underline that this in itself is only a securitizing
move not a securitization. A securitization does not require that extreme measures are actually taken but that the argumentation
for a securitization has created a platform from which it would be possible to legitimize emergency measures that earlier
would not have been possible (Buzan et al. 1998 p 25). According to all of this there is no such thing as objective security. However certain events
can aid the securitization process as facilitating conditions. Securitization is more likely to succeed on the state and nation level. This because it is on these
levels we find the strongest collective identities. If a securitizing actor can convince the referent object that their identity, what makes
them who they are, is threatened a securitization process is likely to succeed. This is why we rarely see successful
securitization on the global level as it is very difficult to unite all of mankind to perceive a threat in the same way (Buzan et al.
1998 p 23)
ENDI 2010 84
Security K Wave 1

FRAMEWORK: DISCOURSE 1ST

Securitization is a speech act: the affirmative’s speech act should be rejected prior to
consideration of the desirability of their policy
Elbe 2006 [Stefan, Senior Lecturer in International Relations at the University of Sussex, March, INTERNATIONAL
STUDIES QUARTERLY, p. 124]

By way of extension, for Buzan, Wæver, and de Wilde, labeling an issue a security issue also constitutes such a performative speech
act. For them (1998:26) security ‗‗is not interesting as a sign referring to something more real; it is the utterance itself that is the
act. By saying the words, something is done (like betting, giving a prom- ise, naming a ship).‘‘ Security is thus not viewed by these
three scholars as something that exists independently of its discursive articulation,13 but rather as a particular form of
performative speech act; security is a social quality political actors, such as intelligence agencies, government officials, and
international organizations, inject into issues by publicly portraying them as existential threats (Buzan, Wæver, and de Wilde
1998:204). Whereas more traditional approaches to security operate within a specific definition of security, revolving for example
around the deployment of armed force in world politics, and then seek to ascertain empirically whether an issue genuinely represents a
security threat, for securitization theory the designation of an issue as a security threat is primarily an intersubjective
practice undertaken by security policy makers. ‗‗It is a choice to phrase things in security . . . terms, not an objective feature
of the issue . . . .‘‘ (Buzan, Wæver, and de Wilde 1998:211); or, as Wæver (1995:65) put it elsewhere, the ‗‗[u]se of the security label does
not merely reflect whether a problem is a security problem, it is also a political choice, that is, a decision for
conceptualization in a special way.‘‘ The leader of a political party, for example, can choose whether to portray immigration as a security issue or
as a human rights issue. Similarly, leaders of international organizations can choose whether they portray HIV/AIDS as a health issue, as a development
issue, or, as they have done more recently, as an international security issue.
ENDI 2010 85
Security K Wave 1

FRAMEWORK: AT: RATIONAL ACTOR

Their model of a "rational actor" relies on specific positivist epistemologies- this makes policy
making useless
Marston, 2004, Bachelor of Social Science (QUT), PHD (UQ) Social policy and Discourse Analysis , p. 14-15
The positivist paradigm informs an idealized rational actor understanding of the policy-making
process. The rational approach to policy-making is an extension of particular forms of positivism and
neo-positivism that seek to purge the social scientist of values (Bryman, 1988, p.14). This idea of reason without
values is maintained through instrumental and technical rationality. Instrumental rationality in
policy-making can be defined as follows: 'in any organization there might be a number of ways of reaching goals; and when
faced with the need to make a choice between alternatives the rational decision maker chooses the alternative most likely to
achieve the desired outcome' (Ham and Hill, 1993, p.77). The idealistic representation of policy as a form of
'rational decision making' between available choices and options is problematic for a
number of reasons. The limitations of rational approaches to policy-making arise from an insufficient
account of the political context, insufficient emphasis on the participants in the process
(and their conflicting interests) and the 'ideal type' nature of the models themselves (Dalton et al, 1996, p.17). A
positivist view of policy-making asserts policy solutions as universal truths waiting to be
discovered by the so-called policy 'expert'. Hillyard and Watson (1996, p.324) argue that this perception
denies the constitutive role of discourse. In short, a positivist epistemology is not an adequate
position for researchers and policy analysts aiming to explore and understand how policy
meanings are discursively constructed, how regulatory functions of the state are being
transformed and how policy actors represent and articulate policy problems and solutions.
By focusing on 'objective' outcomes and grand narratives of 'progress', 'rationality' and 'truth', we
remain blind to the multifaceted nature of policy-making processes. Positivist accounts of the
social world do not recognize the constructive nature of discursive processes that produce knowledge
and identities, or how conflict over policy meanings is manifested within specific policy
environments. While not denying the place of positivist informed research in social planning, this paradigm is
limited when it comes to understanding questions of power as experienced in the production,
reproduction and transformation of policy agendas. As Yanow (1996, p.6) argues. 'positivist knowledge does
not give us information about meanings made by actors in a situation. When we read a policy we see more than just marks on
a page. we hear more than just sound waves'. Exploring the discursive dimensions of policy-making
requires alternative theoretical frameworks and epistemologies that are able to capture the
processes of subjectification and the relationship between agency, identity and discourse in local
policy contexts. The various strands of critical social theory and post-structuralism are areas of theorizing that offer social
policy researchers different ways of thinking about language and culture.
ENDI 2010 86
Security K Wave 1

AT: NO IMPACT TO REPRESENTATION

Representations aren’t harmless; they shape policies and cause the harms the aff attempts to
alleviate.
Doty, 1996 [Roxanne Lynn, assistant professor of political science at Arizona state university, Imperial Encounters, p. 5-
6]

This study begins with the premise that representation is an inherent and important aspect of global political life and therefore a critical and legitimate area
of inquiry. International relations are inextricably bound up with discursive practices-that put into circulation representations that are taken as "truth." The
goal-of-analyzing these practices is not to reveal essential truths that have been obscured, but rather to examine bow certain representations
underlie the production of knowledge and, identities and how these representations make various courses of action possible.
AS Said (1979: 21) notes, Mere is no such thing as a delivered presence, but there is a re-presence, or representation. Such an assertion does not
deny the existence of the material world, but rather suggests that material objects and subjects are constituted as such within
discourse. SO, for example, when U.S. troops march into Grenada, this is certainly "real : though the march of troops across a piece of
geographic space is in itself singularly uninteresting and socially irrelevant outside of the representations that produce meaning. It is only when
"American" is attached to the troops and "Grenada‖ to the geographic space that meaning is created. What the physical
behavior itself is, though, is still far from certain until discursive practices constitute it as an "invasion ; a 'show of force,"
"training exercise, ―a "rescue, ―and SO on. What is "really" going on in such a situation is inextricably linked to the discourse within
which it is located. To attempt a neat separation between discursive and nondiscursive practices, understanding the former
as purely linguistic, assumes a series of Dichotomies – thought/reality appearance essence, mind matter, word/world,
subjective/objective - that a critical genealogy calls into Question. Against this, the perspective taken here affirms the material and
performative character of discourse. 'In suggesting that global politics, and specifically the aspect that has to do with relations between the North and the
South, is linked to representational practices 1 am suggesting that the issues and concerns that constitute these relations occur within a 'reality' whose
content has for the most part been defined by the representational practices of the ‗first world'. Focusing on discursive practices enables one to
examine how the processes that produce "truth" and "knowledge" work and how they are articulated with the exercise of
political, military, and economic power
ENDI 2010 87
Security K Wave 1

AT: PERM- POSITIVISM

Our positivism links outweigh- buying into false scientific epistemology blocks the efficacy of our
criticism
Ashley 1984 [Richard, professor of political science at Arizona State University, ―The Poverty of Neorealism,‖
International Organization, Vol. 38, No. 2 (Spring, 1984), pp. 225-286, jstor]

Despite the contradiction between neorealists' util- itarian conception of politics and their statist commitments, neorealists
are able to perpetuate the state-as-actor illusion in their conception of the inter- national system. They are able to do so
because, as positivists, we are meth- odologically predisposed to look for precisely the kind of model they "reveal."
Without an actor model, we somehow sense, we shall lack any scientific point of entry into a meaningful understanding of
the international system; the system will appear to us, we worry, as a meaningless swirl of "disembodied forces." They are
further able to do so because, as positivists, we join them in excluding from the realm of proper scientific discourse
precisely those modes of criticism that would allow us to unmask the move for what it is. At the very moment we
begin to question this state-as-actor conception, we are given to feel that we have stumbled beyond the legitimate grounds
of science, into the realm of personal ethics, values, loyalties, or ends. We are given to feel that our complaints have no
scientific standing. And so, as scientists, we swallow our questions. We adopt the posture of Waltz's utter detachment,
Gilpin's fatalism, Krasner's wonderment, or Keohane's We- berian resignation with respect to the powers that be. We might
not like it, we say, but this is the world that is. As scientists, we think we cannot say otherwise.
ENDI 2010 88
Security K Wave 1

AT: PERM

The perm ensnares us in the trap of securitization by treating it as an ordering principle of the
political order
Dillon 1996 (Michael is a professor of politics at the University of Lancaster, Politics of Security, p. 16)
Hence we are not only users of language, we are used, the genealogist would argue, by the language we use. We are not simply the people who
employ discourses of security, we are the people who are ensnared in and used by them. Just as there therefore could be no hisotry of
security without a history of the (inter)national politics that seeks to define, pursue, and prosecute order under the various names of security, so also any
individual political transformation would manifest its own particular order of fear. Don‘t ask what a people is, the genealogist of security might sat, ask
how an order of fear forms people. And, in particular, bearing the imprint of the wat determinations of what is political have originated in fear, s/he would
emphasize that security is a principal device for constituting political order and for confing political imagination within the laws of
necessity of the specific rationalities thrown-up by their equally manifold discourses of danger

Searching for a middle ground obscures both epistemological and ontological investigations into
the foundations of securitization
Patomaki and Wight 2000 [Heikki, Professor of International Relations at the University of Helsinki, Colin, professor of
political science at University of Wales, ―After Postpositivism? The Promises of Critical Realism,‖ International Studies
Quarterly, Vol. 44, No. 2 (Jun., 2000), pp. 213-237, jstor]

A synthesis based on two problematic metaphysical systems produces only a synthesis of two problematic metaphysical
positions-not an improved metaphysical position. The problem is how to move forward? How do we move beyond a sterile and debilitating
debate where one side chastises the other for its naive belief in a world "out there," while the other berates its mirror image for making the world "all in
here" and all the while a third position claims legitimacy in terms of its "middle-groundedness." Given that the debate, as currently framed,
tends to be primarily epistemological perhaps a more ontological focus could facilitate a move forward . This is not to say that
ontological considerations do not play a role in current understandings, but we argue that where they have played a role these ontological issues have been
based on epistemological considerations. In this respect we want to reverse a long-standing Western philosophical dogma; that of the privileging of
epistemological questions over ontological ones. Indeed, we think that when viewed from an ontological perspective current
understand- ings of IR take on an altogether different hue. Any attempt to locate oneself in the centre of current
epistemological debates without considering the ontological problematic risks duplicating the worst of both extremes. It is
not simply a scientific ontology we mean here, as in theo- retical disagreements over whether states are the most important
actors, for example. What we mean by ontology is a philosophical ontology; an inquiry into which is logically prior to the
development of any scientific or social ontology (Bunge, 1996). It is here that we think that the philosophy known as critical realism can
be of benefit to IR scholars (for some of the key texts see Archer et al., 1998).1 We suggest that critical realism can incorporate many of
the recent epistemological developments and at the same time move the debate forward due to its focus on ontological
matters. Critical realism highlights the conditions of possibility for a resolution of many of the theoretical, methodological,
and praxiological cul-de- sacs international relations theory currently finds itself in. From a critical realist perspective and contrary to
the dominant understandings within IR theory, the boundaries of negativity and boredom are not diametrically opposed, but share much in common. The
key to any move forward is not simply to take the middle ground, but to engage with and challenge the extremities
that constitute the conditions of possibility for a certain understanding of the middle ground. This can only be achieved
through an examination of the boundaries of boredom and negativity, or better, the theory "problem-field" within which they are constituted. Here lies
one of the benefits of metatheoretical inquiry to IR. In this piece we wish to engage in just such a metatheoretical investigation in the hope of
throwing some light on some of the important contemporary problems facing IR scholars. First, we locate a common structure to both the boundary of
boredom and the boundary of negativity. In this section we aim to show how both are embedded upon a discourse of philosophical anti-realism. Second,
we attempt to show, through the philosophies of David Hume and Immanuel Kant, how this anti- realism constitutes what we call the "problem-field" of
IR, a "problem-field" that, we argue, serves to construct a particular understanding of IR theory with a very particular and restricted understanding of its
own possibilities. Third, we develop a very brief account of our proposed alternative, critical realism. And fourth, we try to show the difference that
critical realism might make to a more ontologi- cally attuned IR. In this section we argue against the incommensurability thesis and in favor of
epistemological pluralism and opportunism. We try to revive causal theorizing by redefining causality in realist terms and by arguing that both meaningful
reasons and social structures are causally efficacious. Drawing on this analysis we discuss the agent-structure problem and suggest how the social world
can be decomposed into causal and ontological elements. We also challenge what we consider to be the misleading manner in which IR theory currently
understands the levels of analysis problem. Finally, we indicate how critical real- ism has also normative implications for the study of IR.
ENDI 2010 89
Security K Wave 1

AT: CASE OUTWEIGHS

There is no status quo- the 1AC advantages are just random factoids politically constructed to
make the plan appear to be a good idea. They merely take a snapshot of a dynamic status quo and
attempt to portray it as a static universality. The impact is that solvency is a rigged game-
construction of the advantages presupposes the necessity of the plan-risk assessment means you
vote negative to avoid error replication
Dillon and Reid, 2000 [Michael, Julian, Global Governance, Liberal Peace, and Complex Emergency. By: Dillon, Michael, Reid, Julian, Alternatives:
Global, Local, Political, 03043754, Jan-Mar, Vol. 25, Issue 1 ]
More specifically, where there is a policy problematic there is expertise, and where there is expertise there, too, a policy problematic will emerge. Such problematics are detailed and elaborated in terms of discrete
forms of knowledge as well as interlocking policy domains. Policy domains reify the problematization of life in certain ways by turning these epistemically and politically
contestable orderings of life into "problems" that require the continuous attention of policy science and the continuous
resolutions of policymakers. Policy "actors" develop and compete on the basis of the expertise that grows up around such problems or clusters of
problems and their client populations. Here, too, we may also discover what might be called "epistemic entrepreneurs." Albeit the market for discourse is prescribed and policed
in ways that Foucault indicated, bidding to formulate novel problematizations they seek to "sell" these, or otherwise have them officially adopted. In principle, there is no limit to the ways in which the management
of population may be problematized. All aspects of human conduct, any encounter with life, is problematizable. Any problematization is capable of becoming a policy
problem. Governmentality thereby creates a market for policy, for science and for policy science, in which problematizations go
looking for policy sponsors while policy sponsors fiercely compete on behalf of their favored problematizations. Reproblematization of problems is constrained by
the institutional and ideological investments surrounding accepted "problems," and by the sheer difficulty of challenging the
inescapable ontological and epistemological assumptions that go into their very formation. There is nothing so fiercely contested as an
epistemological or ontological assumption. And there is nothing so fiercely ridiculed as the suggestion that the real problem with problematizations
exists precisely at the level of such assumptions. Such "paralysis of analysis" is precisely what policymakers seek to avoid since they
are compelled constantly to respond to circumstances over which they ordinarily have in fact both more and less control than
they proclaim. What they do not have is precisely the control that they want. Yet serial policy failure--the fate and the fuel of all policy--
compels them into a continuous search for the new analysis that will extract them from the aporias in which they constantly find
themselves enmeshed.[ 35] Serial policy failure is no simple shortcoming that science and policy --and policy science--will ultimately
overcome. Serial policy failure is rooted in the ontological and epistemological assumptions that fashion the ways in which
global governance encounters and problematizes life as a process of emergence through fitness landscapes that constantly adaptive and changing ensembles have continuously to
negotiate. As a particular kind of intervention into life, global governance promotes the very changes and unintended outcomes that it then serially reproblematizes in terms of policy failure. Thus, global
liberal governance is not a linear problem-solving process committed to the resolution of objective policy problems simply by
bringing better information and knowledge to bear upon them. A nonlinear economy of power/knowledge, it deliberately
installs socially specific and radically inequitable distributions of wealth, opportunity, and mortal danger both locally and globally
through the very detailed ways in which life is variously (policy) problematized by it ..
ENDI 2010 90
Security K Wave 1

AT: CASE OUTWEIGHS

Rational impact assessment goes negative- affirmative scenarios are Orwellian disaster porn.
Endorsing the alternative entails categorically less risk than constant mindless promotion of
militarism and enemy creation
Rule, 2010 [James B, PhD Harvard, MA Oxford, BA Brandeis, The Military State of America and the Democratic Left, Dissent Vol. 57 No 1, Winter]

At this moment, for example,


in 1 984 (if it was 1984), Oceania was at war with Eurasia and in alliance with Eastasia. In no public or
private utterance was it ever admitted that the three powers had at any time been grouped along different lines. Actually, as Winston
well knew, it was only four years since Oceania had been at war with Eastasia and in alliance with Eurasia. But that was merely a piece of furtive knowledge
which he happened to possess because his memory was not satisfactorily under control. We are not there, but the direction of movement is
unmistakable. As the Iraq adventure has demonstrated, shrewd state manipulation of strategic information makes it possible to defuse
criticism and discredit public skepticism, until it is too late. Many trends since Orwell's lifetime have aggravated the hazards
that he anticipated. One is globalism - the growth of an ever-moretightly connected world, so that people, ideas, technologies, and even
weapons move about the earth more and more readily. Such conditions can facilitate terrorism, conceivably on a scale well beyond what the world
has yet witnessed. On the state side of the equation, we see the rise of vast bureaucracies dealing in essentially secret knowledge -
intelligence about military matters and a host of other subjects held vital to national security, yet supposedly too sensitive for
public disclosure. Mobilization of such knowledge in turn requires a high-tech establishment of civilian and military
experts whose activities cannot readily be monitored by outsiders. One result is that government claims about matters of vital
public concern, from weapons of mass destruction to terrorist dangers, are not easily challenged in public debate. As Orwell
warned, the state may change the menu of deadly enemies from year to year but continue the same strictures on public inquiry
and dissent. A few decades ago, Iraq was America's ally; more lately, it reappeared as part of the axis of evil . China rises and falls
in Washington's official designations - sometimes a feared twenty-first-century competitor, more recently an ally in the quest for Asian "stability" and
indispensable supporter of the U.S. economy. Pakistan under its last dictator was a stalwart participant in the so-called War on Terror. But that country could
any day be redefined (with some justification) as a threat to the civilized world. Who can say with confidence what demonic qualities will be
ascribed, perhaps quite accurately, to any of America's present-day allies, with the next shake of this country's foreign-policy
kaleidoscope? And who can say what new military exploits, or domestic restrictions, will be proclaimed essential to repress
these demons of the future? The one thing we can be sure of is that the supply of ugly movements and regimes around the
world shows no sign of running short. If their sheer presence suffices to justify a hypermilitarized America and concomitant
suppression of countervailing voices in domestic life, we are embarked on a long journey in the direction of 1984. There has
to be a better way - as we on the democratic Left should be the first to proclaim. In a dangerous world, any course of action bears risks. No
one can absolutely rule out the possibility that a steady diet of aggressive American military action abroad might forestall
disasters yet unseen. Nor can anyone deny that relentless surveillance of domestic communications, or invocation of national
security to rebuff all challenges to the exercise of government power could, conceivably, help block further terrorist acts on U.S.
soil. But nor, for that matter, can anyone authoritatively deny that such measures might actually make matters much
worse. Political programs are defined as much by the risks they are willing to accept as by the values they seek to promote.
The democratic Left properly welcomes the risks of broader and deeper democracy, at home and abroad. It counsels more
government openness and broader public engagement in governance, even while acknowledging that these things can go wrong. It
seeks to build, however incrementally, supranational structures of authority and conflict-resolution - as against reliance on unilateral intimidation and worse. It
refuses to let American fixation on worldwide dominance to serve as an excuse for not building a strong nation at home - that
is, for neglecting health, employment, environmental responsibility, and education. We on the democratic Left must be quick to
take risks on behalf of these ends - because the alternative risks of endless, deadly international conflict and narrowing
attention to domestic well-being are far more alarming.
ENDI 2010 91
Security K Wave 1

AT: PREDICTIONS/SCENARIO PLANNING GOOD

Worst case predictions cause worst case policy making- recognition of our ignorance makes us
more secure then their fatalistic scenario planning
Schneier 2010 [Bruce, internationally renowned security technologist and author, MA CS American Univ. 3-13
http://www.schneier.com/blog/archives/2010/05/worst-case_thin.html

At a security conference recently, the moderator asked the panel of distinguished cybersecurity leaders what their nightmare scenario was. The answers were the
predictable array of large-scale attacks: against our communications infrastructure, against the power grid, against the financial system, in combination with a
physical attack. I didn't get to give my answer until the afternoon, which was: "My nightmare scenario is that people keep talking about their nightmare
scenarios." There's a certain blindness that comes from worst-case thinking. An extension of the precautionary principle, it involves
imagining the worst possible outcome and then acting as if it were a certainty. It substitutes imagination for thinking,
speculation for risk analysis, and fear for reason . It fosters powerlessness and vulnerability and magnifies social paralysis.
And it makes us more vulnerable to the effects of terrorism. Worst-case thinking means generally bad decision making for several
reasons. First, it's only half of the cost-benefit equation. Every decision has costs and benefits, risks and rewards. By speculating about what
can possibly go wrong, and then acting as if that is likely to happen, worst-case thinking focuses only on the extreme but
improbable risks and does a poor job at assessing outcomes . Second, it's based on flawed logic. It begs the question by
assuming that a proponent of an action must prove that the nightmare scenario is impossible. Third, it can be used to support any
position or its opposite. If we build a nuclear power plant, it could melt down. If we don't build it, we will run short of power
and society will collapse into anarchy. If we allow flights near Iceland's volcanic ash, planes will crash and people will die. If we don't, organs won‘t
arrive in time for transplant operations and people will die. If we don't invade Iraq, Saddam Hussein might use the nuclear weapons he
might have. If we do, we might destabilize the Middle East, leading to widespread violence and death. Of course, not all fears are equal. Those
that we tend to exaggerate are more easily justified by worst-case thinking. So terrorism fears trump privacy fears, and
almost everything else; technology is hard to understand and therefore scary; nuclear weapons are worse than conventional weapons; our children need to
be protected at all costs; and annihilating the planet is bad. Basically, any fear that would make a good movie plot is amenable to worst-case thinking. Fourth
and finally, worst-case thinking validates ignorance. Instead of focusing on what we know, it focuses on what we don't know -- and
what we can imagine. Remember Defense Secretary Rumsfeld's quote? "Reports that say that something hasn't happened are always interesting to me, because
as we know, there are known knowns; there are things we know we know. We also know there are known unknowns; that is to say we know there are some
things we do not know. But there are also unknown unknowns -- the ones we don't know we don't know." And this: "the absence of evidence is not evidence
of absence." Ignorance isn't a cause for doubt; when you can fill that ignorance with imagination, it can be a call to
action. Even worse, it can lead to hasty and dangerous acts. You can't wait for a smoking gun, so you act as if the gun is about to
go off. Rather than making us safer, worst-case thinking has the potential to cause dangerous escalation. The new
undercurrent in this is that our society no longer has the ability to calculate probabilities. Risk assessment is devalued.
Probabilistic thinking is repudiated in favor of "possibilistic thinking": Since we can't know what's likely to go wrong, let's speculate about what
can possibly go wrong. Worst-case thinking leads to bad decisions, bad systems design, and bad security. And we all have direct experience
with its effects: airline security and the TSA, which we make fun of when we're not appalled that they're harassing 93-year-old women or keeping first graders
off airplanes. You can't be too careful! Actually, you can. You can refuse to fly because of the possibility of plane crashes. You can lock your children in the
house because of the possibility of child predators. You can eschew all contact with people because of the possibility of hurt. Steven Hawking wants to avoid
trying to communicate with aliens because they might be hostile; does he want to turn off all the planet's television broadcasts because they're radiating into
space? It isn't hard to parody worst-case thinking, and at its extreme it's a psychological condition. Frank Furedi, a sociology professor at the
University of Kent, writes: "Worst-case thinking encourages society to adopt fear as one of the dominant principles around
which the public, the government and institutions should organize their life. It institutionalizes insecurity and fosters a mood of
confusion and powerlessness. Through popularizing the belief that worst cases are normal, it incites people to feel defenseless
and vulnerable to a wide range of future threats." Even worse, it plays directly into the hands of terrorists, creating a population
that is easily terrorized -- even by failed terrorist attacks like the Christmas Day underwear bomber and the Times Square SUV bomber. When someone is
proposing a change, the onus should be on them to justify it over the status quo. But worst-case thinking is a way of looking at the world that
exaggerates the rare and unusual and gives the rare much more credence than it deserves. It isn't really a principle; it's a cheap
trick to justify what you already believe. It lets lazy or biased people make what seem to be cogent arguments without
understanding the whole issue. And when people don't need to refute counterarguments, there's no point in listening to them.
ENDI 2010 92
Security K Wave 1

AT: REALISM INEVITABLE

Realism isn‘t natural or inevitable—it has to be constantly re-articulated


Lipschutz 1998 [Ronnie, prof of politics at UC Santa Cruz, On Security, ed. Ronnie Lipschutz,
http://www.ciaonet.org/book/lipschutz/index.html]

In this latter scenario, almost all conventional wisdoms about security no longer hold .
The orderly practices of the world of international
relations embodied in neorealist discourse--the practices of power, not the absence of disorder--require constant reiteration
and reification in mantra-like fashion, even as they become increasingly problematic in the hyperreality of the non-place
and time bound worlds of transnational society. The place-bound concerns of neorealists, and their idealized
decisionmakers, matter only insofar as they help to shore up a crumbling world view . Security, its discourses, and its
modes of production thus become a means of stanching the dikes not against the external forces of chaos but the internal
dynamics of state disintegration.

Positivism in IR falsely presupposes the naturalness of states as prior to IR system


Ashley 1984 [Richard, professor of political science at Arizona State University, ―The Poverty of Neorealism,‖
International Organization, Vol. 38, No. 2 (Spring, 1984), pp. 225-286, jstor]

The issue, however, is the theoretical discourse of neorealism as a move- ment, not the protective clauses that individual neorealists deploy to preempt or
deflect criticisms of that discourse's limits. Once one enters this theoretical discourse among neorealists, the state-as-actor model
needs no defense. It stands without challenge. Like Waltz, one simply assumes that states have the status of unitary actors .32
Or, like Gilpin, one refuses to be deterred by the mountainous inconsistencies between the state as a coalition of coalitions (presumably in opposition to
the losing coalitions against which the winning coalition is formed) and the state as a provider of public goods, protector of citizens' welfare, and solver of
the free-rider problem in the name of winners and losers alike. Knowing that the "objectives and foreign policies of states are
determined primarily by the interests of their dominant members or ruling coalitions ,"3 one nonetheless simply joins the
victors in proclaiming the state a singular actor with a unified set of objectives in the name of the collective good . This
proclamation is the starting point of theoretical discourse, one of the unexamined assumptions from which theoretical discourse proceeds. In short, the
state-as-actor assumption is a metaphysical commitment prior to science and exempted from scientific criticism .
Despite neorealism's much ballyhooed emphasis on the role of hard falsifying tests as the measure of theoretical progress,
neorealism immunizes its statist commitments from any form of falsification . Excluded, for instance, is the historically testable
hypothesis that the state-as-actor construct might be not a first-order given of international political life but part of a
historical justificatory framework by which dominant coalitions legitimize and secure consent for their precarious
conditions of rule. Two implications of this "state-centricity," itself an ontological principle of neorealist theorizing, deserve emphasis. The first is
obvious. As a frame- work for the interpretation of international politics, neorealist theory cannot accord recognition to-it
cannot even comprehend-those global collectivist concepts that are irreducible to logical combinations of state-bounded re-
lations. In other words, global collectivist concepts-concepts of transnational class relations, say, or the interests of humankind-can be granted an
objective status only to the extent that they can be interpreted as aggregations of relations and interests having logically and historically prior roots within
state-bounded societies. Much as the "individual" is a prism through which methodological individualists comprehend
collectivist concepts as aggrega- tions of individual wants, needs, beliefs, and actions, so also does the neorealist refract all
global collectivist concepts through the prism of the state.34 Importantly, this means that neorealist theory implicitly takes a side
amidst contending political interests. Whatever the personal commitments of in- dividual neorealists might be, neorealist theory allies with,
accords recognition to, and gives expression to those class and sectoral interests (the apexes of Waltz's domestic hierarchies or Gilpin's victorious
coalitions of coalitions) that are actually or potentially congruent with state interests and legitimations. It implicitly opposes and denies
recognition to those class and human interests which cannot be reduced to concatenations of state interests or transnational
coalitions of domestic interests. The second implication takes longer to spell out, for it relates to neorealist "structuralism"-the neorealist position
with respect to structures of the international system. Reflecting on the fourth element of structuralist ar- gument presented above, one might expect the
neorealist to accord to the structure of the international system an identity independent of the parts or units (states-as-actors in this case); the identities of
the units would be supplied via differentiation. The neorealist orrery disappoints these expectations, how- ever. For the neorealist, the state is
ontologically prior to the international system. The system's structure is produced by defining states as individual unities
and then by noting properties that emerge when several such unities are brought into mutual reference . For the neorealist, it
is impossible to describe international structures without first fashioning a concept of the state-as-actor. The proper analogy, as
Waltz points out, is classical economic theory- microtheory, not macrotheory. As Waltz puts it, "International-political sys- tems, like economic markets,
are formed by the coaction of self-regarding units." They "are individualist in origin, spontaneously generated, and un- intended."35
ENDI 2010 93
Security K Wave 1

AT: REALISM GOOD

Arguments about realism being key to change are based on historically situated understandings
of the idea of change that should be interrogated to expose the fractured multiplicities of realism
Walker 2002 [RBJ, professor of political science at the University of Victoria, ―Realism, Change, and International
Political Theory,‖ International Studies Quarterly, Vol. 31, No. 1 (Mar., 1987), pp. 65-86, jstor]

In fact, this paper will suggest that the


way in which change is conceptualized provides the most powerful point of entry into a
critical analysis of claims to political realism in general, and of recent forms of structural or ''neorealism " in particular.' The most
obvious difficulties that have arisen in this context are readily apparent in the writings of some of the most influential recent exponents of a realist
position. Those, like Kenneth Waltz (1979), who cling most tightly to the promised certainties of atemporal structuralisms and
positivist method, are taken to task for being unable "to account for , or even to describe the inost important contextual change in
international politics in this millennium: the shift from the medieval to the modern international system" (Ruggie, 1983: 273). Those, like Robert
Gilpin (1981, 1984), who work on the basis of a much greater sensitivity to the historicity of our existence, tempt us with the paradox that the world of
humankind is in a state of constant flux and change, and yet a Thucydides reborn would have little difficulty in explaining our contemporary agonies.
Unfortunately, it is not immediately obvious in this case that an appeal to an ahistorical theory of rational choice, on the one hand, and a broad cyclical
theory of hegemonic wars on the other, does much to confront this paradox in a sufficiently serious way. These difficulties underlie much of the recent
debate about realism and neorealism in international political theory. This debate has centered on a confrontation between defenders of
various forms of structuralism and several kinds of historicism. The critics of neorealism argue that the structures of the
international system that neorealists treat as more or less universal and eternal are in fact the specific consequences of
particular historical conditions. Against Waltz, for example, John Ruggie (1983) recommends greater attention to "diachronic processes" as well as
"synchronic articulations." Robert Cox (1981) draws on a variety of historicist writers to insist that the study of international politics itself, including the
form taken by realist theories in different eras, be analyzed more critically in terms of the historical context in which it arose. Ashley (1981, 1984) sees in
the more historicist or "classical" version of political realism a more authentic, that is, more hermeneutical and practical approach to the study of
international politics. At a more general level, R. N. Berki's (1981) extended discussion of political realism also derives its critical force from writers with
strong historicist sympathies. This paper explores the terrain that has been opened up by the recent confrontation between structuralist and historicist forms
of political realism. It seeks to clarify the theoretical and philosophical issues that are at stake in the critical response to neorealism in particular. It argues
that political realism must be understood less as a coherent theoretical position in its own right than as the site of a great
many contested claims and metaphysical disputes. Whether situated against the early 20th century crisis of historicism, analyzed as a
reworking of dilemmas originating between Judeo-Christian and Hellenic civilizational impulses, or taken as yet another benighted footnote to dualisms
inherited from classical Greek philosophy, claims to realism in international political theory carry meanings and implications from a
much broader discourse about politics and philosophy. In all these contexts, the conceptualization of "change" has been
crucial. The interest in change among analysts of international politics, realist or otherwise, does raise a number of daunting theoretical and
philosophical problems. John Vincent (1983) is undoubtedly right to warn us that the study of change in the abstract, without some idea of what it is that is
supposedly changing, is a rather fruitless exercise. It does not help us to know very much, if anything. Yet the plain common sense of this view depends
primarily on an epistemological interest. A number of other complex issues are at stake here, puzzles that have more of an ontological than an epistemic
character. Moreover, to speak of change at all, whether abstractly or with reference to some particular social process, is to do so within theoretical and
philosophical categories that have been constituted in historically specific ways. We have become especially attached to treating stasis and
change, being and becoming, structure and history as mutually exclusive oppositions . These oppositions are central to the claims,
metaphysical disputes, and, I will argue, mystifications around which our understanding of political realism has coalesced.2 The analysis developed here is
primarily critical in intention. It explores the way claims to political realism in international politics have drawn on quite different and
fundamentally conflicting philosophical traditions. To the extent that such differences have been ignored or covered over,
realism becomes less a hard-headed portrayal of international realities than a systematic evasion of the critical skills
necessary for a scholarly analysis of those realities. This analysis is most emphatically not an attack on realism from a
utopian stand- point. On the contrary, it takes the position that realism and utopianism as we have come to know them in
international political theory reflect similar difficulties. However, the analysis is concerned with the conditions under which any resistance to
realist claims is so frequently condemned for its utopian tendencies.
ENDI 2010 94
Security K Wave 1

AT: REALISM GOOD

The form of realism they advocate is an excuse for maintaining status quo justifications for force
and violence and discouraging historical or political thought
Walker 2002 [RBJ, professor of political science at the University of Victoria, ―Realism, Change, and International
Political Theory,‖ International Studies Quarterly, Vol. 31, No. 1 (Mar., 1987), pp. 65-86, jstor]

As it informs a rather large and influential literature on geopolitics and military affairs, realism has often degenerated into
little more than an antipolitical apology for cynicism and physical force. To the extent that deeper roots are sought, the
search often comes to a rather abrupt halt with an arbitrary theology of the fall of man or the ritual invocation of some
seemingly incontestable ancient text. But beyond this, there is clearly a much more serious struggle with important philosophical
issues visible in some of the writings of some of the more prominent realists. The problem of change lies right at the heart
of these struggles. This was particularly the case with writers like Hans J. Morgenthau, E. H. Carr, John Herz, and Raymond Aron who all wrote in the
shadow of the early 20th century crisis in German historicism. More recently, the connection between realism and historicism is brought out in a
particularly useful way in R. N. Berki's (1981) study of political realism in general. In Berki's analysis, the term political realism is taken to encompass a
very wide array of themes, with classical and medieval, as well as more modern, formulations. But it is the general problematic of historicism that forms
the main backdrop for discussion. Even the names of the theorists who are taken to be most important for the reconstruction of a more viable orientation-
Aristotle, Augustine, Machiavelli, and Hegel-are enough to remind us that political realism has to a large extent been informed by deeply rooted
conceptions of time, change, and history. There are, for example, a number of grounds on which one can dispute the claim that Hegel provides the "still
valid standpoint of political realism, in need only of marginal updating and some terminological revision" (Berki, 1981: 69), but not the least of them
concerns Hegel's transformation of earlier concepts of time and change into a particu- larly powerful vision of progressivist history. Yet this continuity of
historicism and political realism is obviously not all there is to it. Some of the most powerful forms of realist analysis in international
political theory draw upon traditions that are less concerned with change and history than with stasis and structure. It is just
this structuralist orientation that has tended to predominate in the more recent literature . Indeed, it has become quite apparent that
not only can realism be understood as one pole of a broader discourse about international politics, in which idealism is
posed as the opposite pole, but that political realism contains polarities and contradictions of its own . These polarities and
contradictions have been seriously obscured by a continuing appeal to a single tradition of realism located somewhere
among the classic texts of the history of political thought. Such an appeal must be refused.

Realist theories of international relations obscure the violence of universalizing norms of


civilization in order to guarantee the preservation of inter-state anarchy
David Campbell\Michael Dillon, ―The end of philosophy and the end of international relations,‖ The Political Subject
of Violence, 1993, pp. 42-3
Above all, however, it is contemporary international relations that regularly insists upon this resolution. And it is there that strategies for defending it are
persistently elaborated: The overriding characteristic of claims about political realism in international relations since mid-century, in fact, has been
the transformation of historicist claims about contingency in time into structuralist claims about anarchies in space ; anarchies,
moreover, that have attracted the fiercest rage for epistemological order in the name of empirical social science. We can consequently reconfigure
international relations, with Walker, as a site for repeating the formula that attempts to resolve characteristically modern spatio-temporal political and
ethical contradictions in largely spatial terms. It thereby serves, also, as modernity's safe-depository of political thinking. Just to emphasise the point made
at the beginning of this Introduction, this is what we mean by the end of international relations. The limit of its thinking; the aim of its thinking; the
realisation or fulfilment of its thinking; and the vantage point from which, on its limit, we can see it as an edifying problematisation of political modernity,
aspiring to represent its truth while complicit in its production of the real through a persistent defence of its reality principle: While theories of
international relations address themselves explicitly to the extremes of violence on the colliding edges of modem states, they shy away from the
violence immanent in a civilization that requires the violent edges of modern states to guarantee claims to goodness, truth
and beauty within. In effect, international relations is the place to which the violence of modernity may be legitimately deferred ... Everyone else can
then ignore the constitutive place of violence in the construction of universalizing ambition, unless universalizing ambition runs up against the presence of
other universalizing ambitions, and war once again reveals the double standards inherent in universalizing ambition. The charge here, effectively, is that
international relations is little more than a cheer-leader for modernity, when the task is to figure-out not whether one says yes or
no to our modern condition, as if we had any choice in the matter of already being modern, but how to bear the inescapable fate of modernity
in ways that do not condemn us to the self-immolation threatened either by its facile affirmation or rejection. In the event, the
possibility of common humanity gave way in political modernity, and continues to give way, before the demands of a citizenship which is the affirmation
of 'inclusive identities within particular states' that continue to make universalist claims to truth and the good. 'For all that modernity has come to be
understood as an obsession with universal reason', Walker concludes for us, 'it is an obsession informed by a priori admissions that we are not after all,
human, at least politically'. For that reason we return to the question of the ethical in a postface.
ENDI 2010 95
Security K Wave 1

AT: REALISM GOOD

Conventional IR can’t grasp new relations of diplomacy-poststructuralism is a superior


epistemology
DER DERIAN 1992 [JAMES, PROF INT‘L STUDIES @ BROWN, ANTIDIPLOMACY: SPIES TERROR SPEED AND
WAR, PAGE 3-4]

If this book attempts to open up a field known for its closure, it is so that we might better understand late modern challenges to traditional diplomatic
practices, to which I have given the name antidiplomacy. A prior work of mine, On Diplomacy: A Genealogy of Western Estrangement, included a
genealogy of the conflict between particularist states and universalist forces which gave rise to an earlier ideological form of antidiplomacy. With Hegel as
my guide, I attempted to show how a universal alienation, when mediated through particular interests, produces new and often violently
antithetical forms of diplomatic relations. In this book I argue that new technological practices and universal dangers, mediated by the
particular interests of the national security state, have generated a new antidiplomacy. In short, what distinguishes late modern
antidiplomacy from earlier forms is how it constitutes and mediates estrangement by new techniques of power and representations of danger. These
new techniques of power are transparent and pervasive, more ―real‖ in time than in space, and produced and sustained through the
exchange of signs rather than goods. They have proven to be resistant if not invisible to traditional methods of analysis. They do not ―fit‖
and therefore they elude the traditional and the re-formed delimitations of the International Relations field: the geopolitics of realism,
the structural political economy of neorealism, the possessive institutionalism of neoliberalism. In contrast, I believe that
poststructuralism can grasp – but never fully capture – the significance of these new forces for international relations. In this book I will
examine three forces that stand out for their discursive power and shared problematic. Their discursive power is chronopolitical and technostrategic,
and they have generated a late modern problematic for a system of states which increasingly seems resistant to comprehension by
traditional styles and systems of thought. To clarify: they are ―chronopolitical‖ in the sense that they elevate chronology
over geography, pace of space in their political effects; they are ―technostrategic‖ in that they use and are used by technology for
the purpose of war; they have a discursive power in that they produce and are sustained by historically transient discourses which mediate our
relations with empirical events; and the problematic is late (or post-) modern because it defies the grand theories or definitive structures which impose
rationalist identities or binary oppositions to explain international relations. Hence, a poststructuralist analysis is called for, to show us
how these new technological and discursive practices, mediate and often dominate relations with other states, but also to
tell us. about their relationship to ourselves; that is, how their power is manifested in the boundaries they establish for what can
be said and who can say it with authority in international theory. The three forces challenging traditional diplomacy that I will examine are spies
(intelligence and surveillance), terror (global terrorism and the national security culture), and speed (the acceleration of pace in war and diplomacy'. The
-
problematic they have generated can be simply put: the closer technology and scientific discourse brings us to the "other" - that is,
the more that the model is congruent with the reality, the image resembles the object, the medium becomes the real-time message - the less we see of
ourselves in the other. Theoretical reflection loses out to techno-scientific reification.

Vote negative – the alternative can bypass realist dichotomies and re-invent understandings in
IR
BILGIN 2001 – PROF IR BIKENT
ALTERNATIVE FUTURE FOR THE MIDDLE EAST, FUTURES, NO 33

Critical approaches to international relations seek to bypass these unhelpful dichotomies of pessimism/optimism and
realism/idealism by pointing to the constitutive role theories play. From a critical perspective, ‗theories do not simply explain or
predict‘, as Steve Smith has maintained. ‗They tell us what possibilities exist for human action and intervention; they define not merely
our explanatory possibilities but also our ethical and political horizons‘ [3]. This is not to say that theories ‗create‘ the world in a philosophical sense of the
term, but that theories help to organise knowledge, which, in turn, informs, enables, privileges and legitimises certain practices
whilst inhibiting or marginalising others. In other words, critical approaches to international relations view the future of world politics as open,
for they believe, in Ken Booth‘s words, that ―social inventions like international relations cannot be uninvented overnight, but they
can be reinvented, over time‖ [4].
ENDI 2010 96
Security K Wave 1

AT: REALISM GOOD: NUCLEAR WAR

Realism can’t address nuclear threats—the notion of autonomous, self-interested nation states is
not viable in the nuclear age
Morgenthau 95 (Hans, was a political scientist who taught at the University of Chicago and at the Graduate Center at the City University of New
York, ―The Intellectual and Political Functions of Theory‖, International Theory: Critical Investigations, edited by James Der Derian, p. 41-42)

It is this repetitive character of internationa politics, that is, the configurations of the balance of power, that lends itself to theoretical sysemization. I would
also hesitate to equate international theory with philosophy of history. Theory is implicit in all great historiography. In historians with a philosophic bent,
such as Thcydides and Ranke, the history of foreign policy appears as a mere demonstration of certain theoretical assumptions which are always present
beneath the surface of historical events to provide the standards for their selection and to give them meaning. In such historians of international politics,
theory is like the skeleton, which, invisible to the naked eye, gives form and function to the body.what distinguishes such a history of international politics
from a theory is not so much its substance as its form. The historian presents his theory in the form of a historical recital, using the chronological sequence
of events as a demonstration of his theory. The theoretician, dispensing with the historical recital, makes the theory explicit and uses historic facts in bit
and pieces to demonstrate his theory. Yet both Wight‘s and my orientation are historical, and it is this historical orientation that sets us apart from the
present fashionable theorizing about international relations. This theorizing is abstract in the extreme and totally unhistoric. It endeavors to reduce
international relations to a sytem of abstract propositions with a predictive function. Such a system transforms
nations into a stereotyped symmetric or assymmetric relations. What Professor Wight has noted of international law
applies with particular force to these theories: the contrast between their abstract rationalism and the actual
configurations of world politics. We are here in the presence of still another type of progressivist theory. Its aim is not
the legalization and organization of international relations in the interest of international order and peace but the rational
manipulation of international relations and, more particularly, of military strategy in the interest of predictable and
controlled results. The ideal toward which these theories try to progress is ultimately international peace and order to be
achieved through scientific precision and predictability in understanding and manipulating international affairs. In
view of their consistent neglect of the contigencies of history and of the concreteness of historical situations that all these theories have in common, they
are destined to share the fate of their progressivist predecessors: they must fail both as guides for theoretical unerstanding and as precepts for action.
However, the practical consequences of their theoretical deficiencies are likely to be more serious than those of the
predecessors. The straits in which the Western democracies found themselves at the beginnning of WWII were, in good measure, the result of the
reliance upon the inner force of legal pronouncements, such as the Stimson Doctrine, which refused to recognize territorial changes brought about by
violence; of legal agreements, such as the Kelogg-Briand Pact and non-aggression treaties; and of international organizations, such a the League of
Nations, which were incapable of collective action. The scientist theories of our day pretend to be capable of manipulating with
scientific precision a society of soverign nations that use weapons of total destruction as instruments of their
respective foreign policies. With that pretense, these theories create the illusion that a society of soverign nations thus
armed can continue the business of foreign policy and military strategy in the traditional manner without risking its
destruction. They create the illusion of the viability of the nation-state in the nuclear age. If statesmen should take
these theories as their pseudoscientific word and act upon them, they would fail, as the statesmen of the interwar period
failed when they acted upon the progressivist theories of their day.
ENDI 2010 97
Security K Wave 1

AT: REALISM GOOD- HOBBES

Hobbes isn’t even a realist


Walker 2002 [RBJ, professor of political science at the University of Victoria, ―Realism, Change, and International
Political Theory,‖ International Studies Quarterly, Vol. 31, No. 1 (Mar., 1987), pp. 65-86, jstor]

It becomes redefined in terms of time: time as the context of political life; temporal images as the source of new vocabularies of political thought within a
discourse dominated by universals; and maxims about how to cope with time, change, and illusions as the distillation of political knowledge. Even where
he appeals to the possibility of fixing political life within a spatial form-lo stato-it is a spatial form with its own temporal contingency. At best, he appeals
to the possibility of establishing a temporary home for man, one that even the greatest efforts of republican virt?u are unable to insulate from inevitable
decay over time.3 In taking its cue from Machiavelli, political realism resonates with all those other discourses that have also given priority to temporality,
to difference through time, and which also attract the indictment of relativism. The Sophistic movement is perhaps relevant here, but it is the historicists of
late 19th century Germany who have been most important for the specific forms of realism that have been influential in international political theory in
this century. Temporality became history. History became the cunning of reason. But by the late 19th century, the cunning of reason no longer seemed
persuasive. History collapsed into historicism, and the stress on temporality and process once again generated the specter of relativism. Whether in
terms of the Nietzschean challenge to prevailing theories of progress, or of the barbarities of a war to end all wars, the
seminal sources of realism in international political theory were acutely aware that the clash between "Enlightenment and Despair"
(Hawthorn, 1976), between philosophies of history grounded in Enlightenment optimism and their radical rejection, constituted
the starting point for almost any serious discussion of politics. International politics provided a particularly compelling case for despair,
but the general problematic had a much wider application. Resistance to this trend took many forms, including the various attempts to re-
establish the ground for epistemology in different genres of neo- Kantian natural science. From the point of view of international political theory, the most
interesting move was to take the nation-state-the German nation-state in particular-as the absolute value by whch the dilemmas and mysteries of relativism
could be resolved. The most important figures here are Friedrich Meinecke and especially Max Weber. They are arguably the most significant figures
through whom the Machiavellian stress on time and change as the essential political reality has been passed on to the contemporary realist traditions of
international political theory. Both Meinecke and Weber took up the Machiavellian problematic directly and explicitly.4 This has been rather obscured in
Weber's case by the rather selective way in which his work has been received into the conventional sociological canon, although it has not been lost on
two of the most influential of modern realist writers, Hans J. Morgenthau and Raymond Aron. Morgenthau himself did not explicitly acknowledge his debt
to Weber until late in life, but the fairly obvious connection has usefully been underlined in a recent study of the way Weber's work has been received and
reinterpreted by later commentators that can be identified. A rather different set of ideas is often implied by an appeal to political realism, one linked less
to Machiavelli than to Thomas Hobbes. Hobbes has been the subject of a rather large recent literature in international political theory, and there is a
commonly identified "Hobbesian tradition" in this field, even though Hobbes himself wrote very little explicitly on international politics as such. The
usual focus is inevitably on the proposition that relations between states are analogous to the relations between individuals
in Hobbes' state of nature. Hedley Bull (1981: 720-721) for example, has reiterated the feeling that we are "entitled to infer that all of what Hobbes
says about the life of individual men in the state of nature may be read as a description of the condition of states in relation to one another. "7 On this
account, states are led to war because of competition for material possessions, mistrust, fear, and the pursuit of glory, with
fear being the prime motive in that it supposedly leads to a concern to secure what we already have . In this "international state of
nature," there is, therefore, only the natural right of self preservation among equals. But Hobbes has not been accepted as a genuine realist
without considerable equivoca- tion. To begin with there are the general problems that there is a good deal more to Hobbes
than his evocation of a state of nature, and that his overall position is susceptible to radically different interpretations. But
even those who are content to stress the state of nature argument acknowledge that there are some problems in applying it
to inter- national politics. After all, Hobbes does remark (1968: 188) that war between sovereigns is relatively tolerable: "there
does not follow from it, that misery which accompanies the liberty of particular men. " Indeed the institution of the contract
itself implies that relations between states are necessarily quite different from relations between individuals. Thus it can be
argued that states are less vulnerable than individuals and cannot be so easily removed with a single blow. The sovereign is
able to make reasonable calculations about relative strategic forces, and can at least ensure some security from the subjects
for whom he is the source of justice and right. The state of war can even stimulate the domestic economy and thus in some
way "improve" life in civil society. Furthermore, the central assumptions of Hobbes' state of nature concern the autonomy and equality of the
individuals in it, an assumption which makes little sense in the international context where inequality and hegemony, in the form of the pro- rogatives and
obligations of "great powers," is itself seen as a principle of order. It is possible to go even further with this line of reasoning. Given that states are
not as vulnerable as individuals, prudence and fear suggest not the necessity of a global Leviathan but the need for some
rules of coexistence; principles of sovereignty and non- intervention, for example, or mechanisms like the balance of
power. In teasing out these themes, Hobbes begins to slide out of the realist camp and becomes a prime example of a theorist of the
international system as a kind of society, rather than an anarchy (Bull, 1981; Vincent, 1981). There is, however, more to it than even this degreee of
ambiguity.
ENDI 2010 98
Security K Wave 1

AT: REALISM GOOD - CRITICAL REASONS

Realism is bankrupt as a critical instrument and locks in the status quo


Ashley 1984 [Richard, professor of political science at Arizona State University, ―The Poverty of Neorealism,‖
International Organization, Vol. 38, No. 2 (Spring, 1984), pp. 225-286, jstor]

Fourth, despite its spirited posturing on behalf of political autonomy and in opposition to the alleged economism of other traditions, neorealist historicism
denies politics. More correctly, neorealism reduces politics to those aspects which lend themselves to interpretation exclusively
within a frame- work of economic action under structural constraints. In so doing, neorealism both immunizes that
economic framework from criticism as to its implicit political content and strips politics of any practical basis for the
autonomous reflection on and resistance to strictly economic demands. It thereby implicitly allies with those segments
of society that benefit from the hegemony of economic logic in concert with the state . Politics in neorealism becomes
pure technique: the efficient achievement of whatever goals are set before the political actor. Political strategy is deprived of
its artful and performative aspect, becoming instead the mere calculation of instruments of control. Absent from neorealist categories is
any hint of politics as a creative, critical enterprise, an enterprise by which men and women might reflect on their goals and strive to shape freely their
collective will. Taken together, reflections on these "four p's" suggest that neorealist struc- turalism represents anything but the
profound broadening and deepening of international political discourse it is often claimed to be. Far from expanding
discourse, this so-called structuralism encloses it by equating structure with external relations among powerful entities as
they would have themselves be known. Far from penetrating the surface of appearances, this so-called structuralism's fixed categories
freeze the given order, reducing the history and future of social evolution to an expression of those interests which can be
mediated by the vectoring of power among competing states-as-actors.76 Far from presenting a structuralism that envisions political
learning on a transnational scale, neorealism presents a structure in which political learning is reduced to the consequence of
instrumental coaction among dumb, un- reflective, technical-rational unities that are barraged and buffeted by tech-
nological and economic changes they are powerless to control.

Neorealism prevents critical interventions into history or state-centeredness


Ashley 1984 [Richard, professor of political science at Arizona State University, ―The Poverty of Neorealism,‖
International Organization, Vol. 38, No. 2 (Spring, 1984), pp. 225-286, jstor]

Again, though, none of this is to say that neorealist "structuralism" is without its attractions. For one thing, and most generally, there
is something
remarkably congenial about a structuralism that pretends to a commanding, objective portrait of the whole while at the
same time leaving undisturbed, even confirming, our commonsense views of the world and ourselves . As compared to
Wallerstein's conception of the modem world system, for in- stance, neorealist structuralism is far more reassuring as to the objective
necessity of the state-as-unit-of-analysis convention among students of politics.77 It thus relieves this particular niche in the
academic division of labor of responsibility for reflection on its own historicity. Its pose of Weberian detachment can be preserved. For
another thing, this strange structuralism finds much of its appeal in the fact that it complements and reinforces the other three commitments of the
neorealist orrery. As already noted, neorealism's atomistic understanding of structure gives priority to-and then reconfirms-the
commitment to the state-as-actor. One might also note that neorealism employs the only form of structuralism that could possibly be consistent
with its utilitarian and positivist conceptions of international society. Anchored as they are in the ideal of rational individual action under
meaningless, quasinatural constraints, these conceptions would be radically challenged by modes of structuralism that
question the dualism of subject and object and thus highlight the deep intersubjective constitution of objective international
structures. Neorealism is able to avoid this radical challenge. It is able to do so by restricting its conception of structure to the physicalist
form of a clockwork, the philo- sophical mechanism so dear to the heart of the Industrial Revolution's intelligentsia.
ENDI 2010 99
Security K Wave 1

AT: REALISM GOOD - CRITICAL REASONS

And, their positivist epistemology is circular and self-perpetuating


Ashley 1984 [Richard, professor of political science at Arizona State University, ―The Poverty of Neorealism,‖
International Organization, Vol. 38, No. 2 (Spring, 1984), pp. 225-286, jstor]

3. The ghost of the old revolution The "secret world," John le Carre writes, "is of itself attractive. Simply by turning on its axis, it can draw the weakly
anchored to its center."78 The same, we can now note, might be said of the neorealist orrery of errors. Having seen its several elements whiz by-statism,
utilitarianism, positivism, structuralism, and statism yet again-we sense that there is a strange unity of contrarieties here. We sense that the whole machine
exerts a centripetal force that is difficult to defy. To be sure, when we slow and examine the elements we find that errors and absurdities abound. We find,
for example, that the utilitarian interpre- tation of international order presupposes a conception of the state-as-actor- a conception that a utilitarian would
want to disown. We find, too, that neorealist statism runs contrary to any genuinely structuralist understanding of the international system. We find that
neorealism appeals to a Weberian interpretation of positivist method, a method that parades as the end of ideology even as it subordinates all criticism to a
scientifically indefensible commitment to technical rationality's objectivity and neutrality. And we find that despite neorealism's pretensions to the status
of a political struc- turalism, neorealist theory is as economistic as they come. Yet the neorealist orrery is meant never to be held at rest. It
presents itself only in motion. And thanks to this, its countless errors become not damning indictments but counterweights
to other errors, balancing and perpetuating the motion of the whole. The limits of positivism obscure the errors of statism in
a state-as-actor conception of international order, which reduces systemic analysis to a physicalist structuralism, which in
turn propels us into the utilitarian world of technical reason and necessity, which brings us around to positivism once again .
Around and around it spins, eroding and then consuming the ground upon which opposition would stand. Around and around it spins, until we lose sight
of the fact that it is only motion. Like le Carres secret world, this neorealist orrery has no center at all.
ENDI 2010 100
Security K Wave 1

AT: SECURITIZATION KEY TO ACTION

Actions resulting from securitization falsely construct a world community in the name of which
violence occurs on a global scale
Kelstrup 2004 [Morten, Writer and editor for Sage Publications, ―Globalisation and Societal Insecurity‖,Contemporary
Security Analysis and Copenhagen Peace Research, pg.113-4]

Perhaps it is more fruitful in social systems with -no clear or generally accepted institutionalisation of normality to see
securitisation as attempts to create 'formative moments', situations in which new norms and maybe new kinds of agency can
be constituted and legitimised. Securitisation can be regarded as appeals for legitimacy in extraordinary situations for new
kinds of action and maybe also for new actors. Thus, talking about threats towards civilisation and of responses from 'the
world community' implies that the 'world community' is somehow articulated as a rather strong identity, and that actions are
legitimised with reference to the defence of this entity. The securitisation can in this view be seen as a quest for a new basis
for legitimacy or for 'extraordinary' legitimacy. This leads us to the last point in this chapter: the securitisation of terrorism
after 11 September can be seen as a turning point in which a new strategy is launched, not only a strategy for security, but a
new strategy for governanceance in the global system. The securitisation in the global system can be seen as an attempt to
legitimise such a new, global strategy.

Securitization in all forms is a replication of violence, death, and war.


Aradau, 2001 [Claudia Research Associate in the Centre for International Relations at King‘s College London, December,
Beyond Good and Evil: Ethics and Securitization /Desecuritization Techniques, Rubikon, Quarterly Academic Journal,
http://venus.ci.uw.edu.pl/~rubikon/forum/claudia2.htm]

Securitization has been defined in performative terms, either as a speech act[7] or as a principle of formation that does
things[8]. In its discursive and non-discursive forms, securitization has profound social and political implications. It
functions as a technique of government which retrieves the ordering force of the fear of violent death by a mythical replay
of the variations of the Hobbesian state of nature. It manufactures a sudden rupture in the routinized, everyday life by fabricating an
existential threat which provokes experiences of the real possibility of violent death.[9] Initially restricted to the possible
‗violent death‘ of the state and therefore focused on its survival, security practices can be expanded to include concerns for
the survival of other different objects: larger or smaller communities or even individuals. The logic of war is translated
invariably from state to society or the individual. ‗Security‘ can thus be inscribed on a discourse on the basis of this underlying logic or what
Barry Buzan has called a ‗specific rhetorical structure‘ (survival, priority of action, the securitized issue is presented as an issue of supreme priority).[10]
In the 1998 book, the CoS define the rhetoric or grammar of security as a plot that includes an existential threat, a point of no return, and a possible way
out to which they add ‗the particular dialects of different sectors‘, such as identity for the societal one.[11]
ENDI 2010 101
Security K Wave 1

AT: POST-STRUCTURALISM BAD

Turn – there’s a difference between denying all reality and critically exposing meaning. Post-
structural analysis recognizes the representational force of IR theory
DER DERIAN, 1992 [James, PROF INT‘L STUDIES @ BROWN ANTIDIPLOMACY: SPIES TERROR SPEED AND
WAR, PAGE 7-8]
First, poststructuralism is not, as many critics have claimed, inherently anti-empirical. This book does in fact contain a "research program,"
but not one which assumes that the object of research is immaculately reproduced by the program. Poststructuralism differs from rationalist
approaches in that it does not hold that international theorists mirror the reality of world politics through their intellectual
analysis. Both use and are used by language: meaning endlessly differs and is deferred through the linguistic interaction of theorist and text.
Rationalists cling to the faith that there is an object, a truth, a reality out there, that is waiting for the right method to come along and in the name of
scientific progress make use of, make sense of, give order to it. Moreover, the realities of world politics increasingly are generated,
mediated, simulated by technical means of production, further distancing and alienating them from some original,
unproblematic meaning. It is this very heterological nature of discourse that the traditional theories of IR, in a demonstrative, hegemonic act, always
dream of fixing, reducing, subjecting to a single, monological meaning. This is not to reduce IR to a linguistic practice, nor to claim there is no
truth, no values, no reality. Rather, it is to refute the claim that there is an external being, supreme epistemology, ultimate
theory that can prove, adjudicate, confirm an existence independent of its representation. A poststructuralist approach proceeds by
recognizing and investigating the interrelationship of power and representational practices that elevate one truth over
another, that subject one identity to another, that make, in short, one discourse matter more than the next.. Such an investigation requires a
semio-critical approach, one that might problematize and dismantle empirico-positivist categories by revealing their
interdiscursive origins, logical inconsistencies and interpretive inadequacies.

Critique of totalist theory can be constructive


DER DERIAN, 1992 [JAMES, PROF INT‘L STUDIES @ BROWN, ANTIDIPLOMACY: SPIES TERROR SPEED AND
WAR, PAGE 7-8]
But more is needed. Poststructuralism is not simply a negative critique – although it has, by its more modish uses, been confused as such. In
most cases – and certainly in the case of Foucault – it clears but does not destroy or deny the existence of the ground for a constructive theory. Even in the
more radical applications of deconstruction it takes aim at totalist, transcendentalist, closed theory - not all theory. Lending critical
support to an essay by Richard Ashley, the political theorist William Connnollv outlines the features of a constructive theory in poststructuralism: One
might seek, not to impose one reading on the field of discourse, but to elaborate a general reading that can contend with others by broadening the
established terms of debates; not to create a transformation of international life grounded in a universal project, but to contribute
to a general perspective that might support reconstitution of aspects of international life; not to root a theory in a transcendental
ground, but to problematize the grounding any theory presupposes while it works out the implications of a particular set of
themes; not merely to invert hierarchies in other theories (a useful task), hut to construct alternative hierarchies that support
modifications in relations between identify and difference.
ENDI 2010 102
Security K Wave 1

AT: POST-STRUCTURALISM BAD

Criticisms of post-structuralist dissidence falsely draw boundaries between positivism and


criticism, ignoring the judgments and constructions implicit in any claims to objective truth
Ashley and Walker 1990 [Richard, Associate Professor of Political Science, Department of Political. Science, Arizona
State University, Tempe, RBJ, professor of political science at the University of Victoria, ―Conclusion: Reading
Dissidence/Writing the Discipline: Crisis and the Question of Sovereignty in International Studies,‖ International Studies
Quarterly, Vol. 34, No. 3, (Sep., 1990), pp. 367-416, jstor]

As texts in their own rights, critical


readings of works of dissidence produce for their audience a similar double bind. On the one hand, they
create for their audience a role of one who must make an either/or choice with respect to the critical readings themselves:
for or against the position taken, belief or disbelief in the judgment cast. It is the role of one who understands that she has
the freedom to make this choice, that it would be irresponsible not to choose, that she must take responsibility for the
choice she makes, and that even a failure to choose counts as a choice in the demand- ing circumstances of the moment.
Depending upon the specific example considered, the choice is presented as one of disciplinary authority versus a gathering
of mar- ginal challengers of unproven legitimacy; the maturity and wisdom of elders versus the bravado of tawdry youth; a
commitment to a position or perspective in history versus a lack (or concealment) of a position or perspective; transparency
of language versus impenetrable elitist jargon; the possession of firm evaluative standards versus the relativism of a
postpositivist void; ethical commitment versus the radical abolition of all ethical codes; reason versus irrationality; the
accomplishments of modernity versus a reckless repudiation of these accomplishments; a forthright facing up to global
problems versus a diffuse and disabling skepticism; a dedication of energies to productive scholarly labors versus the
spending of energies in fleeting pleasures; and so on. Considering such oppositions, the audience is supposed to understand that a
critical reading of marginal and dissident works aligns with the first term in any such pair . The audience is also supposed to
understand that it is called upon to choose this alignment, too. And the audience is supposed to understand that in taking up this position, it will be
opposed to the difficulties, dangers, and illicit seductions connoted by the second term in each instance. On the other hand, even as critical
commentaries such as those considered here urge this choice, they deprive their audience of any basis for the choice it is
called upon to make. In keeping with the fourth observation, they do so by effecting the postures of what De Man (1979:245) calls "exhortative
performatives that require the passage from sheer enunciation to action"-postures that make it plain that just here and now in this active moment of
judgment at the discipline's edges, the foun- dations of rational thought and argument supposedly prevailing at the center of the discipline simply do not
and cannot apply. These commentaries invoke the idea of disciplinary standards at one with a perspective against the spectre of relativism, but they recur
to no standard save the idea of standards, and they honor no perspective save one that knows it needs a perspective. They invoke the abstract image
of analyti- cally detached and dedicated scholarship, but they ask to be received in an attitude of immediate and unquestioning familiarity,
and they exhibit no dedicated scholarship. They invoke the ideals of truth and literal meaning, but they put the question of their own
truth in abeyance as they engage in figural play. What is the audience to make of this? If, in the exercise of its freedom, the
audience chooses to question these critical readings and the supposed discipline or culture they defend, then it would seem
to pass over to the side of the dissidents who are the objects of critical judgment . It at least potentially stands convicted of being
dubious about rational standards, of lacking or concealing a perspective, of favoring relativism, and of being unconcerned about matters of truth. But if
the audience chooses to embrace these critical readings and the judgments they make, then its choice can be based upon no
rational standard; it can reflect no certain perspective; it must be a relativistic choice; and it must be a choice that defers all
encounters with the question of truth.
ENDI 2010 103
Security K Wave 1

AT: CRITICISMS THAT MAKE FUN OF POST-STRUCTURALISM

Criticisms of post-structuralism in IR are based on the false demarcation of boundaries and


disciplines and NOT the objective positivism they claim to deploy
Ashley and Walker 1990 [Richard, Associate Professor of Political Science, Department of Political. Science, Arizona
State University, Tempe, RBJ, professor of political science at the University of Victoria, ―Conclusion: Reading
Dissidence/Writing the Discipline: Crisis and the Question of Sovereignty in International Studies,‖ International Studies
Quarterly, Vol. 34, No. 3, (Sep., 1990), pp. 367-416, jstor]

Example 7: Skepticism about the hubris of hegemonic paradigms is all very well, but surely a celebration of marginality must
signal a retreat from ethical and political commitment, a legitimation of quietism and navel-gazing in the face of the real
perils of global political life. What do such forms of scholarship have to say about the great problems and dangers that
confront us? Surely, in the field of international politics especially, we must aspire to the big picture , to the univer- sal, even to the
heroic. How else are we to respond to problems of war and poverty, environmental degradation or the internationalization of
production? Example 8: How avant garde, this imprecise language of literary allusion. How playful. How enticing. And how
very conservative. All dressed up in a stylish garb, these works flirtatiously whisper the radical code of resistance, and they
thereby seduce the revolutionary mind, but in the end, they induce an antipolitical slumber brought on by a diffuse,
directionless, irrational, and ultimately paralyz- ing reflection on the question, who are we? Our commitment to a cause, our
rational standards of choice and judgment, our prospects for concerted political strategy, our revolutionary energies, our
ability to focus and mobilize our re- sources so that that which represses and endangers us may be defined and overthrown-
all of this is relativized, scattered, and ultimately nullified by this insistent questioning . Peter Dews (1987) is right: what we have
here is a "logic of disintegration." After reading these essays, I'm spent; all I want to do is roll over, smoke a cigarette, and catch a
nap. These fragments of critical readings provide but a few examples of increasingly familiar ways in which scholars of
international relations and the social sciences in general often interpret, interrogate, and reply to works of dissidence that
speak from disciplinary margins. No doubt other examples could be offered. We think these fragments suffice, however, to illustrate a
considerable range of likely critical responses that spans from left to right. Five things about these snippets are notable. First, such critical
commentary is not typically offered or received as the normal, proper activity of a discipline or tradition, however that discipline
or tradition be defined. Such commentary is typically encountered in a footnote, a review essay, a contribution to the occasional symposium on the
discipline's future, a reading semi- nar, or the banter and sideplay of professional conferences. Rarely is it encountered as the main theme
of a
refereed journal article or a formal research presentation at a professional meeting. In brief, such commentary is offered as
parenthesis. It is put forth as a pause that is occasioned by the passing encounter with the moment of dissidence and that is
bracketed and set off from the real projects to which the commentators and their audiences are soon to return . Second, when
critical comments such as these are offered, they are typically pro- nounced in a cool, collected, self-assured voice of an "I"
or "we" that neither stum- bles nor quavers with self-doubt. Sometimes, this posture of self-assurance takes the form of nonchalance, even
indifference, as if the commentary were roughly compa- rable to a remark about the shrubbery overgrowing the side of a highway one travels. An air of
nonchalance is difficult to sustain, however, when dissident events disturb a sense of direction or when marginal works of thought pose questions that are
diffi- cult to ignore. On such occasions, equanimity often gives way to exasperation tinged with embarrassment, a sense that it
would be better if these things did not have to be said, a regret that voices of dissidence-though sometimes raising
interesting ques- tions-are somehow oblivious to the obvious things that truly refined scholars should already know. On still
other occasions, such as conversations between teacher and student, when the addressee of these critical readings cannot yet be presumed to be a mature
member of the profession, an air of cool detachment might be replaced by a tone of sobriety, even solemnity, that reminds the potentially wayward novice
that the reading is a kind of vow that he, like all members, must earnestly recite. Yet all these reading postures-nonchalance, exasperation,
solemnity during the rite of passage-have something in common. As gestures in themselves, they at once presuppose and
indicate the same location. These postures indicate that such critical remarks belong not at the center of the discipline where
its serious and productive work is proudly presented and logically weighed, but at its boundaries, at its edges, at the
thresholds or checkpoints of entry and exit. They indicate, in the same stroke, that the discipline's territorial
boundaries are already marked, that the difference between outside and inside is already given, and that the
discipline, the tradition, the "everybody who knows and agrees with this reading" is already assuredly there . Third, it
follows that these critical commentaries do not come to the relationship between reader and text and effect a posture befitting the situation of marginality
presupposed by the texts themselves. They do not, for example, adopt a posture that is is sometimes called "female response criticism."3 They do not
effect an attitude in which the reader regards her own identity and experience as a kind of text in the process of being written and, in view of this, is given
to question her own political and theoretical position and assumptions even as she reads and criticizes a text. Instead of reading from a situation of
marginality in which the reader's own position and identity are understood to be in process and in dotubt-instead of "reading as a woman -these critical
fragments adopt the attitude of one who is called upon to speak on behalf of a fixed and proudly certain "we," a
"community," a disciplinary center, a modern culture. They adopt the juridical posture of "male response criti- cism."
Regarding the texts as objects ofjudgment, they approach them in a way that privileges the reader as one possessed of a certain identity bound up with an
[CONTINUED]
ENDI 2010 104
Security K Wave 1

AT: CRITICISMS THAT MAKE FUN OF POST-STRUCTURALISM


[CONTINUED]
already- given experience and position that is outside the text and presumably shared with other members of a discipline, a tradition, a point of view. Not
questioning this supposedly pre-given reader's experience, they invoke it as if it provided an authori- tative ground and
standard in terms of which the texts must either prove their merits or be shown to fall short . 3On the question of reading as a woman
see, for example, Felman (1975), Irigaray (1985), Jardine (1985), Kristeva (1975, 1980, 1986ab), Showalter (1979), and Spivak (1980, 1983). An excellent
collection relating the diversity and excitement of Anglo-American feminist replies to the question is Showalter (1985). For introductions of French
Feminist thought to American audiences, see the volumes edited by Marks and Courtivron (1980) and by Eisenstein and Jardine (1980). Jardine (1985)
offers a particularly astute interdisciplinary, intertextual, and inter- cultural analysis of the Franco-American debate in feminist literary theory. Moi (1986)
offers a detailed, theoreti- cally sophisticated, and critical introduction to the problem of the literary text and its relations to the concerns and perspectives
of feminist practice. 372 Reading Dissidence/Writing the Discipline Fourth, in much the same vein, these fragments do not really deserve to be
called arguments or interpretations-not, at least, in a scholarly sense. They do not pre- tend to the status of careful, nuanced
readings of the texts to which they refer. They do not offer themselves as theoretical representations of a referent text, an
offering that would immediately open up not only the question of the faithfulness of the representation to the text but also
the epistemological question of the justification of the position from which representation proceeds. They do not put
themselves forth as profound and thoughtful arguments or knowledge claims whose depths one is invited to mine, whose
assumptions and definitions one is encouraged to examine, or whose logic one is urged critically to explore . Although
those who claim authorship of such critical commentaries on works of dissidence might exhibit a keen concern for
methodological transparency and operational reproducibility in their published research reports to the discipline,
no such concerns are reflected in their critical readings of works at the margin. Simply stated, these critical readings put the
ques- tion of their own truth in suspension, as a question that here and now need not be entertained. Not raising this question, these critical commentaries
instead offer themselves as performances to be appreciated entirely on the surface, in action, in the flow of conduct already underway, and as replies to
challenges of dissidence immediately encountered. They offer themselves as intimately familiar instances of doing without reflecting
on being, as postures of exemplary "men of action" who must pass judg- ment on the exigencies of the times. And like all
such instances, these readings depend for their significance not on a reconstructable logic of argument recurring to
authoritative first principles, but on the way in which they work as cultural practices, as mimesis, as active labors of art.
Their practical significance depends, that is, upon the ways in which they draw upon a variety of metaphors and other cultural re- sources, evoke familiar
orientations or dispositions, and thereby work to cultivate an exemplary and iterable attitude, posture, or style of interpretation and conduct that an
audience will immediately and unquestioningly receive as right for the circum- stances, as "what must be done," as what it, too, is already disposed to do
in answer to the difficulties and dangers posed by dissidence at the margins. Finally, these critical readings put their audience in the
structural situation of a double bind: a situation in which an audience is given the freedom to choose and is called upon to
make a choice even as it is deprived of any basis for doing so.4
ENDI 2010 105
Security K Wave 1

AT: ENVIRONMENTAL SECURITIZATION GOOD

Securitizing the environment hurts their cause in the long term and is politically un-strategic
Waever 1998 [Ole, professor of International Relations at the Department of Political Science, University of Copenhagen,
―Securitization and Desecuritization,‖ On Security, ed. Ronnie Lipschutz,
http://www.ciaonet.org/book/lipschutz/index.html]

These observations point back toward a more general question: Is it a good idea to frame as many problems as possible in terms of
security? Does not such a strategy present the negative prospect of, in a metaphorical sense, militarizing our thinking and seeing problems in terms of
threat-vulnerability-defense, when there are good reasons for not treating them according to this formula? 51 Use of the slogan "environmental
security" is tempting, because it is an effective way of dramatizing environmental problems. In the longer run, however, the
practices resulting from the slogan might lead to an inappropriate social construction of the environment, as a threat/defense
problem. We might find it more constructive, instead, to thematize the problem in terms of an economy-ecology nexus,
where decisions are actually interlinked. 52 Use of the security label does not merely reflect whether a problem is a security problem, it is also
a political choice, that is, a decision for conceptualization in a special way. When a problem is "securitized," the act tends to lead to
specific ways of addressing it: Threat, defense, and often state-centered solutions. This, of course, leaves the environmental
agenda, with its labelling problem, unresolved . One alternative is to view the emerging values of environmentalism as
establishing their own moral basis. As his basis for optimism, for example, Buzan suggests that such values are already emerging as new norms of
international society. 53 Deudney, more lyrically, talks about ecological awareness being linked to "a powerful set of values and symbols" that "draw upon
basic human desires and aspirations," and argues that this, and not regressive security logic, should be the basis for mobilization. 54
ENDI 2010 106
Security K Wave 1

AT: LINK TURNS – AFF STOPS SEEING X AS ENEMY

Securitization leads to the demarcation of us vs. them, spiraling into a neverending cycle of new
enemy creation even when old ones are re-evaluated. This proves we need to rethink the entire
problematic of security
Lipschutz 1998 [Ronnie, prof of politics at UC Santa Cruz, ―Negotiating the Boundaries of Difference and Security at
Millennium's End,‖ On Security, ed. Ronnie Lipschutz, http://www.ciaonet.org/book/lipschutz/index.html]

Security, under these circumstances, is about the drawing and defense of lines and boundaries, about limits, and about exclusion and, in this sense, it is the
quintessential "speech act" described by Ole Wæver. Defining security involves establishing a definition of the collective self vis-à-vis
other collective selves. It is not only about "who is against us," but also, as the observation offered at the beginning of this chapter
suggests, about "who we are" and whom we do not wish to be. It is , to a large degree, about boundaries of difference that are
increasingly difficult to specify and negotiate. Lose an Enemy, Lose Yourself Some years ago, according to a now almost-apocryphal story, a
U.S. diplomat was approached by a Soviet colleague and told, sotto voce , "We are about to do a terrible thing to you. We are going to deprive you of an
enemy." 9 At the time, the story had a certain appealing charm to it: The Soviet Union was the primary threat to, and enemy of, the United States, as forty
years of Cold War had definitively established. Without the Soviet Union as an enemy, a new era in international cooperation could begin. Financial
resources allocated to the defense sector by the two superpowers and their allies could now be redirected to social welfare, basic infrastructure,
technological innovation, and environmental protection. The security dilemma that had resulted in the manufacture of more than 50,000 nuclear weapons,
the deployment of 300,000 American troops and a comparable number of Soviet soldiers in Europe, and the annual global expenditure of close to $1
trillion could be eliminated. A new Concert of states, acting through international institutions, would help to wind down the regional and civil wars
fostered by the East-West conflict. In retrospect, the clarity of those last days of bipolarity, only a few short years ago, was illusory; the Cold War appears
to have been a period of great stability (although this, too, is something of an illusion), inasmuch as the world now seems to be rent by conflict and war to
a degree that would have been difficult to imagine in 1989. These wars and conflicts are, however, largely of a quite unanticipated character: They are
mostly intrastate and social , rather than interstate and political . Today's wars are mostly between literal neighbors, not neighboring states; the security
dilemma has been domesticated and brought into the state (and, in some instances, down to the household level). 10 How can we explain this puzzling
phenomenon? Much of the analysis that currently purports to explain these wars revolves around the concepts of ethnicity and sectarianism :
Increasingly, groups of people are defining themselves collectively, relative to others, in terms of certain shared or acquired
characteristics such as appearance, religion, history, origins, language, and so on. This is not something new, of course; the very ideas of nationalism
and the nation-state are based on such differences. But analyses based on the construction and application of ethnicity generally
ignore the importance of the Other --whom one is not--in fostering the sense of collective identity so important to action
centered on ethnicity or sectarianism. 11 Defining oneself in such terms requires defining someone else in different terms;
differentiation thus draws a boundary between the self and the Other. This Other is not, at first, necessarily a threat in terms
of one's own continued existence, although ethnicity can and does become securitized . 12 But the peaceful acceptance of an
Other requires that boundaries be drawn somewhere else, and that security, the speech act, specify another Other (as
in, for example, South Slavs against the Hapsburgs, or Yugoslavia against the Soviet Union). There are always implicit risks in the peaceful acceptance of
an Other as a legitimate ontology, because doing so raises the possibility, however remote, of accepting the Other's characteristics as a legitimate
alternative and, consequently, of being taken over by the Other. Given this epistemology of threats, it does not take much to be "turned." 13 How else
to account for the life and death character of the distinctions among Serbs, Croats, and Muslims in Bosnia, which the
untutored eye can hardly detect? 14 As James Der Derian puts it in his contribution to this volume, "The desire for security is
manifested as a collective resentment of difference--that which is not us, not certain, not predictable." 15 The loss of an
Enemy can be seen, therefore, as something of a catastrophe for an identity based on that Enemy, and it opens up a search
for a new Other that can function as the new Enemy. And, make no mistake about it: While the myths underlying American identity are
many, during the Cold War the strongest one had to do with not-being, and not-becoming, Communist, both individually and collectively. In a world
dominated by Great Powers and balance-of-power politics, as was the case prior to World War II, losing one enemy was not a problem; there were others
to be found. In the post-bipolar world, the search for enemies and new security threats is less easily solved, inasmuch as the disappearance of the only
Other that counts leaves no other Others that can credibly fill its place.
ENDI 2010 107
Security K Wave 1

AT: LINK TURN – WE ESTABLISH ALLIANCES

Securitization inevitably draws boundaries between self and other—even in the prsence of
alliances, outside threats are carved out as dangerous
Lipschutz 1998 [Ronnie, prof of politics at UC Santa Cruz, ―Negotiating the Boundaries of Difference and Security at
Millennium's End,‖ On Security, ed. Ronnie Lipschutz, http://www.ciaonet.org/book/lipschutz/index.html]

What, then, is security? The contributors to this volume have told us, if nothing else, that it irreducibly involves boundaries. As James Der
Derian points out, it is the drawing of lines between the collective self and what is, in Nietzsche's words, "alien and weaker."
Der Derian argues that "A safe life requires safe truths. The strange and the alien remain unexamined, the unknown
becomes identified as evil, and evil provokes hostility--recycling the desire for security." 22 The boundary between known
and unknown is reified and secured. But where are these boundaries to be drawn? I have suggested above that they are drawn between
the self and the Enemy, between the realm of safety and the realm of danger , between tame zones and wild ones. The practitioners of
national security and security policy conventionally drew these boundaries between states, or between groups of states. By 1989, the roster of states had
been fixed, the books closed for good. There were many "international" boundaries, but these were fixed and all there were or could be. States might
draw imaginary lines, or "bordoids," as Bruce Larkin has stylized them, 23 in defining the parameters of their "national interests." They might extend
their national boundaries in order to incorporate allies, as in practice of extended deterrence in Europe. Enemies and threats
were, however, always across the line.

Securitization relies on threats of annihilation to define and construct the Other—the end of the
Cold War proves that the concept of security does not secure us against all real threats but only
chooses ones based on the violent distancing of difference—the idea that our allies are not threats
to us despite their ability to annihilate us proves the social construction of security
Lipschutz 1998 [Ronnie, prof of politics at UC Santa Cruz, ―Negotiating the Boundaries of Difference and Security at
Millennium's End,‖ On Security, ed. Ronnie Lipschutz, http://www.ciaonet.org/book/lipschutz/index.html]

Nuclear deterrence depended on lines on the ground and in the mind: To be secure, one had to believe that, were the Other
to cross the line, both the self and the Other would cease to exist. The threat of nothingness secured the ontology of
being, but at great political cost to those who pursued this formula . Since 1991, deterrence has ceased to wield its cognitive force,
and the lines in the mind and on the ground have vanished, in spite of repeated efforts to draw them anew. To be sure, the United States and Russia do not
launch missiles against each other because both know the result would be annihilation. But the same is true for France and Britain, or China and Israel. It
was the existence of the Other that gave deterrence its power; it is the disappearance of the Other that has vanquished that
power. Where Russia is now concerned, we are, paradoxically, not secure, because we see no need to be secured. 25 In other
words, as Ole Wæver might put it, where there is no constructed threat, there is no security problem. France is fully capable of
doing great damage to the United States, but that capability has no meaning in terms of U.S. security. The search for new
rationales for security leads, as Beverly Crawford's essay suggests, not to security redefined but to endless iterative loops. To be
secure, we must become more self-reliant, inasmuch as to be reliant means depending on others who are potential Others.
To depend on others means that they are more competitive than we are. To be less competitive means our survival may be
threatened. But to be less reliant means that we forego the fruits of technological collaboration with others. To forego the fruits of collaboration means
that we become less competitive, poorer and less secure than others might be. If we are poorer and less secure, we are more open to
penetration by others, who might well take us over. If we were more like the Japanese, we would be the equal of Japan and
secure; but if we were more like the Japanese, we would be less like Americans and therefore insecure . And so on through this
new Hall of Mirrors. The "new economic security dilemma" is more of a contradiction than a dilemma. While U.S. policymakers fret over competition,
U.S. corporations establish strategic alliances with their Japanese counterparts.
ENDI 2010 108
Security K Wave 1

AT: KRITIK IS IDEOLOGICAL

All theories of international relations are rooted in ideology—its just a question of which is more
epistemologically sound
Walker 2002 [RBJ, professor of political science at the University of Victoria, ―Realism, Change, and International
Political Theory,‖ International Studies Quarterly, Vol. 31, No. 1 (Mar., 1987), pp. 65-86, jstor]

The general problem has a more specific version: all things become other than they were, yet remain somehow enduring. The problem of identity is raised
in terms of time and change. It is with Plato in particular that we conventionally locate the crystallization of a fundamental difference between
metaphysical universals and a realm of becoming, between being and being-in-the-world, the latter having identity and reality only through participation in
the former. This momentous formulation of a radical opposition between eternity and history, between identity and difference in time, continues to haunt
contemporary social and political theory in its search for new horizons. Here structuralism, like the great Cartesian and Kantian
rationalisms before it, has inherited the claim to transtemporal, transspatial abstract universalisms . Structural invariables are
distinguished from the mere succession of events, and, in the more extreme versions we then get a vision of the synchronic
structure of universal mind in which the lived meaning- of history is excluded. Not surprisingly, it is just this lived meaning of
history that is then championed as the alternative ground on which to construct, and reconstruct, a more appropriate account
of human affairs. In some forms, history itself becomes the antithesis of structure; Hegelian or post-Hegelian temporality opposes the atemporal
structuralisms of Kant. In other forms, the stress is on the historically constituted meaning of human experience, and hermeneutics or understanding comes
to oppose the reifying methodologies of positivistic science (von Wright, 1971; Ricoeur, 1978; Gadamer, 1981). More recently, structuralism has
mutated into post-structuralism. The absolute priority of universal structure has given way to an absolute priority of
temporal process, of "trace" and "difference . "9 But whether on the ground of history, meaning of praxis, or of the deconstruction of Western
"logocentricism," modern social and political theory has become intimately concerned with the dilemmas and horizons set up by a discourse about change
organized as a specific form of an opposition between identity and difference. Neither the more arcane intricacies of contemporary debate about structure
and history, nor even the more familiar problems of interpreting long dead political thinkers are usually of much interest to analysts of modern world
politics. Both enterprises seem, and in some senses certainly are, remote and abstract, divorced from the pressing concerns of state policy and global
conflict. Yet in another sense, the very refusal to take the issues that arise from these two contexts seriously is of at least some minor signifi- carice in the
processes through which the "reality" of modern international politics has come to be, and continues to be reproduced. The key issue here is
ideology. Insofar as it is a critical category, rather than a descriptive term, our understanding of ideology is also rooted in
the underlying problematic of identity and difference. The truth of the one is opposed by the illusions of difference, whether of the many or
of the realm of becoming; hence, many of the characteristic moves of ideology-critique. In one direction, the standpoint of identity can be used
to judge the illusory nature of the plural world of change. This is the typical pose of rationalism, structuralism and
positivistic science. Here the analysis of change tends towards a reification into ahistorical and universal laws. In another direction, the claim to
universality is itself challenged as a mere parochialism, whether in space or in time. Here the many critiques of Enlighten- ment science or Marx's critique
of the pretentions of the bourgeois economists are fairly typical. These moves are central to the tension between structuralism and historicism in modern
social and political theory. Structuralist positions generally aspire to scientific status, to ahistorical laws and explanations.
Historicist positions lean towards the categories of hermeneutics and practice. For poststructuralists, the real problem is the
prior framework in which truth and illusion are assumed to guarantee each other .
ENDI 2010 109
Security K Wave 1

******AFF******
ENDI 2010 110
Security K Wave 1

FRAMEWORK – AT: DISCOURSE FIRST

You can only determine the value of policies by their outcomes and not intentions or premises
Waever 1998 [Ole, professor of International Relations at the Department of Political Science, University of Copenhagen,
―Securitization and Desecuritization,‖ On Security, ed. Ronnie Lipschutz,
http://www.ciaonet.org/book/lipschutz/index.html]

From a more Nietzschean perspective, I should also mention that politics always involves an element of exclusion, in which one has to do violence to the
inherent openness of situations, to impose a pattern--and one has not only to remember but also to forget selectively. 77 To act politically means to
take responsibility for leaving an impact, for forcing things in one direction instead of another . Whether such an act is
"good" or "bad" is not defined by any inner qualities of the act or its premises, but by its effects (which depend on the actions of
others, interaction and, therefore, an element of coincidence). As Hannah Arendt pointed out, "Action reveals itself fully only to the storyteller, that is, to
the backward glance of the historian." 78 Acting politically can, consequently, never be risk-free, and "progressiveness" is never
guaranteed by one's political or philosophical attitude. Theoretical practices, as well as any political ones, have to risk their
own respectability and leave traces, letting posterity tell the story about the meaning of an act. Post-structuralists have usually
been arguing that their project is about opening up, implicitly arguing that a situation was too closed, too self-reproducing. Politics is inherently
about closing off options, about forcing the stream of history in particular directions . 79 In the present context, politics and
responsibility can involve prevention and limitation and, at times, the tool of securitization may seem necessary . It is thus not
impossible that a post-structuralist concerned about risks of power rivalry and wars will end up supporting a (re)securitization of "Europe" through
rhetorics such as that of integration/fragmentation. The purpose of this would be to impose limits, but it would have as a side-effect some elements of
state-building linked to the EU project. This could therefore imply that national communities might have to engage in a certain degree of securitization of
identity questions in order to handle the stress from Europeanization. Under such circumstances, there might emerge a complementarity between nations
engaging in societal security and the new quasi-state engaging in "European security." Neither of these two moves are reflections of some objective
"security" that is threatened; they are, instead, possible speech acts , moving issues into a security frame so as to achieve effects different from those that
would ensue if handled in a nonsecurity mode.
ENDI 2010 111
Security K Wave 1

FRAMEWORK – AT: DISCOURSE FIRST

Debate should only include discussions that are policy relevant- their K self maginalizes itself out
of politics and is therefore useless
Joseph Nye, professor at Harvard 2009 University and former dean of the Harvard Kennedy School. , BA suma cum laude Princeton, PhD Harvard,
Former Chair National Intelligence Council, Former Asst. Secretary of Defense for International Security Affairs, you know who he is,
http://www.washingtonpost.com/wp-dyn/content/article/2009/04/12/AR2009041202260_pf.html 4-13

President Obama has appointed some distinguished academic economists and lawyers to his administration, but few high-ranking political
scientists have been named. In fact, the editors of a recent poll of more than 2,700 international relations experts declared that "the walls
surrounding the ivory tower have never seemed so high." While important American scholars such as Henry Kissinger and Zbigniew
Brzezinski took high-level foreign policy positions in the past, that path has tended to be a one-way street. Not many top-ranked
scholars of international relations are going into government, and even fewer return to contribute to academic theory. The 2008 Teaching,
Research and International Policy (TRIP) poll, by the Institute for Theory and Practice in International Relations, showed that of the 25 scholars rated as
producing the most interesting scholarship during the past five years, only three had ever held policy positions (two in the U.S. government and one in the
United Nations). The fault for this growing gap lies not with the government but with the academics. Scholars are paying less attention to
questions about how their work relates to the policy world , and in many departments a focus on policy can hurt one's career. Advancement comes
faster for those who develop mathematical models, new methodologies or theories expressed in jargon that is unintelligible to policymakers.
A survey of articles published over the lifetime of the American Political Science Review found that about one in five dealt with policy prescription or criticism
in the first half of the century, while only a handful did so after 1967. Editor Lee Sigelman observed in the journal's centennial issue that "if 'speaking truth
to power' and contributing directly to public dialogue about the merits and demerits of various courses of action were still
numbered among the functions of the profession, one would not have known it from leafing through its leading journal ." As
citizens, academics might be considered to have an obligation to help improve on policy ideas when they can. Moreover, such
engagement can enhance and enrich academic work, and thus the ability of academics to teach the next generatio n. As former
undersecretary of state David Newsom argued a decade ago, "the growing withdrawal of university scholars behind curtains of
theory and modeling would not have wider significance if this trend did not raise questions regarding the preparation of new
generations and the future influence of the academic community on public and official perceptions of international issues and events.
Teachers plant seeds that shape the thinking of each new generation; this is probably the academic world's most lasting
contribution." Yet too often scholars teach theory and methods that are relevant to other academics but not to the majority of
the students sitting in the classroom before them. Some academics say that while the growing gap between theory and policy may have costs for policy, it has
produced better social science theory, and that this is more important than whether such scholarship is relevant. Also, to some extent, the gap is an inevitable
result of the growth and specialization of knowledge. Few people can keep up with their subfields, much less all of social science. But the danger is that
academic theorizing will say more and more about less and less. Even when academics supplement their usual trickle-down approach to policy
by writing in journals, newspapers or blogs, or by consulting for candidates or public officials, they face many competitors for attention. More than 1,200
think tanks in the United States provide not only ideas but also experts ready to comment or consult at a moment's notice. Some
of these new transmission belts serve as translators and additional outlets for academic ideas, but many add a bias provided by their founders and
funders. As a group, think tanks are heterogeneous in scope, funding, ideology and location, but universities generally offer a more neutral
viewpoint. While pluralism of institutional pathways is good for democracy, the policy process is diminished by the withdrawal of the
academic community. The solutions must come via a reappraisal within the academy itself. Departments should give greater weight to real-
world relevance and impact in hiring and promoting young scholars. Journals could place greater weight on relevance in evaluating submissions.
Studies of specific regions deserve more attention. Universities could facilitate interest in the world by giving junior faculty members greater incentives to
participate in it. That should include greater toleration of unpopular policy positions. One could multiply such useful suggestions, but young
people should not hold their breath waiting for them to be implemented. If anything, the trends in academic life seem to be headed in the opposite direction.
ENDI 2010 112
Security K Wave 1

AT: REPS FIRST


Changing representational practices hinders understanding of policy by overlooking questions
of agency and material structures
Tuathail, 96 (Gearoid, Department of Georgraphy at Virginia Polytechnic Institute, Political Geography, 15(6-7), p. 664,
science direct)

While theoretical debates at academic conferences are important to academics, the discourse and concerns of foreign-policy
decision- makers are quite different, so different that they constitute a distinctive problem- solving, theory-averse, policy-making
subculture. There is a danger that academics assume that the discourses they engage are more significant in the practice of
foreign policy and the exercise of power than they really are . This is not, however, to minimize the obvious importance of academia as a general institutional
structure among many that sustain certain epistemic communities in particular states. In general, I do not disagree with Dalby‘s fourth point about politics and discourse except to
note that his statement-‗Precisely because reality could be represented in particular ways political decisions could be taken, troops and material moved
and war fought‘-evades the important question of agency that I noted in my review essay. The assumption that it is representations that make
action possible is inadequate by itself. Political, military and economic structures, institutions, discursive networks and
leadership are all crucial in explaining social action and should be theorized together with representational practices. Both here and
earlier, Dalby‘s reasoning inclines towards a form of idealism. In response to Dalby‘s fifth point (with its three subpoints), it is worth noting, first, that his book is about the CPD,
not the Reagan administration. He analyzes certain CPD discourses, root the geographical reasoning practices of the Reagan administration nor its public-policy reasoning on national
security. Dalby‘s book is narrowly textual; the general contextuality of the Reagan administration is not dealt with. Second, let me simply note that I find that the distinction between
critical theorists and post- structuralists is a little too rigidly and heroically drawn by Dalby and others. Third, Dalby‘s interpretation of the reconceptualization of national security in
Moscow as heavily influenced by dissident peace researchers in Europe is highly idealist, an interpretation that ignores the structural and ideological crises facing the Soviet elite at
that time. Gorbachev‘s reforms and his new security discourse were also strongly self- interested, an ultimately futile attempt to save the Communist Party and a discredited regime of
power from disintegration. The issues raised by Simon Dalby in his comment are important ones for all those interested in the practice of critical geopolitics. While I agree with
Dalby that questions of discourse are extremely important ones for political geographers to engage, there is a danger of fetishizing this concern with
discourse so that we neglect the institutional and the sociological, the materialist and the cultural, the political and the
geographical contexts within which particular discursive strategies become significant. Critical geopolitics, in other words, should not be a
prisoner of the sweeping ahistorical cant that sometimes accompanies ‗poststructuralism nor convenient reading strategies like the identity politics narrative; it needs to always be
open to the patterned mess that is human history.

A focus on representations destroys social change by ignoring political and material constraints
Taft-Kaufman, 95 (Jill, professor, Department of Speech Communication And Dramatic Arts, at Central Michigan University,
Southern Communication Journal, Spring, proquest)
The postmodern passwords of "polyvocality," "Otherness," and "difference," unsupported by substantial analysis of the concrete contexts of subjects, creates a solipsistic quagmire. The political sympathies of the
despite their adversarial posture and
new cultural critics, with their ostensible concern for the lack of power experienced by marginalized people, aligns them with the political left. Yet,
talk of opposition, their discourses on intertextuality and inter-referentiality isolate them from and ignore the conditions that
have produced leftist politics--conflict, racism, poverty, and injustice. In short, as Clarke (1991) asserts, postmodern emphasis on new subjects conceals the old subjects, those who have limited
access to good jobs, food, housing, health care, and transportation, as well as to the media that depict them. Merod (1987) decries this situation as one which leaves no vision, will, or commitment to activism. He
notes that academic lip service to the oppositional is underscored by the absence of focused collective or politically active intellectual communities. Provoked by the academic manifestations of this problem Di
Leonardo (1990) echoes Merod and laments: Has there ever been a historical era characterized by as little radical analysis or activism and as much radical-chic writing as ours? Maundering on about Otherness:
phallocentrism or Eurocentric tropes has become a lazy academic substitute for actual engagement with the detailed histories and contemporary realities of Western racial minorities, white women, or any Third
World population. (p. 530) Clarke's assessment of the postmodern elevation of language to the "sine qua non" of critical discussion is an even stronger indictment against the trend. Clarke examines Lyotard's (1984)
The Postmodern Condition in which Lyotard maintains that virtually all social relations are linguistic, and, therefore, it is through the coercion that threatens speech that we enter the "realm of terror" and society
I can think of few more striking indicators of the political and intellectual impoverishment of a view
falls apart. To this assertion, Clarke replies:
of society that can only recognize the discursive. If the worst terror we can envisage is the threat not to be allowed to speak, we are appallingly ignorant of terror in its elaborate
contemporary forms. It may be the intellectual's conception of terror (what else do we do but speak?), but its projection onto the rest of the world would be calamitous....(pp. 2-27) The realm of the discursive is
derived from the requisites for human life, which are in the physical world, rather than in a world of ideas or symbols.(4) Nutrition, shelter, and protection are basic human needs that require collective activity for
Postmodern emphasis on the discursive without an accompanying analysis of how the discursive emerges from
their fulfillment.
material circumstances hides the complex task of envisioning and working towards concrete social goals (Merod, 1987). Although the material
conditions that create the situation of marginality escape the purview of the postmodernist, the situation and its consequences are not overlooked by scholars from marginalized groups. Robinson (1990) for
example, argues that "the justice that working people deserve is economic, not just textual" (p. 571). Lopez (1992) states that "the starting point for organizing the program content of education or political action
must be the present existential, concrete situation" (p. 299). West (1988) asserts that borrowing French post-structuralist discourses about "Otherness" blinds us to realities of American difference going on in front
of us (p. 170). Unlike postmodern "textual radicals" who Rabinow (1986) acknowledges are "fuzzy about power and the realities of socioeconomic
constraints" (p. 255), most writers from marginalized groups are clear about how discourse interweaves with the concrete circumstances that create lived experience. People whose lives
form the material for postmodern counter-hegemonic discourse do not share the optimism over the new recognition of their
discursive subjectivities, because such an acknowledgment does not address sufficiently their collective historical and current
struggles against racism, sexism, homophobia, and economic injustice. They do not appreciate being told they are living in a world in which there are no more real
subjects. Ideas have consequences. Emphasizing the discursive self when a person is hungry and homeless represents both a cultural and
humane failure. The need to look beyond texts to the perception and attainment of concrete social goals keeps writers from
marginalized groups ever-mindful of the specifics of how power works through political agendas, institutions, agencies, and the
budgets that fuel them.
ENDI 2010 113
Security K Wave 1

POSITIVISM GOOD

There are no prior questions to problem oriented IR- empirical validity is a sufficient
justification for action. Emphasis on metaphysical hurdles destroys any chance of effectively
describing the world and guiding action
David Owen, Reader of Political Theory at the Univ. of Southampton, Millennium Vol 31 No 3 2002 p. 655-7

Commenting on the ‗philosophical turn‘ in IR, Wæver remarks that ‗[a] frenzy for words like ―epistemology‖ and
―ontology‖ often signals this philosophical turn‘, although he goes on to comment that these terms are often used
loosely.4 However, loosely deployed or not, it is clear that debates concerning ontology and epistemology play a central role in
the contemporary IR theory wars. In one respect, this is unsurprising since it is a characteristic feature of the social sciences
that periods of disciplinary disorientation involve recourse to reflection on the philosophical commitments of different
theoretical approaches, and there is no doubt that such reflection can play a valuable role in making explicit the
commitments that characterise (and help individuate) diverse theoretical positions. Yet, such a philosophical turn is not
without its dangers and I will briefly mention three before turning to consider a confusion that has, I will suggest, helped
to promote the IR theory wars by motivating this philosophical turn. The first danger with the philosophical turn is that it
has an inbuilt tendency to prioritise issues of ontology and epistemology over explanatory and/or
interpretive power as if the latter two were merely a simple function of the former. But while the
explanatory and/or interpretive power of a theoretical account is not wholly independent of its ontological
and/or epistemological commitments (otherwise criticism of these features would not be a criticism that had any value), it is
by no means clear that it is, in contrast, wholly dependent on these philosophical commitments. Thus, for
example, one need not be sympathetic to rational choice theory to recognise that it can provide powerful accounts of certain
kinds of problems, such as the tragedy of the commons in which dilemmas of collective action are foregrounded. It may, of
course, be the case that the advocates of rational choice theory cannot give a good account of why this
type of theory is powerful in accounting for this class of problems (i.e., how it is that the relevant actors come to exhibit
features in these circumstances that approximate the assumptions of rational choice theory) and, if this is the case, it is
a philosophical weakness—but this does not undermine the point that, for a certain class of
problems, rational choice theory may provide the best account available to us. In other words,
while the critical judgement of theoretical accounts in terms of their ontological and/or
epistemological sophistication is one kind of critical judgement, it is not the only or even necessarily
the most important kind. The second danger run by the philosophical turn is that because prioritisation of
ontology and epistemology promotes theory-construction from philosophical first principles, it cultivates a
theory-driven rather than problem-driven approach to IR. Paraphrasing Ian Shapiro, the point can
be put like this: since it is the case that there is always a plurality of possible true descriptions of a given action,
event or phenomenon, the challenge is to decide which is the most apt in terms of getting a perspicuous
grip on the action, event or phenomenon in question given the purposes of the inquiry; yet, from this standpoint,
‗theory-driven work is part of a reductionist program’ in that it ‗dictates always opting for the
description that calls for the explanation that flows from the preferred model or theory‘.5 The
justification offered for this strategy rests on the mistaken belief that it is necessary for social science
because general explanations are required to characterise the classes of phenomena studied in similar
terms. However, as Shapiro points out, this is to misunderstand the enterprise of science since
‗whether there are general explanations for classes of phenomena is a question for social-scientific
inquiry, not to be prejudged before conducting that inquiry‘.6 Moreover, this strategy easily slips into
the promotion of the pursuit of generality over that of empirical validity. The third danger is that the
preceding two combine to encourage the formation of a particular image of disciplinary debate in IR—what might
be called (only slightly tongue in cheek) ‗the Highlander view‘—namely, an image of warring theoretical
approaches with each, despite occasional temporary tactical alliances, dedicated to the strategic achievement of
sovereignty over the disciplinary field. It encourages this view because the turn to, and prioritisation of, ontology
and epistemology stimulates the idea that there can only be one theoretical approach
which gets things right, namely, the theoretical approach that gets its ontology and epistemology right. This
image feeds back into IR exacerbating the first and second dangers, and so a potentially vicious
circle arises.
ENDI 2010 114
Security K Wave 1

POSITIVISM GOOD

Positivism is essential to any epistemology—relativism fails to define and address real problems
GEORG SØRENSEN, MA, political science (1975) PhD, social science (1983) (Trans national Corporations and Economic Development) Dr.scient.pol.
(1993) (Democracy and Development.) Professor, International Politics and Economics, Aarhus Univ. Review of International Studies 24, 1998 ,IR Theory
after the cold war p. 87-88

What, then, are the more general problems with the extreme versions of the postpositivist position? The first problem is that they tend to overlook, or downplay,
the actual insights produced by non-post-positivists, such as, for example, neorealism. It is entirely true that anarchy is no given, ahistorical, natural condition to
which the only possible reaction is adaptation. But the fact that anarchy is a historically specific, socially constructed product of human practice
does not make it less real. In a world of sovereign states, anarchy is in fact out there in the real world in some form. In other
words, it is not the acceptance of the real existence of social phenomena which produces objectivist reification . Reification is
produced by the transformation of historically specific social phenomena into given, ahistorical, natural conditions.21 Despite their
shortcomings, neorealism and other positivist theories have produced valuable insights about anarchy, including the factors in play in balance-of-power dynamics and in
patterns of cooperation and conflict. Such insights are downplayed and even sometimes dismissed in adopting the notion of 'regimes of truth'.
It is, of course, possible to appreciate the shortcomings of neorealism while also recognizing that it has merits. One way of doing so is set forth by Robert Cox. He considers neorealism to be a
'problem-solving theory' which 'takes the world as it finds it, with the prevailing social and power relationships . . . as the given framework for action . . . The strength of the
problem-solving approach lies in its ability to fix limits or parameters to a problem area and to reduce the statement of a particular
problem to a limited number of variables which are amenable to relatively close and precise examination' .22 At the same time, this 'assumption
of fixity' is 'also an ideological bias . . . Problem-solving theories (serve) . . . particular national, sectional or class interests, which are comfortable within the given order'.23 In sum, objectivist theory such
as neorealism contains a bias, but that does not mean that it is without merit in analysing particular aspects of international relations from a particular point of view.
The second problem with post-positivism is the danger of extreme relativism which it contains. If there are no neutral grounds for deciding
about truth claims so that each theory will define what counts as the facts, then the door is, at least in principle, open to anything goes. Steve Smith has confronted this problem
in an exchange with Øyvind Østerud. Smith notes that he has never 'met a postmodernist who would accept that "the earth is flat if you say so". Nor has any postmodernist I have read argued or implied that "any
narrative is as good as any other"'.24 But the problem remains that if we cannot find a minimum of common standards for deciding about truth claims a post-
modernist position appears unable to come up with a metatheoretically substantiated critique of the claim that the earth is flat. In
the absence of at least some common standards it appears difficult to reject that any narrative is as good as any other.25 The final problem with extreme post-positivism I wish to address here
concerns change. We noted the post-modern critique of neorealism's difficulties with embracing change; their emphasis is on 'continuity and repetition'. But extreme post-positivists have their own
problem with change, which follows from their metatheoretical position. In short, how can post-positivist ideas and projects of change be distinguished from
pure utopianism and wishful thinking? Post-positivist radical subjectivism leaves no common ground for choosing between different
change projects. A brief comparison with a classical Marxist idea of change will demonstrate the point I am trying to make. In Marxism, social change ( e.g. revolution) is, of course, possible. But that
possibility is tied in with the historically specific social structures (material and non-material) of the world. Revolution is possible under certain social conditions but not
under any conditions. Humans can change the world, but they are enabled and constrained by the social structures in which
they live. There is a dialectic between social structure and human behaviour .26 The understanding of 'change' in the Marxist tradition is thus closely related to an
appreciation of the historically specific social conditions under which people live; any change project is not possible at any time. Robert Cox makes a similar point in writing about critical theory: 'Critical
theory allows for a normative choice in favor of a social and political order different from the prevailing order, but it limits the
range of choice to alternative orders which are feasible transformations of the existing world . . . Critical theory thus contains an
element of utopianism in the sense that it can represent a coherent picture of an alternative order, but its utopianism is
constrained by its comprehension of historical processes. It must reject improbable alternatives just as it rejects the permanency of the existing order'.27 That
constraint appears to be absent in post-positivist thinking about change, because radical post-positivism is epistemologically
and ontologically cut off from evaluating the relative merit of different change projects. Anything goes, or so it seems. That view is hard to
distinguish from utopianism and wishful thinking. If neorealism denies change in its overemphasis on continuity and repetition, then radical post-positivism is metatheoretically compelled to embrace any
conceivable change project.28
ENDI 2010 115
Security K Wave 1

AT: SCENARIO PLANNING BAD

Their alternative fails- security as a specific concept can’t be deconstructed. The


ethical response is to engage in scenario planning and try to minimize violence
Ole Weaver, 2k International relations theory and the politics of European integration, p. 284-285

The other main possibility is to stress' responsibility. Particularly in a field like security one has to make choices a
nd deal with the challenges and risks that one confronts – and not shy away into long-range or principled trans-
formations. The meta political line risks (despite the theoretical commitment to the concrete other) implying that politics
can be contained within large 'systemic questions . In line with he classical revolutionary tradition, after the change (now
no longer the revolution but the meta-physical transformation), there will be no more problems whereas in our situation
(until the change) we should not deal with the 'small questions' of politics, only with the large one (cf. Rorty 1996).
However, the ethical demand in post-structuralism (e.g. Derrida's 'justice') is of a kind that can never be instantiated in
any concrete political order – It is an experience of the undecidable that exceeds any concrete solution and
reinserts politics. Therefore, politics can never be reduced to meta-questions there is no way to erase the small,
particular, banal conflicts and controversies. In contrast to the quasi-institutionalist formula of radical democracy which
one finds in the 'opening' oriented version of deconstruction, we could with Derrida stress the singularity of the event. To take a
position, take part, and 'produce events' (Derrida 1994: 89) means to get involved in specific struggles. Politics takes place 'in the
singular event of engagement' (Derrida 1996: 83). Derrida's politics is focused on the calls that demand
response/responsibility contained in words like justice, Europe and emancipation. Should we treat security in this
manner? No, security is not that kind of call. 'Security' is not a way to open ( or keep open) an ethical horizon.
Security is a much more situational concept oriented to the handling of specifics. It belongs to the sphere of how
to handle challenges – and avoid 'the worst' (Derrida 1991). Here enters again the possible pessimism which for the
security analyst might be occupational or structural. The infinitude of responsibility (Derrida 1996: 86) or the tragic nature
of politics (Morgenthau 1946, Chapter 7) means that one can never feel reassured that by some 'good deed', 'I have assumed my
responsibilities ' (Derrida 1996: 86). If I conduct myself particularly well with regard to someone, I know that it is to the detriment of
an other; of one nation to the detriment of my friends to the detriment of other friends or non-friends, etc. This is the
infinitude that inscribes itself within responsibility; otherwise there would he no ethical problems or decisions. (ibid.; and
parallel argumentation in Morgenthau 1946; Chapters 6 and 7) Because of this there will remain conflicts and risks -
and the question of how to handle them. Should developments be securitized (and if so, in what terms)? Often,
our reply will be to aim for de-securitization and then politics meet meta-politics; but occasionally the
underlying pessimism regarding the prospects for orderliness and compatibility among human aspirations will
point to scenarios sufficiently worrisome that responsibility will entail securitization in order to block the worst.
As a security/securitization analyst, this means accepting the task of trying to manage and avoid spirals and
accelerating security concerns, to try to assist in shaping the continent in a way that creates the least insecurity
and violence - even if this occasionally means invoking/producing `structures' or even using the dubious
instrument of securitization. In the case of the current European configuration, the above analysis suggests the use of
securitization at the level of European scenarios with the aim of preempting and avoiding numerous instances of local
securitization that could lead to security dilemmas and escalations, violence and mutual vilification.
ENDI 2010 116
Security K Wave 1

AT: PREDICTIONS FAIL


Turn—rejecting strategic predictions of threats makes them inevitable—decisionmakers will rely
on preconceived conceptions of threat rather than the more qualified predictions of analysts
Fitzsimmons, 07 (Michael, Washington DC defense analyst, ―The Problem of Uncertainty in Strategic Planning‖, Survival,
Winter 06-07, online)

But handling even this weaker form of uncertainty is still quite challeng- ing. If not sufficiently bounded, a high degree of
variability in planning factors can exact a significant price on planning. The complexity presented by great variability strains the
cognitive abilities of even the most sophisticated decision- makers.15 And even a robust decision-making process sensitive to
cognitive limitations necessarily sacrifices depth of analysis for breadth as variability and complexity grows. It should follow,
then, that in planning under conditions of risk, variability in strategic calculation should be carefully tailored to available
analytic and decision processes. Why is this important? What harm can an imbalance between complexity and cognitive or
analytic capacity in strategic planning bring? Stated simply, where analysis is silent or inadequate, the personal beliefs of
decision-makers fill the void. As political scientist Richard Betts found in a study of strategic sur- prise, in ‗an environment
that lacks clarity, abounds with conflicting data, and allows no time for rigorous assessment of sources and validity, ambiguity
allows intuition or wishfulness to drive interpretation ... The greater the ambiguity, the greater the impact of preconceptions.‘16
The decision-making environment that Betts describes here is one of political-military crisis, not long-term strategic planning.
But a strategist who sees uncertainty as the central fact of his environ- ment brings upon himself some of the pathologies of
crisis decision-making. He invites ambiguity, takes conflicting data for granted and substitutes a priori scepticism about the
validity of prediction for time pressure as a rationale for discounting the importance of analytic rigour. It is important not to
exaggerate the extent to which data and ‗rigorous assessment‘ can illuminate strategic choices. Ambiguity is a fact of life, and
scepticism of analysis is necessary. Accordingly, the intuition and judgement of decision-makers will always be vital to strategy,
and attempting to subordinate those factors to some formulaic, deterministic decision-making model would be both
undesirable and unrealistic. All the same, there is danger in the opposite extreme as well. Without careful analysis of what is
relatively likely and what is relatively unlikely, what will be the possible bases for strategic choices? A decision-maker with no
faith in prediction is left with little more than a set of worst-case scenarios and his existing beliefs about the world to confront
the choices before him. Those beliefs may be more or less well founded, but if they are not made explicit and subject to
analysis and debate regarding their application to particular strategic contexts, they remain only beliefs and premises, rather than
rational judgements. Even at their best, such decisions are likely to be poorly understood by the organisations charged with
their implementation. At their worst, such decisions may be poorly understood by the decision-makers themselves.
ENDI 2010 117
Security K Wave 1

2AC CEDE THE POLITICAL

Their alternative cedes the political- Weimar proves


Lord William Wallace, 1996 of Baron AND Saltaire, PhD Cornell, Former IR Prof London School of Economics, Total Badass, Review of international Studies (22)
The failure of the Weimar Republic to establish its legitimacy owed something to the irresponsibility of intellectuals of the right and left,
preferring the private certainties of their ideological schools to critical engagement with the difficult compromises of democratic
politics. The Frankfurt School of Adorno and Marcuse were Salonbolschewisten, 'relentless in their hostility towards the capitalist system' while 'they never abandoned the
lifestyle of the haute bourgeoisie'?x The followers of Nietzsche on the right and those of Marx on the left both worked to denigrate the
limited achievements and the political compromises of Weimar, encouraging their students to adopt their own radically
critical positions and so contribute to undermining the republic . Karl Mannheim, who had attempted in Ideology and Utopia to build on
Weber's conditional and contingent sociology of knowledge, was among the first professors dismissed when the Nazis came to
power. Intellectuals who live within relatively open civil societies have a responsibility to the society within which they live: to
act themselves as constructive critics, and to encourage their students to contribute to the strengthening of civil society rather
than to undermine it.32 (308-9)
ENDI 2010 118
Security K Wave 1

AT: STATE LINKS

The neg essentializes states with infinite unpredictability and unique vulnerabilities
Buzan 1998 [Barry, professor of IR at the London School of Economics, ―Security, the State, the "New World Order," and
Beyond,‖ On Security, ed. Ronnie Lipschutz, http://www.ciaonet.org/book/lipschutz/index.html]

There can be no question that the object we refer to when we use the term "state" is not fixed in character. The essential meaning of the
term refers to autonomous, territorially organized political entities in which the machinery of government is in some sense recognizably separate from the
organization of society. States are distinguished from tribes and other less complex forms of "stateless societies" by this differentiation of the political
from the societal. Within this definition lies a very wide array of possible sociopolitical constructions, though there will always
be a relationship between rulers and subjects, a territorial domain of some sort, and a societal realm (or realms) as well as a
political one. In most, but not all, cases there will also be relations with other autonomous political entities. Where this is so,
states will face security problems arising from the interplay of threats and vulnerabilities among them. Beyond these basics,
there can be virtually infinite diversity in how the internal components of the state are constructed and arranged . The
relationship between rulers and subjects can range from remote and detached at one end of the spectrum (imperial China, Tsarist Russia), to close and
strongly connected at the other (contemporary Scandinavia). Similarly, the structures of government and society may be quite distinct (Chinese-controlled
Tibet) or tightly interwoven (USA). It is uncommon, except in colonies, to have a complete divorce between government and society, but very common
for large sections of society to be alienated from the government, as in Sudan, Pakistan, Afghanistan, Sri Lanka, Canada, Iraq, Turkey and many others.
Government, society, and territory can have long and deep connections (Japan), or they can be superficial and ephemeral (Jordan, Yugoslavia, Chad). How
society, government and territory are organized depends heavily on the nature of the prevailing social and material technologies, and on the relationship
between the holders of coercive power and the holders of capital. 1 It may also depend on how the state came into being: in Giddens's typology, whether it
is classical (France), colonized (United States), postcolonial (Nigeria) or modernizing (Japan). 2

Abandoning the state worse than endorsing it


Michael Walzer, 2010 BA Brandeis, PhD Harvard, ,Response to: The Military State of America and the Democratic Left, Dissent Vol. 57 No 1, Winter

He thinks that the most important thing happening in the world today is the growth of international institutions and the erosion
of state sovereignty. But, again, he never tells us how or where this is "happening." Right now, no political leader in his right mind
would entrust the safety of his people to any international organization. Right now, what the most oppressed and impoverished
people in the world most urgently need is a decent and effective sovereign state. This is obviously true for Tibetans, Kurds, and
Palestinians, but it is also true of people across Africa and in many parts of Asia, living in failed states, ruled by warlords,
without effective police, without welfare systems or functioning schools. In the world as it is today, only sovereign states can
provide these services. But Jim wants to get past state sovereignty. Maybe one day there will be pleasant pastures "beyond" the
state. Right now, there is only a wasteland.
ENDI 2010 119
Security K Wave 1

AT: THREAT CONSTRUCTION

Literature and psychological bias runs towards threat deflation- we are the opposite of paranoid
Schweller 4 [Randall L. Schweller, Associate Professor in the Department of Political Science at The Ohio State University, ―Unanswered Threats A
Neoclassical RealistTheory of Underbalancing,‖ International Security 29.2 (2004) 159-201, Muse]

Despite the historical frequency of underbalancing, little has been written on the subject . Indeed, Geoffrey Blainey's memorable
observation that for "every thousand pages published on the causes of wars there is less than one page directly on the causes of peace" could have been made
with equal veracity about overreactions to threats as opposed to underreactions to them.92 Library shelves are filled with books on the causes and
dangers of exaggerating threats, ranging from studies of domestic politics to bureaucratic politics, to political psychology, to organization theory. By
comparison, there have been few studies at any level of analysis or from any theoretical perspective that directly explain why states have
with some, if not equal, regularity underestimated dangers to their survival . There may be some cognitive or normative bias at
work here. Consider, for instance, that there is a commonly used word, paranoia, for the unwarranted fear that people are, in some way,
"out to get you" or are planning to do oneharm. I suspect that just as many people are afflicted with the opposite psychosis: the delusion
that everyone loves you when, in fact, they do not even like you. Yet, we do not have a familiar word for this phenomenon. Indeed, I am
unaware of any word that describes this pathology (hubris and overconfidence come close, but they plainly define something other than what I have described).
That noted, international relations theory does have a frequently used phrase for the pathology of states' underestimation of threats to
their survival, the so-called Munich analogy. The term is used, however, in a disparaging way by theorists to ridicule those who employ it. The central
claim is that the naïveté associated with Munich and the outbreak of World War II has become an overused and inappropriate analogy because few leaders are
as evil and unappeasable as Adolf Hitler. Thus, the analogy either mistakenly causes leaders [End Page 198] to adopt hawkish and overly competitive policies or
is deliberately used by leaders to justify such policies and mislead the public. A more compelling explanation for the paucity of studies on
underreactions to threats, however, is the tendency of theories to reflect contemporary issues as well as the desire of theorists and
journals to provide society with policy- relevant theories that may help resolve or manage urgent security problems . Thus, born in
the atomic age with its new balance of terror and an ongoing Cold War, the field of security studies has naturally produced theories of and
prescriptions for national security that have had little to say about —and are, in fact, heavily biased against warnings of—the
dangers of underreacting to or underestimating threats. After all, the nuclear revolution was not about overkill but, as Thomas Schelling pointed
out, speed of kill and mutual kill.93 Given the apocalyptic consequences of miscalculation, accidents, or inadvertent nuclear war, small wonder
that theorists were more concerned about overreacting to threats than underresponding to them . At a time when all of humankind
could be wiped out in less than twenty-five minutes, theorists may be excused for stressing the benefits of caution under conditions of uncertainty and erring on
the side of inferring from ambiguous actions overly benign assessments of the opponent's intentions. The overwhelming fear was that a crisis "might unleash
forces of an essentially military nature that overwhelm the political process and bring on a war thatnobody wants. Many important conclusions about the risk of
nuclear war, and thus about the political meaning of nuclear forces, rest on this fundamental idea."94 Now that the Cold War is over, we can begin to redress
these biases in the literature. In that spirit, I have offered a domestic politics model to explain why threatened states often fail to adjust in a prudent and
coherent way to dangerous changes in their strategic environment. The model fits nicely with recent realist studies on imperial under- and overstretch.
Specifically, it is consistent with Fareed Zakaria's analysis of U.S. foreign policy from 1865 to 1889, when, he claims, the United States had the national power
and opportunity to expand but failed to do so because it lacked sufficient state power (i.e., the state was weak relative to society).95 Zakaria claims that the
United States did [End Page 199] not take advantage of opportunities in its environment to expand because it lacked the institutional state strength to harness
resources from society that were needed to do so. I am making a similar argument with respect to balancing rather than expansion: incoherent, fragmented
states are unwilling and unable to balance against potentially dangerous threats because elites view the domestic risks as too high, and they are unable to
mobilize the required resources from a divided society. The arguments presented here also suggest that elite fragmentation and disagreement within a
competitive political process, which Jack Snyder cites as an explanation for overexpansionist policies, are more likely to produce
underbalancing than overbalancing behavior among threatened incoherent states.96 This is because a balancing strategy carries certain political
costs and risks with few, if any, compensating short-term political gains, and because the strategic environment is always
somewhat uncertain. Consequently, logrolling among fragmented elites within threatened states is more likely to generate overly cautious responses to
threats than overreactions to them. This dynamic captures the underreaction of democratic states to the rise of Nazi Germany during the interwar period.97 In
addition to elite fragmentation, I have suggested some basic domestic-level variables that regularly intervene to thwart balance of power predictions.
ENDI 2010 120
Security K Wave 1

AT: PSYCHOLOGY LINKS

Their psychology claims are nonsense- can‘t explain the actual track record of war
Kagan, 1985 [David, Former Dean of Yale, currently Sterling Professor of Classics and History at Yale MA Brown, PhD Ohio
State, NATIONAL AFFAIRS FROM ISSUE NUMBER 78 - WINTER 1985]

This essay is typical in its rejection of the particular in favor of the general , in its search for unconscious and uncontrolled
motives to the neglect of the conscious and purposeful motives of people, in its rejection of the willed and rational in favor of the
non-rational or irrational. Its authors make no claim to originality, and allow that their theory may seem to be nothing more than enlightened common
sense. But in fact it is an attack on common sense. The reader may have thought wars were the results of ignorance, bad character,
or evil intentions—I mean fear, suspicion, greed, jealousy, and hatred—but they are really the results of inevitable ambivalence leading to
morbid anxiety which is transformed into displaced or projected aggression, over all of which the individual has no control. And
if this seems arcane, the reader can fall back on the real teaching of this essay—that wars are caused by various things. Like the
other scientizing students of society, the authors leave the crucial questions not only unanswered but unasked. Presumably,
people are always repressing, displacing, and projecting their aggressive feelings. Why do these processes lead to war at some times
and not at others? And if we leave cloudcuckooland for a moment, we soon recognize that the decisions that produce wars are
not made by great masses of psychologically disturbed people but by a small number of leaders charged with the conduct of
foreign affairs. A proper theory ought to consider what moves such people to bring on war. If they are moved by forces such as those I
have discussed, it should be possible to show that they are, and to show how they are. If they merely respond to popular feeling, we should be
told precisely how, when, and why they do. Statesmen who make decisions usually offer reasoned explanations for their actions.
If the stated reasons are false, as they often are, should we not seek unstated reasons before abandoning reason altogether? (p. 50-
51)
ENDI 2010 121
Security K Wave 1

AT: MIDDLE EAST LINK

Multiple factors make critical theory the wrong approach for the middle east
Bilgin, 2005 [Pinar, PhD International Politics, University of Wales, Aberystwyth, Department of International Relations
Bilkent Univ., Regional Security in the Middle East p. 9-10]

The Middle East is arguably a hard case for critical approaches to engage in. It has for long been
viewed as a region that 'best fits the realist view of international politics' (Nye 2000:163); or 'an
―exceptional‖ case eternally out of step with history and immune to trends affecting other parts of the world' (Aarts 1999:911). 'While the
rest of the world worries about new and non-traditional threats to national security', argued one author, most
countries in the Middle East are still poised to counter the same old, traditional threats . In the Middle East,
to use Thomas Hobbes's famous line, 'there is continuall feare, and danger of violent death' and 'the life of man' (and woman) is still 'poore,
nasty, brutish and short'. The Cold War has had a revolutionary impact on the security agenda of most states in the world, with the exception
of the Middle East. (Shehadi 1998:134)
Accordingly, it has been argued that whereas critical approaches to security may have relevance within the
Western European context, in other parts of the world - such as the Middle East - more traditional
approaches retain their validity (see Ayoob 1995:8-12). The Gulf War (1990-91), the US-led war on Iraq (2003),
the stall in Arab-Israeli peace-making and the seeming lack of enthusiasm for addressing the problem of regional insecurity,
especially when viewed against the backdrop of increasing regionalisation in security relations in other
parts of the world (see, for example, Rosecrance 1991; Hettne and Inotai 1994; Alagappa 1995; Fawcett and Hurrell 1995; Gamble and
Payne 1996; Lake and Morgan 1997; Hettne and Söderbaum 1998), does indeed suggest that the Middle East is a place
where traditional conceptions and practices of security still prevail.

SQ Failures result from a failure to utilize realism- the plan moves us back towards the right track
Leverett 06 [Flynt, a senior fellow and the director of the geopolitics-of-energy initiative at the New America Foundation. He is also
a visiting professor of political science at MIT. American Prospect Sept

The basic flaw in the Bush administration's Middle East strategy is that it departs from the essential
propositions of foreign-policy realism. In his days as the principal architect of American foreign policy under Presidents
Richard Nixon and Gerald Ford, Henry Kissinger established a paradigm for U.S. grand strategy in the Middle
East. In this paradigm, American policy should seek always to empower moderates and marginalize
radicals. The best way to do this was through careful management of the region's balance of power,
primarily through diplomatic means. The essence of such diplomacy is "carrots-and-
sticks" engagement--credibly threatening negative consequences for regional actors who work against U.S. goals, but also
promising strategically significant benefits in exchange for cooperation. This paradigm guided U.S. policy in the Middle
East throughout Kissinger's tenure in office and through subsequent administrations. At the height of
the Cold War, for example, the realist paradigm guided American efforts across three administrations to
draw Egypt out of its alliance with the Soviet Union and into a strategic partnership with the United
States, which provided subsequent administrations a dramatically improved platform for projecting
political influence and, when necessary, military power in the region. By taking Egypt out of the Arab-Israeli military
equation through the U.S.-brokered Camp David accords in 1978, the realist paradigm also fundamentally strengthened
Israel's security by rendering impossible a recurrence of a generalized Arab-Israeli war like those of 1948,1967,
and 1973. Similar logic animated America's ongoing strategic partnership with Saudi Arabia and, after the first Gulf War, the launch of a
more comprehensive Arab-Israeli peace process at the 1991 Madrid conference. Although the Clinton administration's efforts to broker peace
treaties between Israel and the Palestinians and Israel and Syria in the late 1990s proved unsuccessful, the peace process
nonetheless bolstered the American and Israeli positions in the region by establishing conceptual
frameworks for an ultimate resolution of the Arab-Israeli conflict. It also provided a practical
framework for keeping a lid on "hot spots" such as southern Lebanon and, as a result of Israeli-Palestinian security
cooperation in the late 1990s, significantly reducing the incidence of anti-Israeli terrorism by groups such as Hamas and Islamic Jihad.
ENDI 2010 122
Security K Wave 1

AT: TERROR LINK

Believing that a nuclear WMD terrorist attack is possible is necessary to prevent it from
happening
Allison, 10 – professor of government and director of the Belfer Center for Science and International Affairs at Harvard (1/25/10, Graham, ―A Failure to
Imagine the Worst: The first step toward preventing a nuclear 9/11 is believing it could happen,‖
http://www.foreignpolicy.com/articles/2010/01/25/a_failure_to_imagine_the_worst?print=yes&hidecomments=yes&page=full, JMP)

In his first speech to the U.N. Security Council, U.S. President Barack Obama challenged members to think about the impact of a
single nuclear bomb.He said: "Just one nuclear weapon exploded in a city -- be it New York or Moscow, Tokyo or Beijing, London or Paris -- could kill
hundreds of thousands of people." The consequences, he noted, would "destabilize our security, our economies, and our very way of life." Before the Sept. 11,
2001, assault on the World Trade Center and Pentagon, who could have imagined that terrorists would mount an attack on the American homeland that would
kill more citizens than Japan did at Pearl Harbor? As then-Secretary of State Condoleezza Rice testified to the 9/11 Commission: "No one could have imagined
them taking a plane, slamming it into the Pentagon ... into the World Trade Center, using planes as missiles." For most Americans, the idea of international
terrorists conducting a successful attack on their homeland, killing thousands of citizens, was not just unlikely. It was inconceivable. As is now evident,
assertions about what is "imaginable" or "conceivable," however, are propositions about our minds, not about what is
objectively possible. Prior to 9/11, how unlikely was a megaterrorist attack on the American homeland? In the previous decade, al Qaeda attacks on the
World Trade Center in 1993, U.S. embassies in Kenya and Tanzania in 1998, and the USS Cole in 2000 had together killed almost 250 and injured nearly 6,000.
Moreover, the organization was actively training thousands of recruits in camps in Afghanistan for future terrorist operations. Thinking about risks we face
today, we should reflect on the major conclusion of the bipartisan 9/11 Commission established to investigate that catastrophe. The U.S. national security
establishment's principal failure prior to Sept. 11, 2001, was, the commission found, a "failure of imagination." Summarized in a single
sentence, the question now is: Are we at risk of an equivalent failure to imagine a nuclear 9/11? After the recent attempted terrorist attack on Northwest
Airlines Flight 253, this question is more urgent than ever. The thought that terrorists could successfully explode a nuclear bomb in an American city killing
hundreds of thousands of people seems incomprehensible. This essential incredulity is rooted in three deeply ingrained presumptions. First, no one could
seriously intend to kill hundreds of thousands of people in a single attack. Second, only states are capable of mass destruction; nonstate actors would be unable
to build or use nuclear weapons. Third, terrorists would not be able to deliver a nuclear bomb to an American city. In a nutshell, these presumptions lead to the
conclusion: inconceivable. Why then does Obama call nuclear terrorism "the single most important national security threat that we
face" and "a threat that rises above all others in urgency?" Why the unanimity among those who have shouldered responsibility for U.S. national
security in recent years that this is a grave and present danger? In former CIA Director George Tenet's assessment, "the main threat is the nuclear one. I am
convinced that this is where [Osama bin Laden] and his operatives desperately want to go." When asked recently what keeps him awake at night, Secretary of
Defense Robert Gates answered: "It's the thought of a terrorist ending up with a weapon of mass destruction, especially nuclear." Leaders who have
reached this conclusion about the genuine urgency of the nuclear terrorist threat are not unaware of their skeptics'
presumptions. Rather, they have examined the evidence, much of which has been painstakingly compiled here by Rolf Mowatt-
Larssen, former head of the CIA's terrorism and weapons-of-mass-destruction efforts, and much of which remains classified. Specifically, who is
seriously motivated to kill hundreds of thousands of Americans? Osama bin Laden, who has declared his intention to kill "4 million Americans -- including 2
million children." The deeply held belief that even if they wanted to, "men in caves can't do this" was then Pakistani President Pervez Musharraf's view when
Tenet flew to Islamabad to see him after 9/11. As Tenet (assisted by Mowatt-Larssen) took him step by step through the evidence, he discovered that indeed
they could. Terrorists' opportunities to bring a bomb into the United States follow the same trails along which 275 tons of drugs and 3 million people crossed
U.S. borders illegally last year. In 2007, Congress established a successor to the 9/11 Commission to focus on terrorism using
weapons of mass destruction. This bipartisan Commission on the Prevention of WMD Proliferation and Terrorism issued its report to Congress and the
Obama administration in December 2008. In the commission's unanimous judgment: "it is more likely than not that a weapon of
mass destruction will be used in a terrorist attack somewhere in the world by the end of 2013." Faced with the possibility of an
American Hiroshima, many Americans are paralyzed by a combination of denial and fatalism. Either it hasn't happened, so it's
not going to happen; or, if it is going to happen, there's nothing we can do to stop it. Both propositions are wrong. The
countdown to a nuclear 9/11 can be stopped, but only by realistic recognition of the threat, a clear agenda for action,
and relentless determination to pursue it.
ENDI 2010 123
Security K Wave 1

2AC IMPACT CALC - CONSEQUENCES FIRST


Personal pacific beliefs have no place in policy making circles-consequences key
Lewy, 2005, [Guenter has been on the faculties of Columbia University, Smith College, and the University of Massachusetts Pacifism and Citizenship- Can
they Coexist?]

Pacifists, when truly and consistently committed to the supreme value of nonviolence, remind the rest of us who are not pacifists of the
link between means and ends. Their personal "No!" to killing carries an important ethical message. The pacifist vision of a world free of the threat of war
can help build support for the development of an ordered community at the international level that is able to resolve conflicts peacefully and justly. However,
the moment the pacifist enters the political arena to seek to influence the policies of his nation, he ceases to speak as a pacifist
and becomes subject to what Weber called the "ethic of responsibility," which takes account of the realities of power and the
likely consequences of political decisions. The personal "No!" of the pacifist, representing an act of conscience, is morally
unassailable if this act of refusal does not jeopardize the well-being of others. In view of the fact that pacifists are usually a small minority of a country's
population, this condition is satisfied in most cases. On the other hand, the national policies proposed by pacifists should , like all policies, be
judged in terms of foreseeable results. As George Weigel has correctly pointed out, "The morality of political judgment must include
a consequential criterion. To argue, for example, that unilateral disarmament is the sole moral option, even if its results would be to
make war more likely, is not an act of prophetic witness but a moral absurdity."5 Similarly, the pacifist's commitment to
nonviolence can inspire others to abandon force and coercion, although pacifists will not necessarily be the only ones to urge a course of
nonviolence. A social movement too weak to prevail in armed conflict, such as Solidarity in Poland, often will opt for nonviolent tactics of resistance for strictly
prudential reasons. Indeed, I would argue, a prudential criterion should always be employed . The decision whether to choose a nonviolent response
should be made within the context of likely consequences. Pacifists may choose nonviolence as an absolute moral imperative, but they
should not mislead others into thinking that nonviolence will stop all acts of aggression and evil. To do otherwise leads to follies
like Gandhi's advice to the Jews of Europe to use satyagraha to prevent the Nazis from carrying out their plan to destroy the
Jewish people. The pacifist is entitled to participate in the political process and to propose policies like any other citizen. He should recognize, however, that
when he enters the policy arena he must adopt standards of judgment distinct from those he applies in his personal life . He
should not urge a course of action that, if implemented, would leave his country undefended or would tip the balance of power
in the world in favor of expansionist and aggressor nations. It may be noble , Reinhold Niebuhr argued during World War II, for an
individual to sacrifice his life rather than to participate in the defense of order and justice. But one cannot ignore the "distinction between an
individual act of self-abnegation and a policy of submission to injustice, whereby lives and interests other than one's own are
defrauded or destroyed."6 Individual perfection is not a basis on which to build a political platform. Pacifists have every right to avoid the
moral dilemmas posed by the world of statesmanship and statecraft and to seek individual salvation through ethical absolutism
and purity, but they have no right to sacrifice others for the attainment of this vocation. In the best of all possible worlds,
pacifist activity could be both morally pure and politically relevant. In the real world, that is usually not possible . When pacifists
present their language of the heart as a political alternative to the pressures and compromises of the political order, they, as
Niebuhr noted, "invariably betray themselves into a preference for tyranny. " For the moral ambiguities of history and the world of politics,
Niebuhr insisted, ambiguous methods and ambiguous answers are required. "Let those who are revolted by such ambiguities have the decency and courtesy to
retire to the monastery where the medieval perfectionists found their asylum."7 No one expects pacifists to be active supporters of nuclear
deterrence, of the use of force against terrorists, or even of military aid to weak regimes facing the threat of foreign-sponsored subversion. But neither
should pacifists obstruct all such policies that the democratically elected government of the United States pursues in order to
assure its own survival as a free society. When the pacifist's conscience does not allow him to support policies that utilize force
or the threat of force, the proper course for him is to remain silent, to abstain from taking a stand on that policy. A historical
precedent for such a stance is the withdrawal of Quaker politicians from the government of the province of Pennsylvania in 1756 because they wanted neither
to interfere with nor be party to war against the Indians. To prevent misunderstandings, I repeat the central point of my argument: I am not suggesting
that pacifists stop being citizens; I am suggesting that when pacifists act as citizens, they should accept the test of consequences
to which all public policies must be subject. Pacifists in World War II accepted this verity. They recognized that not only would it have
been undemocratic to try to stop a war that their nations had democratically decided to wage, but that to do so would help bring about the triumph of a political
system of unparalleled evil. I argue that pacifists should return to this view of their role in a democratic society. They should look at the foreseeable
consequences of their actions. For example, they should take into account the fact that their pressure for disarmament in the Western democracies has no
counterpart in the Soviet Union. (18-20)
ENDI 2010 124
Security K Wave 1

2AC IMPACT CALC – AT: VALUE TO LIFE


Sanctity more important than quality of life
Federer 2003 [William, a best-selling author and president of Amerisearch Inc. 10-18
http://www.worldnetdaily.com/news/article.asp?ARTICLE_ID=35138]

Even before the rise of Adolf Hitler's Third Reich, the way for the gruesome Nazi Holocaust of human
extermination and cruel butchery was being prepared in the 1930 German Weimar Republic through the medical establishment
and philosophical elite's adoption of the "quality of life" concept in place of the "sanctity of life." The Nuremberg
trials, exposing the horrible Nazi war crimes, revealed that Germany's trend toward atrocity began with their
progressive embrace of the Hegelian doctrine of "rational utility," where an individual's worth is in
relation to their contribution to the state, rather than determined in light of traditional moral, ethical
and religious values. This gradual transformation of national public opinion, promulgated through media and education, was
described in an article written by the British commentator Malcolm Muggeridge entitled "The Humane Holocaust" and in an article written
by former United States Surgeon General, C. Everett Koop, M.D., entitled "The Slide to Auschwitz," both published in The Human Life
Review, 1977 and 1980 respectively. Muggeridge stated: "Near at hand, we have been accorded, for those that have eyes to see, an object
lesson in what the quest for 'quality of life' without reference to 'sanctity of life' can involve ... [namely] the great Nazi Holocaust, whose TV
presentation has lately been harrowing viewers throughout the Western world. In this televised version, an essential consideration has been
left out – namely, that the origins of the Holocaust lay, not in Nazi terrorism and anti-Semitism, but in pre-Nazi Weimar Germany's
acceptance of euthanasia and mercy-killing as humane and estimable. ... "It took no more than three decades to transform a war crime into
an act of compassion, thereby enabling the victors in the war against Nazism to adopt the very practices for which the Nazis had been
solemnly condemned at Nuremberg." The transformation followed thus: The concept that the elderly and terminally ill should have the right
to die was promoted in books, newspapers, literature and even entertainment films, the most popular of which were entitled "Ich klage an (I
accuse)" and "Mentally Ill."
One euthanasia movie, based on a novel by a National Socialist doctor, actually won a prize at the world-famous Venice Film Festival!
Extreme hardship cases were cited, which increasingly convinced the public to morally approve of euthanasia. The medical
profession gradually grew accustomed to administering death to patients who, for whatever reasons,
felt their low "quality of life" rendered their lives not worth living, or as it was put, lebensunwerten Lebens, (life
unworthy of life). In an Associated Press release published in the New York Times Oct. 10, 1933, entitled "Nazi Plan to Kill Incurables to End
Pain; German Religious Groups Oppose Move," it was stated: "The Ministry of Justice, in a detailed memorandum explaining the Nazi aims
regarding the German penal code, today announced its intentions to authorize physicians to end the sufferings of the incurable patient. The
memorandum ... proposed that it shall be possible for physicians to end the tortures of incurable patients, upon request, in the interest of
true humanity. "This proposed legal recognition of euthanasia – the act of providing a painless and peaceful death – raised a number of
fundamental problems of a religious, scientific and legal nature. The Catholic newspaper Germania hastened to observe: 'The Catholic faith
binds the conscience of its followers not to accept this method.' ... In Lutheran circles, too, life is regarded as something that God alone can
take. ... Euthanasia ... has become a widely discussed word in the Reich. ... No life still valuable to the State will be wantonly destroyed."
Nationalized health care and government involvement in medical care promised to improve the public's "quality of life." Unfortunately, the
cost of maintaining government medical care was a contributing factor to the growth of the national debt, which reached astronomical
proportions. Double and triple digit inflation crippled the economy, resulting in the public demanding that government cut expenses. This
precipitated the 1939 order to cut federal expenses. The national socialist government decided to remove "useless" expenses from the budget,
which included the support and medical costs required to maintain the lives of the retarded, insane, senile, epileptic, psychiatric patients,
handicapped, deaf, blind, the non-rehabilitatable ill and those who had been diseased or chronically ill for five years or more. It was labeled
an "act of mercy" to "liberate them through death," as they were viewed as having an extremely low "quality of life," as well as being a tax
burden on the public. The public psyche was conditioned for this, as even school math problems compared distorted medical costs incurred
by the taxpayer of caring for and rehabilitating the chronically sick with the cost of loans to newly married couples for new housing units.
The next whose lives were terminated by the state were the institutionalized elderly who had no relatives and no financial resources. These
lonely, forsaken individuals were needed by no one and would be missed by no one. Their "quality of life" was considered low by everyone's
standards, and they were a tremendous tax burden on the economically distressed state. The next to be eliminated were the parasites on the
state: the street people, bums, beggars, hopelessly poor, gypsies, prisoners, inmates and convicts. These were socially disturbing individuals
incapable of providing for themselves whose "quality of life" was considered by the public as irreversibly below standard, in addition to the
fact that they were a nuisance to society and a seed-bed for crime. The liquidation grew to include those who had been unable to work, the
socially unproductive and those living on welfare or government pensions. They drew financial support from the state, but contributed
nothing financially back. They were looked upon as "useless eaters," leeches, stealing from those who worked hard to pay the taxes to
support them. Their unproductive lives were a burden on the "quality of life" of those who had to pay the taxes. The next to be
eradicated were the ideologically unwanted, the political enemies of the state, religious extremists and
those "disloyal" individuals considered to be holding the government back from producing a society
which functions well and provides everyone a better "quality of life." The moving biography of the imprisoned
Dietrich Bonhoffer chronicled the injustices. These individuals also were a source of "human experimental material," allowing military
medical research to be carried on with human tissue, thus providing valuable information that promised to improve the nation's health.
Continues…
ENDI 2010 125
Security K Wave 1

AT: VALUE TO LIFE


Continued…
Finally, justifying their actions on the purported theory of evolution, the Nazis considered the German, or "Aryan," race as "ubermenschen,"
supermen, being more advanced in the supposed progress of human evolution. This resulted in the twisted conclusion that all other races,
and in particular the Jewish race, were less evolved and needed to be eliminated from the so-called "human gene pool," ensuring that future
generations of humans would have a higher "quality of life." Dr. Koop stated: "The first step is followed by the second step. You can say that
if the first step is moral then whatever follows must be moral. The important thing, however, is this: Whether you diagnose the first step as
being one worth taking or being one that is precarious rests entirely on what the second step is likely to be. ... I am concerned about this
because when the first 273,000 German aged, infirm and retarded were killed in gas chambers there was no outcry from that medical
profession either, and it was not far from there to Auschwitz." Can this holocaust happen in America? Indeed, it has already begun. The idea
of killing a person and calling it "death with dignity" is an oxymoron. The "mercy-killing" movement puts us on the same path as pre-Nazi
Germany. The "quality of life" concept, which eventually results in the Hegelian utilitarian attitude of a person's worth being based on their
contribution toward perpetuating big government, is in stark contrast to America's founding principles. This philosophy which
lowers the value of human life, shocked attendees at the Governor's Commission on Disability, in Concord, N.H., Oct. 5, 2001, as
they heard the absurd comments of Princeton University professor Peter Singer. The Associated Press reported Singer's comments: "I do
think that it is sometimes appropriate to kill a human infant," he said, adding that he does not believe a newborn has a right to life until it
reaches some minimum level of consciousness. "For me, the relevant question is, what makes it so seriously wrong to take a life?" Singer
asked. "Those of you who are not vegetarians are responsible for taking a life every time you eat. Species is no more relevant than race in
making these judgments." Singer's views, if left unchecked, could easily lead to a repeat of the atrocities of Nazi
Germany, if not something worse.
ENDI 2010 126
Security K Wave 1

AT: STRUCTURAL VIOLENCE IMPACT


Structural violence obscures analysis necessary to reduce poverty and violence-
Boulding, 1977 [Kenneth Prof Univ. of Michigan and UC Boulder, Journal of Peace Research; 14; 75 p.
Boulding p. 83-4

Finally, we come to the great Galtung metaphors of ‘structural


violence‘ and ‘positive peace‘. They are metaphors rather than models, and for that
very reason are suspect. Metaphors always imply models and metaphors have much more persuasive power than models
do, for models tend to be the preserve of the specialist. But when a metaphor implies a bad model it can be very dangerous,
for it is both persuasive and wrong. The metaphor of structural violence I would argue falls right into this category. The metaphor is that poverty, deprivation,
ill health, low expectations of life, a condition in which more than half the human race lives, is ‘like‘ a thug beating up the victim and taking his money away from him in the street, -or it is ‘like‘ a conqueror stealing
the land of the people and reducing them to slavery. The implication is that poverty and its associated ills are the fault of the thug or the conqueror
and the solution is to do away with thugs and conquerors. While there is some truth in the metaphor, in the modem world at least there
is not very much. Violence, whether of the streets and the home, or of the guerilla, of the police, or of the armed forces, is a very different phenomenon from
poverty. The processes which create and sustain poverty are not at all like the processes which create and sustain violence,
although like everything else in the world, everything is somewhat related to everything else. There is a very real problem of the structures which lead to violence,
but unfortunately Galtung‘s metaphor of structural violence as he has used it has diverted attention from this problem.
Violence in the behavioral sense, that is, somebody actually doing damage to somebody else and trying to make them worse off, is a ‘threshold‘ phenomenon, rather like
the boiling over of a pot. The temperature under a pot can rise for a long time without its boiling over, but at some threshold boiling over will
take place. The study of the structures which underlie violence are a very important and much neglected part of peace research and
indeed of social science in general. Threshold phenomena like violence are difficult to study because they represent ‘breaks‘ in the system
rather than uniformities. Violence, whether between persons or organizations, occurs when the ‘strain‘ on a system is too great for its ‗~s~trength‘. The
metaphor here is that violence is like what happens when we break a piece of chalk. Strength and strain, however, especially in social
systems, are so interwoven historically that it is very difficult to separate them. The diminution of violence involves two
possible strategies, or a mixture of the two; one is the increase in the strength of the system, ~the other is the diminution of the strain.
The strength of systems involves habit, culture, taboos, and sanctions, all these things, which enable a system to stand Increasing strain without breaking down into violence. The strains on the
system are largely dynamic in character, such as arms races, mutually stimulated hostility, changes in relative economic
position or political power, which are often hard to identify. Conflict of interest are only part of the strain on a system, and not always the most important part. It is very
hard for people to know their interests, and misperceptions of interests take place mainly through the dynamic processes, not through the structural ones. It is only perceptions of interest
which affect people‘s behavior, not the ‘real‘ interests, whatever these may be, and the gap between perception and reality can
be very large and resistant to change. However, what Galitung calls structural violence (which has been defined by one unkind
commentator as anything that Galltung doesn‘~t like) was originally defined as any unnecessarily low expectation of life, an
that assumption that anybody who dies before the allotted span has been killed, however unintentionally and unknowingly,
by somebody else. The concept has been expanded to include all the problems off poverty, destitution, deprivation, and
misery. These are enormously real and are a very high priority for research and action, but they belong to systems which are
only peripherally related to the structures which, produce violence. This is not to say that the cultures of violence and the
cultures of poverty are not sometimes related, though not all poverty cultures are culture of violence, and certainly not all cultures of violence are poverty cultures. But the
dynamics of poverty and the success or failure to rise out off ‘it are of a complexity far beyond anything which the metaphor
of structural violence can offer. While the metaphor of structural violence performed a ‘service in calling attention to a
problem, it may have done a disservice in preventing us from finding the answer.

The alternative can‘t change the system and negative peace outweighs
Boulding, [1990 Kenneth E.Prof UC Boulder Conflict: Readings in Management and Resolution, Ed: John Burton, pg. 40-41 ]
I must confess that when I first became involved with the peace research enterprise 25 years ago I had hopes that it might produce something like the
Keynesian revolution in economics, which was the result of some rather simple ideas that had never really been thought out clearly before (though they had been anticipated by Malthus and
others), coupled with a substantial improvement in the information system with the development of national income statistics which reinforced this new theoretical framework. As a result, we have had in a
single generation a very massive change in what might be called the "conventional wisdom" of economic policy , and even though this
conventional wisdom is not wholly wise, there is a world of difference between Herbert Hoover and his total failure to deal with the Great Depression, simply because of everybody's ignorance, and the moderately
skillful handling of the depression which followed the change in oil prices in 1-974, which, compared with the period 1929 to 1932, was little more than a bad cold compared with a galloping pneumonia. In the
international system, however, there has been only glacial change in the conventional wisdom. There has been some improvement. Kissinger was
an improvement on John Foster Dulles. We have had the beginnings of detente, and at least the possibility on the horizon of stable peace between the United States and the Soviet Union, indeed in the whole
temperate zone-even though the tropics still remain uneasy and beset with arms races, wars, and revolutions which we cannot really afford. Nor can we pretend that peace around the temperate zone is stable
The qualitative arms race goes on and could easily take us over the cliff . The record of peace
enough so that we do not have to worry about it.
research in the last generation, therefore, is one of very partial success. It has created a discipline and that is something of long-run consequence, most certainly for the good.
It has made very little dent on the conventional wisdom of the policy makers anywhere in the world. It has not been able to prevent
an arms race, any more, I suppose we might say, than the Keynesian economics has been able to prevent inflation. But whereas inflation is an inconvenience, the arms race may well be
another catastrophe. Where, then, do we go from here? Can we see new horizons for peace and conflict research to get it out of the doldrums in which it has been now for almost ten years? The
challenge is surely great enough. It still remains true that war, the breakdown of Galtung's "negative peace," remains the greatest clear and present
danger to the human race, a danger to human survival far greater than poverty, or injustice, or oppression, desirable and
necessary as it is to eliminate these things. (347-8)
ENDI 2010 127
Security K Wave 1

2AC-PERMUTATION
Their K incorrectly essentializes realism-it’s not a static entity, it can incorporate critical
insights
COZETTE 2008 [MURIELLE BA (Hons) (Sciences Po Paris), MA (King's College London), MA (Sciences Po Paris), PhD (LSE) is a John Vincent Postdoctoral fellow in
the Department of International Relations. Review of International Studies (2008), 34, 5–27]

This article concentrates on Morgenthau‘s views on the ethics of scholarship and argues that all his works must be read in the light of his central goal:
speaking truth to power. Morgenthau wrote at length, and held very specific views about, the role and function of scholars in
society. It is therefore legitimate to claim that, as a scholar himself, Morgenthau attempted to live up to his very demanding definition of scholarly activity,
and his assertion that scholars have the moral responsibility to speak truth to power informed all his major works. While Morgenthau‘s conception of the ethics
of scholarship is generally ignored or neglected, it is, however, indispensable to take it into account when approaching his writings. Indeed, it demonstrates that
for Morgenthau, a realist theory of international politics always includes two dimensions , which are intrinsically linked:
it is supposed to explain international relations, but it is also, fundamentally, a normative and critical project which
questions the existing status quo. While the explanatory dimension of realism is usually discussed at great length, its
critical side is consistently – and conveniently – forgotten or underestimated by the more recent, self-named ‗critical‘
approaches. However diverse these recent approaches may be in their arguments, what unites them all is what they are
supposedly critical of: the realist tradition. The interpretation they provide of realism is well known, and rarely
questioned. Although it is beyond the scope of this article to review it at length, it is worth stressing some of the main features which are constantly
emphasised. First then, realism is a state-centric approach, by which is meant that it stresses the importance of anarchy and the struggle for power
among states. From this, most critical approaches jump to the conclusion that realism is therefore strikingly ill-equipped to deal
with the contemporary era where the state is increasingly regarded as outdated and/or dangerous, because it stands in the path of different, more
emancipatory modes of political organisation. Realism, it is also argued, pretends to be objective and to depict ‗things as they are‘: but this
cannot obscure the fact that theories are never value-neutral and constitute the very ‗reality‘ they pretend to ‗describe‘. This leads to the
idea that realism is in fact nothing but conservatism: it is portrayed as the voice of (great) powers, with the effect of reifying (and therefore
legitimising) the existing international order. This explains why Rothstein can confidently argue that realism ‗is . . . implicitly a conservative doctrine attractive
to men concerned with protecting the status quo‘, and that it is a ‗deceptive and dangerous‘ theory, not least because it ‗has provided the necessary psychological
and intellectual support to resist criticism, to persevere in the face of doubt, and to use any means to outwit or to dupe domestic dissenters‘.2 Such
views
represent a fundamental misunderstanding of the realist project, but are nonetheless widely accepted as commonsense in the
discipline. A typical example of this is the success of Cox‘s famous distinction between ‘problem solving’ and ‘critical’ theory.
Unsurprisingly, realism is the archetypal example of a problem-solving theory for Cox. His account of the realist tradition sweepingly equates Morgenthau and
Waltz, who are described as ‗American scholars who transformed realism into a form of problem-solving theory‘.3 Thereafter in his famous article ‗Social Forces,
States and World Orders‘, Cox refers to the works of both scholars by using the term ‗neo-realism‘. Problem solving theory (and therefore realism) ‗takes the
world as it finds it . . . as the given framework for action‘, while by contrast, the distinctive trait of ‗critical theory‘ is to ‗stand apart from the prevailing order of
the world and asks how that order came about‘.4 Problem-solving theory, says Cox, ‗serves particular national sectional or class interests, which are comfortable
within the given order‘, which therefore means that its purpose is ‗conservative‘.5 Problem-solving theory also pretends to be ‗value free‘, while Cox is keen to
remind his reader that it contains some ‗latent normative elements‘, and that its ‗non normative quality is however, only superficial‘.6 By contrast to what Cox
presents as a problem-solving theory, being ‗critical‘ in IR means being openly normative, challenging the status quo, and seeking to advance human
emancipation( s), however this concept is to be defined.7 The picture Cox proposes is therefore simple: critical theory is named as such because of its
commitment to ‗bringing about an alternative order‘ and because of its openly normative stance, while realism, by contrast, is
presented as a
theory which in effect reproduces and ‗sustain[s] the existing order‘.8 To be fair, not all critical theorists promote such a simplistic
vision of what realism stands for – Cox himself, in some of his later works, recognised that classical realism possesses an undeniable critical dimension. In 1992,
providing a more nuanced analysis of the school, he thus accepted that ‗classical realism is to be seen as a means of empowerment of the less powerful, a means
of demystification of the manipulative instruments of power‘.9 He did not, however, investigate the critical dimension of realism in much depth, and failed to
identify its emancipatory dimension. Other critical theorists demonstrate an awareness of the richness and subtlety of Morgenthau‘s ideas. The best example
remains Ashley‘s famous piece on the poverty of neorealism, where he justly argues that the triumph of the latter has obscured the insights provided by classical
realism. Ashley‘s analysis remains, however, problematic as his interpretation of Morgenthau does not identify all the critical dimensions of his writings, and
ultimately continues to present classical realism as the ‗ideological apparatus‘ of one particular ruling group, that of statesmen, which remains essentially
incapable of realising its own limitations. As he writes: It is a tradition whose silences and omissions, and failures of self critical nerve join it in secret complicity
with an order of domination that reproduces the expectation of inequality as a motivating force, and insecurity as an integrating principle. As the ‗organic
intellectuality of the world wide public sphere of bourgeois society, classical realism honors the silences of the tradition it interprets and participates in
exempting the ‗private sphere‘ from public responsibility.10 (emphasis added) The ‗picture‘ of classical realism which is provided by Ashley therefore does
not adequately capture its inherent critical dimension, as it ultimately presents it as reproducing the existing order and silencing dissent.
Cox‘s distinction clearly echoes the now classic one between ‗orthodox‘ and ‗critical‘ approaches (a label broad enough to include the self-named Critical Theory,
Feminism, Normative theory, Constructivism and Post-Structuralism). The diversity
of critical approaches should not obscure the fact
that crucially, what allows them to think of themselves as critical is not simply a set of epistemological (usually
‗post-positivist‘) or ontological assumptions they may share. It is also, fundamentally, the image they think lies in the mirror
when they turn it to realism. In most cases then, it seems to be enough to oppose a simplistic picture of realism like that
provided by Cox to deserve the much coveted label ‗critical‘. This leads to the idea that it is impossible to be at the same time
a realist scholar and critical, as the two adjectives are implicitly presented as antithetical. This clearly amounts to an insidious
high-jacking of the very adjective ‗critical‘, which more often than not merely signals that one does not adopt a realist approach. The

Continues…
ENDI 2010 128
Security K Wave 1

2AC PERMUTATION
continued…
meaning of the adjective is therefore presented as self-evident, and realism is denied any critical dimension. This is highly
problematic as this reinforces a typical ‗self-righteousness‘ from these ‗critical‘ approaches, which tend to rely on a
truncated and misleading picture of what realism stands for and conveniently never properly engage with realists‘
arguments. The fact that Waltz is always the primary target of these approaches is no coincidence: this article demonstrates that realism as expressed by
Morgenthau is at its very core a critical project. In order to challenge the use of the adjective ‗critical‘ by some who tend to think of themselves as such simply by
virtue of opposing what they mistakenly present as a conservative theoretical project, the article highlights the central normative and critical dimensions
underlying Morgenthau‘s works. It does so by assessing his views about the ethics of scholarship. The article is divided into two parts. First, it investigates
Morgenthau‘s ideal of the scholarly activity, which rests upon a specific understanding of the relationship between truth and power. Second, it focuses on some
features which, for Morgenthau, constitute a ‗betrayal‘ of this ideal (a term he borrowed from Julien Benda). The article demonstrates that contrary to
the
common interpretation of realism as a theoretical outlook that holds an implicit and hidden normative
commitment to the preservation of the existing order, Morgenthau‘s formulation of realism is rooted in his claim
that political science is a subversive force, which should ‘stir up the conscience of society’, and in doing so, challenge the
status quo. For Morgenthau, IR scholars have the responsibility to seek truth, against power if needed, and then to speak this truth to power even though
power may try to silence or distort the scholar‘s voice.11 Giving up this responsibility leads to ideology and blind support for power, which is something that
Morgenthau always saw as dangerous, and consistently opposed. His commitment to truth in turn explains why, according to him, political science is always, by
definition, a revolutionary force whose main purpose is to bring about ‗change through action‘. In
complete contrast to what ‗critical
approaches‘ consistently claim, the realist project is therefore best understood as a critique of the powers-that-
be.
ENDI 2010 129
Security K Wave 1

CRITICAL REALISM PERM


Adopting critical realism gets past exclusive focus on discourse or events to address the structural possibilities for
the status quo
Patomaki and Wight 2000 [Heikki, Professor of International Relations at the University of Helsinki, Colin, professor of
political science at University of Wales, ―After Postpositivism? The Promises of Critical Realism,‖ International Studies
Quarterly, Vol. 44, No. 2 (Jun., 2000), pp. 213-237, jstor]

Critical Realism Every theory of knowledge must also logically presuppose a theory of what the world is like (ontology) for knowledge (epistemology) to
be possible. Or as Bhaskar, inverting a Hegelian aphorism, puts it, "all philosophies, cognitive discourses and practical activities presuppose a realism-in
the sense of some ontology or general account, of the world-of one kind or another" (Bhaskar, 1989:2). The question is not of whether to be a
realist, but of what kind of realist to be. We have attempted to show how the boundaries of both negativity and boredom share a common
"problem-field," which is structured by various forms of anti- realism/scepticism. We have also argued that those beyond the boundary of boredom tend to
be empirical realists and those beyond the boundary of nega- tivity tend towards linguistic realism. We want to now situate a different "problem- field":
one that takes the possibility of a deeper realism to be a condition of possibility for both empirical and linguistic realism. The form of realism we advocate
can be called critical realism (for essential readings, see Archer et al., 1998). There are two distinct ways in which critical realism differs
from empirical and linguistic realism. First, according to critical realism the world is composed not only of events, states of
affairs, experiences, impressions, and discourses, but also of underlying structures, powers, and tendencies that exist,
whether or not detected or known through experience and/or discourse . For critical realists this under- lying reality provides
the conditions of possibility for actual events and perceived and/or experienced phenomena. According to critical realists, empirical
and linguistic realists collapse what are, in effect, different levels of reality into one (Bhaskar, 1975:56). For both the underlying reality that makes
experience pos- sible and the course of events that is not experienced/spoken are reduced to what can be experienced or become an object of discourse.
Second, for critical realism the different levels may be out of phase with each other. What we mean is that although the underlying level may
possess certain powers and tendencies, these are not always manifest in experience, or even for that matter realized. A nuclear
arsenal has the power to bring about vast destruc- tion and this power exists irrespective of being actualized. Moreover, this
power is itself based on more than that which we directly experience. The conception we are proposing is that of a world
composed, in part, of complex things (includ- ing systems and complexly structured situations) that, by virtue of their struc- tures,
possess certain powers, potentials, and capacities to act in certain ways even if those capacities are not always realized. The
world on this view consists of more than the actual course of events and experiences and/or discourses about them .
ENDI 2010 130
Security K Wave 1

2AC- ALT FAILS

The alternative fails to offer a solution to policy-makers—this makes solvency impossible.


Walt 99 (Stephen, Professor of International Affairs at Harvard University, ―Rigor or Rigor Mortis? Rational Choice and Security Studies,‖ International
Security, 23(4),)
Taken together, these characteristics help explain why recent formal work has had relatively little to say about important real-world security
issues. Although formal techniques produce precise, logically consistent arguments, they often rest on unrealistic assumptions and the
results are rarely translated into clear and accessible conclusions . And because many formal conjectures are often untested,
policymakers and concerned citizens have no way of knowing if the arguments are valid . In this sense, much of the recent formal work in
security studies reflects the "cult of irrelevance " that pervades much of contemporary social science. Instead of using their
expertise to address important real-world problems, academics often focus on narrow and trivial problems that may impress their
colleagues but are of little practical value. If formal theory were to dominate security studies as it has other areas of political science,
much of the scholarship in the field would likely be produced by people with impressive technical skills but little or no substantive
knowledge of history, politics, or strategy.[111] Such fields are prone to become "method-driven" rather than "problem-driven,"
as research topics are chosen not because they are important but because they are amenable to analysis by the reigning methode du jour.[112] Instead of
being a source of independent criticism and creative, socially useful ideas, the academic world becomes an isolated community engaged solely
in dialogue with itself.[113] Throughout most of the postwar period, the field of security studies managed to avoid this danger. It has been
theoretically and methodologically diverse, but its agenda has been shaped more by real-world problems than by methodological
fads. New theoretical or methodological innovations have been brought to bear on particular research puzzles, but the field as a whole has retained
considerable real-world relevance. By contrast, recent formal work in security studies has little to say about contemporary security issues.
Formal rational choice theorists have been largely absent from the major international security debates of the past decade (such as the nature of the post-Cold
War world; the character, causes, and strength of the democratic peace; the potential contribution of security institutions; the causes of ethnic conflict; the
future role of nuclear weapons; or the impact of ideas and culture on strategy and conflict). These debates have been launched and driven primarily by scholars
using nonformal methods, and formal theorists have joined in only after the central parameters were established by others.[114] Thus one of the main
strengths of the subfield of security studies--namely, its close connection to real-world issues--could be lost if the narrow tendencies of the
modeling community took control of its research agenda .

The alternative can‘t address a root cause or end enemy creation- it only causes war
Andrew Sullivan, PhD Harvard, 1-15-3 " Sheryl Crow, brain-dead peacenik in sequins. ―
http://www.salon.com/opinion/sullivan/2003/01/15/crow/index.html
One is also required to ask: If war is "not the answer," what exactly is the question? I wonder if, in her long interludes of geopolitical analysis, Ms.
Crow even asks herself that. Perhaps if she did -- let's say the question is about the threat of weapons of mass destruction in the hands of
terrorists -- we might have an inkling about what her "answer" might actually be. Mercifully, Ms. Crow provides us with what she believes is an argument. Are
you sitting down? Here it comes: "I think war is based in greed and there are huge karmic retributions that will follow. I think war is never the answer to solving
any problems. The best way to solve problems is to not have enemies." Let's take this bit by bit. "War is based in greed." Some wars, surely. The pirate
wars of the 17th century. Saddam's incursion into Kuwait. Early British forays in the Far East and India. But all wars? The United States' intervention in
the Second World War? The Wars of Religion in the 17th century? Many wars are fueled by nationalism, or by ideology, or by
expansionism. And many wars have seen their protagonists not enriched but impoverished. Take Britain's entry into the war against Nazi
Germany. It would have been far more lucrative for the Brits to have made a deal with Hitler, to preserve their wealth and empire. Instead, they waged war, lost
their entire imperial project and ransacked their own domestic wealth. Where would that fit into Ms. Crow's worldview? And then there's the concept of a just
war -- wars that have to be fought to defeat a greater evil. Wars of self-defense. Wars of prevention. Wars against tyrants. Ms. Crow's remarks seem to
acknowledge no such distinction. Does she believe that removing Hitler from power solved nothing? That preventing further genocide in the
Balkans solved nothing? That ending 50 years of Soviet tyranny meant nothing? Apparently so. There's only one word for this kind of
argument: Asinine. Then we have this wonderful insight: "The best way to solve problems is to not have enemies." Wow. Like,
wow. Like, war. It's bad. Bad karma. But, ahem, what if you have no choice in the matter? What if an enemy decides, out of hatred or
fanaticism or ideology, simply to attack you? I'm not sure where Ms. Crow was on Sept. 11, 2001. But the enemy made its point palpably clear. Does
wishing that these crazed religious nuts were not our enemies solve any problems? I'm taking her too seriously, of course. I should ignore
her. But the "antiwar" movement (I put it in quotation marks because any kind of appeasement this time will only make a bloodier future war
inevitable) is happy to use celebrities for its own purposes. And so their presence in the debate has to be acknowledged, if only to be decried. So let's decry
this moronic celebrity convergence. The weak arguments of the appease-Saddam left just got a little weaker. And the karmic retributions are gonna be harsh,
man. Way harsh.
ENDI 2010 131
Security K Wave 1

REALISM GOOD
Abandoning realism doesn’t eliminate global violence—alternative worldviews will be just as
violent or worse
O'Callaghan, 02 (Terry , lecturer in the school of International Relations at the University of South Australia, International
Relations and the third debate, ed: Jarvis, 2002, p. 79-80)
In fact, if we explore the depths of George's writings further, we find remarkable brevity in their scope, failing to engage with practical issues beyond platitudes
and homilies. George, for example, is concerned about the violent, dangerous and war-prone character of the present international system. And rightly so. The
world is a cruel and unforgiving place, especially for those who suffer the indignity of human suffering beneath tyrannous
leaders, warrior states, and greedy self-serving elites. But surely the problem of violence is not banished from the international
arena once the global stranglehold of realist thinking is finally broken? It is important to try to determine the levels of violence that might be
expected in a nonrealist world. How will internecine conflict be managed? How do postmodernists like George go about managing conflict
between marginalized groups whose "voices" collide? It is one thing to talk about the failure of current realist thinking, but there
is absolutely nothing in George's statements to suggest that he has discovered solutions to handle events in Bosnia, the Middle
East, or East Timor. Postmodern approaches look as impoverished in this regard as do realist perspectives. Indeed, it is interesting to note that George
gives conditional support for the actions of the United States in Haiti and Somalia "because on balance they gave people some hope where there was none"
(George, 1994:231). Brute force, power politics, and interventionism do apparently have a place in George's postmodem world. But even so, the Haitian and
Somalian cases are hardly in the same intransigent category as those of Bosnia or the Middle East. Indeed, the Americans pulled out of Somalia as soon as
events took a turn for the worse and, in the process, received a great deal of criticism from the international community. Would George have done the same
thing? Would he have left the Taliban to their devices in light of their complicity in the events of September 11 ? Would he have left the
Somalians to wallow in poverty and misery? Would he have been willing to sacrifice the lives of a number of young men and women (American, Australian,
French, or whatever) to subdue Aidid and his minions in order to restore social and political stability to Somalia? To be blunt, I wonder how much better off
the international community would be if Jim George were put in charge of foreign affairs. This is not a fatuous point. After all, George wants to suggest that
students of international politics are implicated in the trials and tribulations of international politics. All of us should be willing, therefore, to accept such a role,
even hypothetically. I suspect, however, that were George actually to confront some of the dilemmas that policymakers do on a daily
basis, he would find that teaching the Bosnian Serbs about the dangers of modernism, universalism and positivism, and asking
them to be more tolerant and sensitive would not meet with much success . True, it may not be a whole lot worse than current realist
approaches, but the point is that George has not demonstrated how his views might make a meaningful difference. Saying that they
will is not enough, especially given that the outcomes of such strategies might cost people their lives . Nor, indeed, am I asking George
to develop a "research project" along positivist lines. On the contrary, I am merely asking him to show how his position can make a difference
to the "hard cases" in international politics. My point is thus a simple one. Despite George's pronouncements, there is little in his work
to show that he has much appreciation for the kind of moral dilemmas that Augustine wrestled with in his early writings and that confront
human beings every day. Were this the case, George would not have painted such a black-and- white picture of the study of international politics.

No alternative to realism- the alt results in civil war


Hussein Solomon Senior Researcher, Human Security Project, Institute for Defence Policy Published in African
Security Review Vol 5 No 2, 1996 http://www.iss.co.za/pubs/ASR/5No2/5No2/InDefence.html
The post-modern/critical theory challenge to realism has been tested, and proved wanting. Realism
remains the single most reliable analytical framework through which to understand and evaluate
global change. Post-modernism can provide no practical alternatives to the realist paradigm. We know
what a realist world looks like (we are living in one!); but what does a post-modernist world look like?
As long as humanity is motivated by hate, envy, greed and egotism, realism will continue to be
invaluable to the policy-maker and the scholar. In this regard it has to be pointed out that from the end of
World War II until 1992, hundreds of major conflicts around the world have left some twenty million human beings dead.109
Neither has the end of the Cold War showed any sign that such conflict will end. By the end of 1993 a record of 53 wars were
being waged in 37 countries across the globe.110 Until a fundamental change in human nature occurs, realism
will continue to dominate the discipline of international relations. The most fundamental problem
with post-modernism is that it assumes a more optimistic view of human nature. Srebrenica, Bihac,
Tuzla, Zeppa, Goma, Chechnya, Ogoniland, and KwaZulu-Natal all bear testimony to the folly of
such a view.

Vous aimerez peut-être aussi